You are on page 1of 107

MANAGEMENT ADVISORY SERVICES CAPITAL BUDGETING

BASIC CONCEPTS D. Capital budgeting is a bottom-up process, while strategic planning is a top-down process
Capital Budgeting Defined
1
. The capital budget is a(n) Agency Problem in Capital Budgeting
a. Plan to insure that there are sufficient funds available for the operating needs of the 15. The following are agency problems in capital budgeting except:
company. A. Reduced effort C. Empire building
b. Exercise that sets the long-range goals of the company including the consideration of B. Need for good information D. Perks B&M
external influences.
c. Plan that coordinates and communicates a company’s plan for the coming year to all 16. The following are agency problems in capital budgeting except:
departments and divisions. A. Empire building C. Avoiding risks
d. Plan that assesses the long-term needs of the company for plant and equipment B. Entrenching investment D. Reducing forecast bias B&M
purchases. CMA 0695 3-17
17. The following are agency problems in capital budgeting except:
2
. In planning and controlling capital expenditures, the most logical sequence is to begin with A. Avoiding risks
a. Analyzing capital addition proposals. B. The free-rider problem
b. Making capital and expenditure decisions. C. Compensation
c. Analyzing and evaluating all promising alternatives. D. All of the above are agency problems B&M
d. Identifying capital addition projects and other capital needs. CMA 0696 3-11
18. The following are agency problems in capital budgeting except:
21. Which of the following involves significant financial investments in projects to develop new A. Monitoring C. Avoiding risks
products, expand production capacity, or remodel current production facilities? (E) B. Compensation D. Economic value added B&M
a. Capital budgeting c. Master budgeting
b. Working capital d. Project-cost budgeting Horngren Uses
107.A capital budget is used by management to determine
*. The detailed plan for the acquisition and replacement of major portions of property, plant, and Barfield a. b. c. d.
equipment is known as the In what to invest No No Yes Yes
a. capital budget. c. commitments budget. How much to invest No Yes No Yes
b. purchases budget. d. treasury budget. Barfield
3
. Capital budgeting is concerned with
2. _____________ decisions are concerned with the process of planning, setting goals and A. Decisions affecting only capital intensive industries.
priorities, arranging financing, and identifying criteria for making long-term investments. B. Analysis of short-range decisions.
a. Limited resources c. Capital investment C. Analysis of long-range decisions.
b. Sell now or process further d. Make or buy H&M D. Scheduling office personnel in office buildings. Gleim

Capital Budgeting vs. Strategic Planning 4


. Capital budgeting is used for the decision analysis of
3. Which of the following statements regarding capital budgeting and strategic planning is true? A. Adding product lines or facilities. C. Lease-or-buy decisions.
A. Capital budgeting and strategic planning are bottom-up processes B. Multiple profitable alternatives. D. All of the answers are correct. Gleim
B. Capital budgeting and strategic planning are top down processes B&M
C. Capital budgeting is a top-down process, while strategic planning is a bottom-up process 5
. Capital budgeting techniques are least likely to be used in evaluating the (E)
CMA EXAMINATION QUESTIONS Page 1 of 107
MANAGEMENT ADVISORY SERVICES CAPITAL BUDGETING

a. Acquisition of new aircraft by a cargo company. a. identification stage. c. information-acquisition stage.


b. Design and implementation of a major advertising program. b. search stage. d. selection stage. Horngren
c. Trade for a star quarterback by a football team. CMA 0693 4-19
d. Adoption of a new method of allocating non-traceable costs to product lines. Search Stage
24. The stage of the capital budgeting process which explores alternative capital investments that
Application will achieve organization objectives is the (E)
2. A company can replace the machinery currently used to manufacture its product with more a. identification stage. c. information-acquisition stage.
efficient machinery. The new machinery will reduce labor and also will reduce the percentage b. search stage. d. selection stage. Horngren
of spoiled units. It is expected to have a useful life of 5 years. The most appropriate technique
for determining whether or not the company should replace its machinery with the new, more Selection Stage
efficient machinery is: 26. The stage of the capital budgeting process which chooses projects for implementation is the
A. cost-volume-profit analysis C. regression analysis (E)
B. capital-budgeting analysis D. linear programming CIA adapted a. selection stage. c. identification stage.
b. search stage. d. management-control stage. Horngren
Not-for-Profit Entities
10. Not-for-profit entities Appropriation Requests
a. Cannot use capital budgeting techniques because profitability is irrelevant to them. 8. Which of the following statements regarding appropriation requests is true?
b. Cannot use discounted cash flow techniques because the time value of money is A. Usually submitted by head office staff
irrelevant to them. B. Represents the first step in the capital budgeting process
c. Might have serious problems in quantifying the benefits expected from an investment. C. Authorization tends to be reserved for senior management
d. Should use the IRR method to make investment decisions. L&H D. None of the above B&M

Life Cycle Costing 9. Which of the following statements regarding appropriation requests is true?
22. The accounting system that corresponds to the project dimension in capital budgeting is the A. Usually submitted by head office staff
(E) B. Usually requires a detailed analysis using more than one investment criterion
a. net present value method. c. accrual accounting rate of return. C. Usually submitted to a single review at the head office
b. internal rate of return. d. life-cycle costing. Horngren D. None of the above B&M

Stages Implementation & Control Stage


Identification Stage 27. The stage of the capital-budgeting process in which projects get underway and performance is
23. The stage of the capital budgeting process which distinguishes which types of capital monitored is the (E)
expenditure projects are necessary to accomplish organization objectives is the (E) a. implementation and control stage. c. identification stage.
a. identification stage. c. information-acquisition stage. b. search stage. d. management-control stage. Horngren
b. search stage. d. selection stage. Horngren
Acquisition Considerations
Information-Acquisition Stage 6. Effective planning and control is important for the effective administration of a capital
25. The stage of the capital budgeting process which considers the expected costs and the expenditure program because:
expected benefits of alternative capital investments is the (E) A. the long-term commitment increases financial risk
CMA EXAMINATION QUESTIONS Page 2 of 107
MANAGEMENT ADVISORY SERVICES CAPITAL BUDGETING

B. the magnitude of expenditures is substantial and the economic penalties for unwise a. Top management raises the target rate of return.
decisions are usually severe b. The interest rate on long-term debt rises.
C. decisions made in this area provide the structure for operation of the firm c. The income tax rate rises.
D. all of the above Carter & Usry d. The IRS allows companies to expense purchases of fixed assets, instead of depreciating
them over their lives. L&H
7. A company manual used for detailing policies and procedures required for administering the
capital expenditure program should: Qualitative Factors
A. encourage people to work on and submit new ideas 37. Qualitative issues could increase the acceptability of a project under which of the following
B. focus attention on useful analytical tasks conditions?
C. facilitate rapid project development and expeditious review a. The IRR is less than the company’s cutoff rate.
D. all of the above Carter & Usry b. The project has a negative NPV.
c. The payback period is longer than the company’s cutoff period.
14. The normal methods of analyzing investments d. All of the above. L&H
a. Cannot be used by not-for-profit entities.
b. Do not apply if the project will not produce revenues. 40. Qualitative factors can influence managers to
c. Cannot be used if the company plans to finance the project with funds already available a. Accept an investment project having a negative NPV.
internally. b. Reject an investment project having an IRR greater than the company’s cutoff rate.
d. Require forecasts of cash flows expected from the project. L&H c. Raise the “ranking” of an investment project.
d. Take any of the above courses of action. L&H
6
. High-Tech Industries is considering the acquisition of a new state-of-the-art manufacturing
machine to replace a less efficient machine. Hi-Tech has completed a net present value 76. Which of the following statements is (are) true about automation?
analysis and found it to be favorable. Which one of the following factors should not be of a. Automation is inexpensive.
concern to Hi-Tech in its acquisition considerations? b. Automation should be adopted as soon as new technology is available.
A. The availability of any necessary financing. c. Automation should be adopted after a company makes the most efficient use of existing
B. The probability of near-term technological changes to the manufacturing process. technology.
C. The investment tax credit. CMA 1290 4-18 d. All of the above are true H&M
D. Maintenance requirements, warranties, and availability of service arrangements.
Ethical Consideration
8. A number of evaluations of a single capital expenditure proposal may be necessary because 4. Common problems related to ethical considerations in the capital budgeting include all of the
of: following, except:
A. circumstances that change during the time span from the origin of the project idea to its A. superiors and associates sometimes apply pressure to circumvent the approval process
completion B. pressure may exist to write-off or devalue assets below their true value to justify
B. alternative solutions of the problem for which the project is designed replacement
C. assumptions that vary as to the amount and timing of cash flows C. the economic benefit of capital projects may be exaggerated to increase the likelihood of
D. all of the above Carter & Usry approval
D. the accountant may mistakenly go to the individuals involved in the ethical conflict first,
23. Which of the following events is most likely to increase the number of investments that meet a rather than first discussing it with the accounting supervisor
company’s acceptance criteria? (M) E. all of the above are ethical problems related to capital budgeting AICPA adapted
CMA EXAMINATION QUESTIONS Page 3 of 107
MANAGEMENT ADVISORY SERVICES CAPITAL BUDGETING

Post- Investment Audit Expansion Expenditures


69. Comparison of the actual results for a project to the costs and benefits expected at the time 10. In which of the following types of capital expenditure decisions does the basis for a decision
the project was selected is referred to as (E) most markedly shift from cost savings to increased profits and cash flow?
a. the audit trail. c. a postinvestment audit. A. replacement expenditures C. improvement expenditures
b. management control. d. a cost-benefit analysis. Horngren B. expansion expenditures D. allowance expenditures Carter & Usry

10. Post audit is conducted: Improvement Expenditures


A. Before starting a project 11. The capital expenditures in which the benefits are most difficult to quantify are:
B. Before authorizing a project A. replacement expenditures C. improvement expenditures
C. Shortly after a project has started operating B&M B. expansion expenditures D. allowance expenditures Carter & Usry
D. Long after the project has been completed and the salvage has been realized
Not a Capital Expenditure
77. A post audit compares 6. The following capital expenditures may not appear in the capital budget except:
a. estimated benefits and costs with budgeted benefits and cost A. Investment in information technology
b. estimated benefits with estimated costs B. Investment in research and development
c. actual benefits with actual costs C. Investment in training and personal development
d. actual benefits and costs with estimated benefits and costs H&M D. Investment in a new office building B&M

27 The post-audit is used to(E) 7. The following capital expenditures may not appear in the capital budget except:
a. Improve cash flow forecasts. A. Marketing
b. Stimulate management to improve operations and bring results into line with forecasts. B. Training and personnel development
c. Eliminate potentially profitable but risky projects. C. Investment in a new machine
d. Statements a and b are correct. D. Investment in research and development B&M
e. All of the statements above are correct. Brigham
4. Which of the following capital expenditure may not appear in capital budget?
11. A post audit will: A. Investment in a new plant
A. Identify the problem that needs to be fixed B. Investment in a new machine
B. Check the accuracy of the cash flow forecasts C. Investment in information technology
C. Suggest questions that should have been asked before D. All of the above are included in capital budget B&M
D. All of the above B&M
5. Which of the following capital expenditures may not appear in capital budget?
Type of Capital Expenditure A. Investment in a new building
Replacement Expenditures B. Investment in a new machine
9. The following capital expenditures that compare the future costs of the old assets with the C. Investment in research and development
future costs of the new assets as a basis for making a decision are: D. All of the above are included in capital budget B&M
A. replacement expenditures C. improvement expenditures
B. expansion expenditures D. allowance expenditures Carter & Usry
CMA EXAMINATION QUESTIONS Page 4 of 107
MANAGEMENT ADVISORY SERVICES CAPITAL BUDGETING

Types of Projects a. Be different if the project is accepted rather than rejected.


Independent Projects b. Be saved if the project is accepted rather than rejected.
1. ______________ are projects that when accepted or rejected will NOT affect the cash flows of c. Be deductible for tax purposes.
another project. d. Affect net income in the period that they are incurred. L&H
a. Independent projects c. Dependent projects
7
b. Mutually exclusive projects d. Both b and c H&M . Which of the following rules are essential to successful cash flow estimates, and ultimately, to
successful capital budgeting? (M)
Mutually Exclusive Projects a. The return on invested capital is the only relevant cash flow.
4. ______________are projects that when accepted preclude the acceptance of competing b. Only incremental cash flows are relevant to the accept/reject decision.
projects. c. Total cash flows are relevant to capital budgeting analysis and the accept/reject decision.
a. Independent projects c. Dependent projects d. Statements a and b are correct.
b. Mutually exclusive projects d. Both b and c H&M e. All of the statements above are correct. Brigham

Inflation Element Salvage Value


*. The “inflation element” refers to the 60. The relevant terminal disposal price of a machine equals (M)
a. Impact that future price increases will have on the original cost of a capital expenditure. a. the difference between the salvage value of the old machine and the ultimate salvage
b. Fact that the real purchasing power of a monetary unit usually increases over time. value of the new machine.
c. Future deterioration of the general purchasing power of the monetary unit. b. the total of the salvage values of the old machine and the new machine.
d. Future increases in the general purchasing power of the monetary unit. RPCPA 0597 c. the salvage value of the old machine.
d. the salvage value of the new machine. Horngren
Relevant and Irrelevant Costs
28. Capital budgeting emphasizes two factors (E) *. Karen Company is considering replacing an old machine with a new machine. Which of the
a. qualitative and nonfinancial. c. quantitative and financial following items is economically relevant to Karen’s decisions? (M)
b. quantitative and nonfinancial. d. qualitative and financial. Horngren RPCPA 0598 a. b. c. d.
Carrying amount of old machine Yes Yes No No
29. Which of the following are NOT included in the formal financial analysis of a capital budgeting Disposal value of new machine Yes No Yes No
program? (E)
a. Quality of the output c. Cash flow *. You are the treasurer of the Hibang Corp. The company is considering a proposed project
b. Safety of employees d. Neither (a) nor (b) are included Horngren which has an expected economic life of seven years. Net present value is the capital
budgeting technique the president wants you to use. Salvage value of the project would be
56. The focus in capital budgeting should be on (E) (M)
a. the tax consequences of different investment strategies. a. Treated as cash inflow at estimated salvage value.
b. the internal rate of return of different strategies. b. Treated as cash flow at its present value.
c. expected future cash flows that differ between alternatives. c. Irrelevant cash flow item.
d. none of the above. Horngren d. Treated as cash inflow at the future value. RPCPA 1096
32. The only future costs that are relevant to deciding whether to accept an investment are those Working Capital
that will
CMA EXAMINATION QUESTIONS Page 5 of 107
MANAGEMENT ADVISORY SERVICES CAPITAL BUDGETING

66. In the analysis of a capital budgeting proposal, for which of the following items are there no 32. In connection with a capital budgeting project, an investment in working capital is normally
after-tax consequences? (E) recovered
a. Cash flow from operations a. At the end of the project’s life. c. Evenly through the project’s life. L&H
b. Gain or loss on the disposal of the asset b. In the first year of the project’s life. d. When the company goes out of business.
c. Reduction of working capital balances at the end of the useful life of the capital asset
d. There are no after-tax consequences of any of the above. Horngren 12. The proper treatment of an investment in receivables and inventory is to
a. Ignore it.
11. A major difference between an investment in working capital and one in depreciable assets is b. Add it to the required investment in fixed assets.
that (M) c. Add it to the required investment in fixed assets and subtract it from the annual cash
a. An investment in working capital is never returned, while most depreciable assets have flows.
some residual value. d. Add it to the investment in fixed assets and add the present value of the recovery to the
b. An investment in working capital is returned in full at the end of the project’s life, while an present value of the annual cash flows. L&H
investment in depreciable assets has no residual value.
c. An investment in working capital is not tax-deductible when made, not taxable when 34. The cash inflow from the return on an investment in working capital is
returned, while an investment in depreciable assets does allow tax deductions. a. Adjusted for taxes due.
d. Because an investment in working capital is usually returned in full at the end of the b. Discounted to present value.
project’s life, it is ignored in computing the amount of the investment required for the c. Ignored if any depreciable assets also involved in the project have no expected residual
project. L&H value.
d. Not real. L & H, RPCPA 1001
*. Mahlin Movers, Inc. is planning to purchase equipment to make its operations more efficient.
This equipment has an estimated useful life of six years. As part of this acquisition, a 29. XYZ Co. is adopting just-in-time principles. When evaluating an investment project that would
P150,000 investment in working capital is required. In a discounted cash flow analysis, this reduce inventory, how should XYZ treat the reduction?
investment in working capital should be (E) a. Ignore it.
a. Amortized over the useful life of the equipment. b. Decrease the cost of the investment and decrease cash flows at the end of the project’s
b. Disregarded because no cash is involved. RPCPA 1095 life.
c. Treated as a recurring annual cash flow that is recovered at the end of six years. c. Decrease the cost of the investment.
d. Treated as an immediate cash outflow that is recovered at the end of six years. d. Decrease the cost of the investment and increase the cash flow at the end of the project’s
life. L&H
8
. Fast Freight, Inc. is planning to purchase equipment to make its operations more efficient.
This equipment has an estimated life of 6 years. As part of this acquisition, a $75,000 4. Net Working Capital should be considered in project cash flows because:
investment in working capital is anticipated. In a discounted cash flow analysis, the investment A. They are sunk costs
in working capital (E) B. Firms must invest cash in short-term assets to produce finished goods
a. Should be amortized over the useful life of the equipment. C. Firms need positive NPV projects for investment
b. Should be treated as a recurring cash outflow over the life of the equipment. D. None of the above B&M
c. Should be treated as an immediate cash outflow. CMA 0691 4-20
d. Should be treated as an immediate cash outflow recovered at the end of 6 years. Opportunity Costs
10. The value of a previously purchased machine expected to be used by a proposed project is an
example of:
CMA EXAMINATION QUESTIONS Page 6 of 107
MANAGEMENT ADVISORY SERVICES CAPITAL BUDGETING

A. Sunk costs C. Fixed costs b. Subtracted from net income because it is an outflow of cash.
B. Opportunity costs D. None of the above B&M c. Subtracted from net income because it is an expense.
d. Added back to net income because it is not an outflow of cash. RPCPA 1001
Cash Outflows
14. All of the following are common cash outflows from capital expenditure programs, except: 59. Depreciation is usually not considered an operating cash flow in capital budgeting because
A. equipment installation D. increased working capital requirements (E)
B. employee training E. salvage value at the end of the project a. depreciation is usually a constant amount each year over the life of the capital investment.
C. computer programming and fine tuning Carter & Usry b. deducting depreciation from operating cash flows would be counting the lump-sum
amount twice.
33. For investments that have only costs (no revenues or cost savings), an appropriate decision c. depreciation usually does not result in an increase in working capital.
rule is to accept the project that has the d. depreciation usually has no effect on the disposal price of the machine. Horngren
a. Longest payback period.
b. Lower present value of cash outflows. 25. Which of these could occur in practice where the capital expenditure relates to the production
c. Higher present value of future cash outflows. of an established product or service, the demand for which is expected to vary in response to
d. Lowest internal rate of return. L&H temporary changes in consumer taste?
A. perfectly correlated cash flows C. independent cash flows
Cash Inflows B. negative cash flows D. mixed cash flows Carter & Usry
15. As to a capital investment, net cash inflow is equal to the
a. cost savings resulting from the investment. 3. Which of the following is NOT relevant in calculating annual net cash flows for an investment?
b. sum of all future revenues from the investment. a. Interest payments on funds borrowed to finance the project.
c. net increase in cash receipts over cash payments. b. Depreciation on fixed assets purchased for the project.
d. net increase in cash payments over cash receipts. Barfield c. The income tax rate.
d. Lost contribution margin if sales of the product invested in will reduce sales of other
8. Which of the following describes the annual returns that are discounted in determining the products. L&H
NPV of an investment?
a. Net incomes expected to be earned by the project. 39. Which of the following is NOT relevant in calculating net cash flows for Project N?
b. Pre-tax cash flows expected from the project. a. Interest payments on funds that would be borrowed to finance Project N.
c. After-tax cash flows expected from the project. b. Depreciation on assets purchased for Project N.
d. After-tax cash flows adjusted for the time value of money. L&H c. The contribution margin the company would lose if sales of the product introduced by
Project N will reduce sales of other products.
57. Annual after-tax corporate net income can be converted to annual after-tax cash flow by (E) d. The income tax rate applicable to the entity. L&H
a. adding back the depreciation amount.
b. deducting the depreciation amount. Barfield 13. All of the following are common cash inflows related to capital expenditure proposals, except:
c. adding back the quantity (t x depreciation deduction), where t is the corporate tax rate. A. additional revenues from increased sales
d. deducting the quantity [(1- t) x depreciation deduction], where t is the corporate tax rate. B. increased working capital requirements
C. reduction in inventory carrying costs
. To approximate annual cash inflow, depreciation is D. salvage value at the end of the project Carter & Usry
a. Added back to net income because it is an inflow of cash.
CMA EXAMINATION QUESTIONS Page 7 of 107
MANAGEMENT ADVISORY SERVICES CAPITAL BUDGETING

Tax Shield on Depreciation b. When evaluating corporate projects it is important to include all relevant externalities in
8. Which of the following cash flows should be treated as incremental flows when deciding the estimated cash flows.
whether to go ahead with an electric car? c. Interest expenses should be included in project cash flows.
A. The cost of research and development undertaken for developing the electric car in the d. Statements a and b are correct. Brigham
past three years
10
B. The annual depreciation charge . When evaluating potential projects, which of the following factors should be incorporated as
C. The reduction in taxes resulting from the depreciation charges part of a project’s estimated cash flows? (E)
D. Dividend payments B&M a. Any sunk costs that were incurred in the past prior to considering the proposed project.
b. Any opportunity costs that are incurred if the project is undertaken.
Irrelevant Costs c. Any externalities (both positive and negative) that are incurred if the project is undertaken.
9. Money that a firm has already spent or committed to spend regardless of whether a project is d. Statements b and c are correct. Brigham
taken is called:
11
A. Sunk costs C. Fixed costs . Which one of the following statements concerning cash flow determination for capital
B. Opportunity costs D. None of the above B&M budgeting purposes is not correct?
a. Tax depreciation must be considered since it affects cash payments for taxes.
*. In capital expenditures decisions, the following are relevant in estimating operating costs b. Book depreciation is relevant since it affects net income.
except (E) d. Net working capital changes should be included in cash flow forecasts.
a. Future costs. c. Differential costs. c. Sunk costs are not incremental flows and should not be included. CMA 1295 4-11
b. Cash costs. d. Historical costs. RPCPA 1094
12
. A company is considering a new project. The company’s CFO plans to calculate the project’s
58. An example of a sunk cost in a capital budgeting decision for new equipment is (E) NPV by discounting the relevant cash flows (which include the initial up-front costs, the
a. increase in working capital required by a particular investment choice. operating cash flows, and the terminal cash flows) at the company’s cost of capital (WACC).
b. the book value of the old equipment. Which of the following factors should the CFO include when estimating the relevant cash
c. the necessary transportation costs on the new equipment. flows? (E)
d. all of the above are examples of sunk costs. Horngren a. Any sunk costs associated with the project.
b. Any interest expenses associated with the project.
7. The following cash flows should be treated as incremental flows when deciding whether to go c. Any opportunity costs associated with the project.
ahead with an electric car except: (M) d. Statements b and c are correct. Brigham
A. The consequent deduction in sales of the company's existing gasoline models
13
B. The expenditure on new plants and equipment . Which of the following statements is correct? (M)
C. The value of tools that can be transferred from the company's existing plants a. An asset that is sold for less than book value at the end of a project’s life will generate a
D. Interest payment on debt B&M loss for the firm and will cause an actual cash outflow attributable to the project.
b. Only incremental cash flows are relevant in project analysis and the proper incremental
Comprehensive cash flows are the reported accounting profits because they form the true basis for
9
. Which of the following statements is most correct? (E) investor and managerial decisions.
a. When evaluating corporate projects it is important to include all sunk costs in the c. It is unrealistic to expect that increases in net operating working capital that are required
estimated cash flows. at the start of an expansion project are simply recovered at the project’s completion. Thus,
these cash flows are included only at the start of a project.
CMA EXAMINATION QUESTIONS Page 8 of 107
MANAGEMENT ADVISORY SERVICES CAPITAL BUDGETING
18
d. Equipment sold for more than its book value at the end of a project’s life will increase . Sanford & Son Inc. is thinking about expanding their business by opening another shop on
income and, despite increasing taxes, will generate a greater cash flow than if the same property they purchased 10 years ago. Which of the following items should be included in the
asset is sold at book value. Brigham analysis of this endeavor? (M)
a. The property was cleared of trees and brush 5 years ago at a cost of $5,000.
14
. Which of the following statements is most correct? (E) b. The new shop is expected to affect the profitability of the existing shop since some current
a. The rate of depreciation will often affect operating cash flows, even though depreciation is customers will transfer their business to the new shop. Sanford and Son estimate that
not a cash expense. profits at the existing shop will decrease by 10 percent.
b. Corporations should fully account for sunk costs when making investment decisions. c. Sanford & Son can lease the entire property to another company (that wants to grow
c. Corporations should fully account for opportunity costs when making investment flowers on the lot) for $5,000 per year.
decisions. d. Both statements b and c should be included in the analysis. Brigham
d. Statements a and c are correct. Brigham
19
. Pickles Corp. is a company that sells bottled iced tea. The company is thinking about
. Which of the following is not a cash flow that results from the decision to accept a project? (E)
15
expanding its operations into the bottled lemonade business. Which of the following factors
a. Changes in net operating working capital. d. Opportunity costs. should the company incorporate into its capital budgeting decision as it decides whether or not
b. Shipping and installation costs. e. Externalities. to enter the lemonade business? (M)
c. Sunk costs. Brigham a. If the company enters the lemonade business, its iced tea sales are expected to fall 5
percent as some consumers switch from iced tea to lemonade.
16
. Adams Audio is considering whether to make an investment in a new type of technology. b. Two years ago the company spent $3 million to renovate a building for a proposed project
Which of the following factors should the company consider when it decides whether to that was never undertaken. If the project is adopted, the plan is to have the lemonade
undertake the investment? (M) produced in this building.
a. The company has already spent $3 million researching the technology. c. If the company doesn’t produce lemonade, it can lease the building to another company
b. The new technology will affect the cash flows produced by its other operations. and receive after-tax cash flows of $500,000 a year.
c. If the investment is not made, then the company will be able to sell one of its laboratories d. Statements a and c are correct. Brigham
for $2 million.
20
d. Statements b and c should be considered. Brigham . Which of the following statements is correct? (M)
a. In a capital budgeting analysis where part of the funds used to finance the project are
17
. Laurier Inc. is a household products firm that is considering developing a new detergent. In raised as debt, failure to include interest expense as a cost in the cash flow statement
evaluating whether to go ahead with the new detergent project, which of the following items when determining the project’s cash flows will lead to an upward bias in the NPV.
should Laurier explicitly include in its cash flow analysis? (M) b. The preceding statement would be true if “upward” were replaced with “downward.”
a. The company will produce the detergent in a vacant facility that they renovated five years c. The existence of “externalities” reduces the NPV to a level below the value that would
ago at a cost of $700,000. exist in the absence of externalities.
b. The company will need to use some equipment that it could have leased to another d. If one of the assets that would be used by a potential project is already owned by the firm,
company. This equipment lease could have generated $200,000 per year in after-tax and if that asset could be leased to another firm if the project is not undertaken, then the
income. net rent that could be obtained should be charged as a cost to the project under
c. The new detergent is likely to significantly reduce the sales of the other detergent consideration.
products the company currently sells. e. The rent referred to in statement d is a sunk cost, and as such it should be ignored.
d. Statements b and c are correct. Brigham Brigham

CMA EXAMINATION QUESTIONS Page 9 of 107


MANAGEMENT ADVISORY SERVICES CAPITAL BUDGETING

B. bar graph E. payoff table


. Which of the following constitutes an example of a cost that is not incremental, and therefore,
21
C. nonnormal distribution Carter & Usry
not relevant in an accept/reject decision? (M)
a. A firm has a parcel of land that can be used for a new plant site, or alternatively, can be 28. The standard deviation of the expected net present value is determined by summing the
used to grow watermelons. discounted standard deviations for each period over the life of the project when the cash flows
b. A firm can produce a new cleaning product that will generate new sales, but some of the in each of the periods are:
new sales will be from customers who switch from another product the company currently A. independent D. negative
produces. B. positive E. perfectly correlated
c. A firm orders and receives a piece of new equipment that is shipped across the country C. mixed Carter & Usry
and requires $25,000 in installation and set-up costs.
d. Statements a, b, and c are examples of incremental cash flows, and therefore, relevant 12. Cinzano Inc. wants to use discounted cash flow techniques when analyzing its capital
cash flows. Brigham investment projects. The company is aware of the uncertainty involved in estimating future
cash flows. A simple method some companies employ to adjust for the uncertainty inherent in
. Which of the following is not considered a relevant concern in deter- mining incremental cash
22
their estimates is to:
flows for a new product? (M) A. ignore salvage values
a. The use of factory floor space that is currently unused but available for production of any B. average the expectations of several different managers
product. C. use accelerated depreciation
b. Revenues from the existing product that would be lost as a result of some customers D. adjust the minimum desired rate of return
switching to the new product. E. increase the estimates of the cash flows CMA adapted
c. Shipping and installation costs associated with preparing the machine to be used to
produce the new product. Tax Factor
d. The cost of a product analysis completed in the previous tax year and specific to the new In general
product. Brigham 58. Income taxes are levied on
a. net cash flow.
Uncertainty b. income as measured by accounting rules.
1. Which of the following best identifies the reason for using probabilities in capital budgeting c. net cash flow plus depreciation.
decisions? d. income as measured by tax rules. Barfield
A. uncertainty C. time value of money
B. cost of capital D. projects with unequal lives AICPA adapted 18. Which of the following statements is true?
a. All revenue is taxed.
*. Which of the following best identifies the reason for using probabilities in capital budgeting is b. All expenses are tax-deductible.
(E) c. Some revenues and expenses have no tax effects.
a. Different life of projects. c. Uncertainty. RPCPA 0577, 0588, 1093 d. Income taxes are based solely on revenues and expenses. L&H
b. Cost of capital. d. Time value of money.
*. In capital budgeting decisions, the following items are considered among others: (M)
26. In capital expenditure analysis, which of the following can be constructed to evaluate 1. Cash outflow for the investment.
alternative levels of investment? 2. Increase in working capital requirements.
A. normal distribution D. pie chart 3. Profit on sale of old asset
CMA EXAMINATION QUESTIONS Page 10 of 107
MANAGEMENT ADVISORY SERVICES CAPITAL BUDGETING

4. Loss on write-off of old asset. Depreciation Tax Shield


For which of the above items would taxes be relevant? (D) *. The accounting area in which the only objective of depreciation accounting relates to the effect
a. Items 1 and 3 only. c. All items. of depreciation charges upon tax payments is (E)
b. Items 3 and 4 only. d. Items 1, 3 and 4 only. RPCPA 0594 a. Income determination. c. Cost/volume/profit analysis.
b. Financial reporting. d. Capital budgeting. RPCPA 0587
6. The government could encourage increases in investment by
a. Increasing tax rates. 29. Which statement describes the relevance of depreciation in calculating cash flows?
b. Lengthening the MACRS period. a. Depreciation is relevant only when income taxes exist.
c. Letting a company expense fixed assets in the year acquired instead of through annual b. Depreciation is always relevant.
depreciation charges. c. Depreciation is never relevant.
d. Taking actions that would increase interest rates. L&H d. Depreciation is relevant only with discounted cash flow methods. L&H

61. Which of the following are tax deductible under U.S. tax law? (M) 56. Multiplying the depreciation deduction by the tax rate yields a measure of the depreciation tax
a. interest payments to bondholders c. common stock dividends (D)
b. preferred stock dividends d. all of the above Barfield a. shield. c. payable.
b. benefit. d. loss. Barfield
No Income Tax Situation
21. If there were no income taxes, 3. Depreciation is incorporated explicitly in the cash flow analysis of an investment proposal
a. Depreciation would be ignored in capital budgeting. because it:
b. The NPV method would not work. A. is a cost of operations that cannot be avoided
c. Income would be discounted instead of cash flow. B. results in an annual cash outflow
d. All potential investments would be desirable. L&H C. is a cash inflow
D. reduces the cash outlay for income taxes
35. If a company is NOT subject to income tax, which of the following is true of a proposed E. represents the initial cash outflow spread over the life of the investment CMA adapted
investment?
23
a. The project’s IRR equals the entity’s cost of capital. . A depreciation tax shield is
b. The project’s NPV is zero. a. An after-tax cash outflow.
c. Depreciation on assets required for the project is irrelevant to the evaluation. b. A reduction in income taxes.
d. The expected annual increase in future cash flows equals the investment required to c. The cash provided by recording depreciation.
undertake the project. L&H d. The expense caused by depreciation. CMA 1293 4-14, RPCPA 0596
24
53. The pre-tax and after-tax cash flows would be the same for all of the following items except (D) . The annual tax depreciation expense on an asset reduces income taxes by an amount equal
a. the liquidation of working capital at the end of a project's life. to
b. the initial (outlay) cost of an investment. a. The firm’s average tax rate times the depreciation amount.
c. the sale of an asset at its book value. b. One minus the firm’s average tax rate times the depreciation amount.
d. a cash payment for salaries and wages. Barfield c. The firm’s marginal tax rate times the depreciation amount.
d. One minus the firm’s marginal tax rate times the depreciation amount. CMA 1293 4-20

CMA EXAMINATION QUESTIONS Page 11 of 107


MANAGEMENT ADVISORY SERVICES CAPITAL BUDGETING

Accelerated Method vs. Straight-line Method competitors, Shanahan’s CFO estimates that Division A’s beta is 1.5, while Division B’s beta is
25
. The use of an accelerated method instead of the straight-line method of depreciation in 0.9. The risk-free rate is 5 percent and the market risk premium is 5 percent. The company is
computing the net present value of a project has the effect of 100 percent equity-financed. (WACC = ks, the cost of equity).
a. Raising the hurdle rate necessary to justify the project. Division B is considering the following projects given below. Each of the projects has the same
b. Lowering the net present value of the project. risk and all have the same risk as a “typical” Division B project.
c. Increasing the present value of the depreciation tax shield. Project Capital required IRR
d. Increasing the cash outflows at the initial point of the project. CMA 0695 4-3 1 $400 million 14.0%
2 300 million 10.7
Changes in Tax Rates 3 250 million 10.5
*. Your company is purchasing a transport equipment as part of its territorial expansion strategy. 4 320 million 10.0
The technical services department indicated that this equipment needs overhauling in year 4 5 230 million 9.0
or year 5 of its useful life. The overhauling cost will be expected during the year the The company is debating which cost of capital they should use to evaluate Division B’s
overhauling is done. The finance officer insists that the overhauling be done in year 4, not in projects. John Green argues that Shanahan should use the same cost of capital for each of its
year 5. The most likely reason is (M) RPCPA 0594 divisions, and believes it should base the cost of equity on Shanahan’s overall beta. Becky
a. There is lower tax rate in year 5. c. The time value of money is considered. White argues that the cost of capital should vary for each division, and that Division B’s beta
b. There is higher tax rate in year 5 d. Due statements A and C above. should be used to estimate the cost of equity for Division B’s projects.
If the company uses White’s approach, how much larger will the capital budget be than if it
Optimal Capital Budget uses Green’s approach? (E)
26
. An optimal capital budget is determined by the point where the marginal cost of capital is (D) a. Capital budget is $320 million larger using White’s approach.
A. Minimized. b. Capital budget is $550 million larger using White’s approach.
B. Equal to the average cost of capital. c. Capital budget is $870 million larger using White’s approach.
C. Equal to the rate of return on total assets. d. Capital budget is $1,200 million larger using White’s approach.
D. Equal to the marginal rate of return on investment. CIA 1187 IV-43 e. The capital budget is the same using the two approaches. Brigham
27
. A firm seeking to optimize its capital budget has calculated its marginal cost of capital and INVESTMENT OPTIONS
projected rates of return on several potential projects. The optimal capital budget is determined Abandonment option
by 29
. Which of the following statements best describes the likely impact that an abandonment option
A. Calculating the point at which marginal cost of capital meets the projected rate of return, will have on a project’s expected cash flow and risk? (E)
assuming that the most profitable projects are accepted first. a. No impact on expected cash flow, but risk will increase.
B. Calculating the point at which average marginal cost meets average projected rate of b. Expected cash flow increases and risk decreases.
return, assuming the largest projects are accepted first. c. Expected cash flow increases and risk increases.
C. Accepting all potential projects with projected rates of return exceeding the lowest d. Expected cash flow decreases and risk decreases.
marginal cost of capital. e. Expected cash flow decreases and risk increases. Brigham
D. Accepting all potential projects with projected rates of return lower than the highest
marginal cost of capital. CIA 1191 IV-57 Investment timing option
30
28
. Commodore Corporation is deciding whether it makes sense to invest in a project today, or to
. Shanahan Inc. has two divisions: Division A makes up 50 percent of the company, while postpone this decision for one year. Which of the following statements best describes the
Division B makes up the other 50 percent. Shanahan’s beta is 1.2. Looking at stand-alone issues that Commodore faces when considering this investment timing option? (E)
CMA EXAMINATION QUESTIONS Page 12 of 107
MANAGEMENT ADVISORY SERVICES CAPITAL BUDGETING

a. The investment timing option does not affect the expected cash flows and should a. Its estimated capital budget is too small because it fails to consider abandonment and
therefore have no impact on the project’s risk. growth options.
b. The more uncertainty about the project’s future cash flows the more likely it is that b. Its estimated capital budget is too large because it fails to consider abandonment and
Commodore will go ahead with the project today. growth options.
c. If the project has a positive expected NPV today, this means that its expected NPV will be c. Failing to consider abandonment options makes the optimal capital budget too large, but
even higher if it chooses to wait a year. failing to consider growth options makes the optimal capital budget too small, so it is
d. All of the above statements are correct. unclear what impact this policy has on the overall capital budget.
e. None of the above statements is correct. Brigham d. Failing to consider abandonment options makes the optimal capital budget too small, but
failing to consider growth options makes the optimal capital budget too large, so it is
Real options unclear what impact this policy has on the overall capital budget.
31
. Which of the following is an example of a flexibility option? (E) e. Neither abandonment nor growth options should have an effect on the company’s optimal
a. A company has the option to invest in a project today or to wait a year. capital budget. Brigham
b. A company has the option to back out of a project that turns out to be
unproductive. CAPITAL BUDGET EVALUATION METHODS
c. A company pays a higher cost today in order to be able to reconfigure the In general
project’s input or outputs at a later date. *. “Net present value” is an example of which concept? (E)
d. A company invests in a project today that may lead to enhanced technological a. Capital budgeting. c. Managerial control.
improvements that allow it to expand into different markets at a later date. b. Project feasibility. d. Management by exception. RPCPA 0580
e. All of the statements above are correct. Brigham
*. “Net present value” is an example of which concept? (E)
4. Which of the following are not real options? (M) a. Capital budgeting c. Management control
a. The option to expand production if the product is successful. b. Project feasibility d. Management by objectives RPCPA 0577
b. The option to buy additional shares of stock if the stock price goes up.
c. The option to expand into a new geographic region. *. All of the following are methods that aid management in analyzing the expected results of
d. The option to abandon a project. capital budgeting decisions, except (E) Horngren, RPCPA 1095, 1096
e. The option to switch sources of fuel used in an industrial furnace. Brigham a. Accrual accounting rate of return. c. Future value cash flow.
b. Payback period. d. Discounted cash flow rate of return.
5. Which of the following will not increase the value of a real option? (M)
a. An increase in the time remaining until the real option must be exercised. 1. The following measures are used by firms when making capital budgeting decisions except:
b. An increase in the volatility of the underlying source of risk. A. Payback period C. Net present value
c. An increase in the risk-free rate. B. Internal rate of return D. P/E ratio B&M
d. An increase in the cost of exercising the real option.
e. Statements b and d. Brigham Use of Net Income
*. A number of techniques are commonly used in the analysis of capital budgeting decisions.
Abandonment and growth options Each method involves the measurement of cash flows, except the (E)
32
. Clueless Corporation never considers abandonment options or growth options when a. Internal rate of return. c. Average rate of return method.
estimating its optimal capital budget. What impact does this policy have on the company’s b. Payback period method. d. Net present value. RPCPA 1097
optimal capital budget? (M)
CMA EXAMINATION QUESTIONS Page 13 of 107
MANAGEMENT ADVISORY SERVICES CAPITAL BUDGETING

20. The technique that does NOT use cash flows is a. NPV. c. Book rate of return.
a. Payback. c. IRR. b. Payback. d. All of the above. L&H
b. NPV. d. Book rate of return. L&H
8. The method of project selection that considers the time value of money in a capital budgeting
9. Which of the following capital budgeting methods does NOT consider the time value of decision computes the:
money? A. accounting rate of return on average investment
a. IRR. c. Time-adjusted rate of return. B. internal rate of return
b. Book rate of return. d. NPV. L&H C. payback period
D. return on investment
33
. Which one of the following capital investment evaluation methods does not take the time value E. accounting rate of return on initial investment AICPA adapted
of money into consideration?
a. Net present value. c. Internal rate of return. 52. Which of the following methods consider the time value of money?
b. Discounted payback. d. Accounting rate of return. CMA 0696 4-26 a. payback and accounting rate of return
b. payback and internal rate of return
Use of Cash Flows c. internal rate of return and accounting rate of return
In general d. internal rate of return and net present value H&M
*. In capital budgeting, these techniques are applied: payback (PB) method, net present value
(NPV) method and time-adjusted rate of return (TARR) method. PB method has this in *. Which of the following capital investment rating procedures recognize(s) the time value of
common with NPV and TARR methods. (M) money? (E)
a. Use of cash flows. RPCPA 0590 a. b. c. d.
b. Consideration of the time value of money. Profitability index Yes No Yes No
c. Use of discounting. Discounted rate of return Yes Yes No No
d. Use of accrual method of accounting. RPCPA 1094
1. Which of the following groups of capital budgeting techniques uses the time value of money?
57. All of the following are major categories of cash flows in capital investment decisions EXCEPT (E)
(E) a. Book rate of return, payback, and profitability index.
a. the initial investment in machines and working capital. b. IRR, payback, and NPV.
b. recurring operating cash flows. c. IRR, NPV, and profitability index.
c. the initial working capital investment d. IRR, book rate of return, and profitability index. L&H
d. depreciation expense reported on the income statement. Horngren
30. Which of the following combinations of capital budgeting techniques includes only discounted
Without Time Value of Money cash flow techniques?
1. Which of the following capital budgeting techniques ignores the time value of money? a. Book rate of return, payback, and profitability index.
a. payback period c. internal rate of return b. NPV, IRR and profitability index.
b. net present value d. profitability index Barfield c. IRR, payback, and NPV.
d. Profitability index, NPV, and payback. L&H
With Time Value of Method
28. Which of the following is a discounted cash flow method? Use of Time Value of Money vs. No Time Value of Money
CMA EXAMINATION QUESTIONS Page 14 of 107
MANAGEMENT ADVISORY SERVICES CAPITAL BUDGETING

27. In contrast to the payback period and book rate of return methods, the NPV and IRR methods A. Is synonymous with the internal rate of return.
a. Consider the time value of money. c. Use after-tax cash flows. B. Focuses on income as opposed to cash flows.
b. Ignore depreciation. d. All of the above. L&H C. Is inconsistent with the divisional performance measure known as return on investment.
D. Recognizes the time value of money. CMA 0691 4-18
18. The net present value and internal rate of return methods of capital budgeting are superior to
the payback method in that they: (M) 11. Which of the following methods uses income instead of cash flows?
a. are easier to implement. a. payback c. internal rate of return
b. consider the time value of money. b. accounting rate of return d. net present value H&M
c. require less input.
37
d. reflect the effects of depreciation and income taxes. AICPA adapted . The capital budgeting technique known as accounting rate of return uses
a. b. c. d.
ACCOUNTING RATE OF RETURN Revenue over life of project No No Yes Yes
Definition Depreciation expense Yes No No Yes
*. A capital budgeting method that provides a rough approximation of an investment’s profitability
as measured with net income from the income statement is known as: (E)
a. Average rate of return method. c. Payback period. RPCPA 1097
b. Net present value method. d. Internal rate of return method.
34
. The technique that measures the estimated performance of a capital investment by dividing
the project's annual after-tax net income by the average investment cost is called the (E)
A. Bail-out payback method. C. Profitability index method. CMA 0692 4-21
B. Internal rate of return method. D. Accounting rate of return method.
35
. The technique that measures the estimated performance of a capital investment by dividing
the project's annual after-tax net income by the average investment cost is called the
A. Average rate of return method. C. Capital asset pricing model. CMA 1290 4-
15
B. Internal rate of return method. D. Accounting rate of return method.

31. The capital budgeting method that divides a project's annual incremental net income by the
initial investment is the: (M)
a. internal rate of return method.
b. the simple ( or accounting) rate of return method.
c. the payback method.
d. the net present value method. CMA adapted

Characteristics
36
. The accounting rate of return
CMA EXAMINATION QUESTIONS Page 15 of 107
MANAGEMENT ADVISORY SERVICES CAPITAL BUDGETING

99. The capital budgeting technique known as accounting rate of return uses (D) 65. For capital budgeting decisions, the use of the accrual accounting rate of return for evaluating
Barfield a. b. c. d. performance is often a stumbling block to the implementation of the (E)
Salvage value No No Yes Yes a. net cash flow.
Time value of money No Yes Yes No b. most effective goal-congruence choice.
c. discounted cash flow method for capital budgeting.
Formula d. most effective tax strategy. Horngren
38
. The method that divides a project’s annual after-tax net income by the average investment
cost to measure the estimated performance of a capital investment is the 19. The disadvantages of the book rate of return method is/are:
a. Internal rate of return method. c. Payback method. CMA 1294 4-24 A. It uses net income instead of cash flows
b. Accounting rate of return method. d. Net present value (NPV) method. B. The pattern of income has no impact on the book rate of return
C. There is no clear cut decision rule
100.In computing the accounting rate of return, the ______ level of investment should be used as D. All of the above B&M
the denominator.
a. Average c. Residual PAYBACK PERIOD
b. Initial d. Cumulative Barfield Definition
10. The payback period is the
16. The accounting rate of return on original investment is calculated as a. length of time over which the investment will provide cash inflows.
a. original investment/net income c. net income/original investment b. length of time over which the initial investment is recovered.
b. net income/debt d. assets/debt H&M c. shortest length of time over which an investment may be depreciated.
d. shortest length of time over which the net present value will be positive. Barfield
64. The approach to capital budgeting which divides an accounting measure of income by an
39
accounting measure of investment is (E) . The length of time required to recover the initial cash outlay of a capital project is determined
a. net present value. c. payback method. Horngren by using the CMA 1294 4-20
b. internal rate of return. d. accrual accounting rate of return. a. Discounted cash flow method. c. Weighted net present value method.
b. Payback method. d. Net present value method.
18. The book rate of return on a project is calculated as:
A. Book Cash Flow/book assets C. Book assets/book income *. An investment rating approach which measures the length of time required to recover the initial
B. Book income/book assets D. None of the above B&M outlay for a particular investment proposal is the (E)
a. Accounting rate of return c. Net present value
Limitation b. Payback period d. Present value index RPCPA 0590
*. The following statements refer to the accounting rate of return (ARR)
40
1. The ARR is based on the accrual basis, not cash basis. . The technique that measures the number of years required for the after-tax cash flows to
2. The ARR does not consider the time value of money. recover the initial investment in a project is called the
3. The profitability of the project is considered. A. Net present value method. C. Profitability index method. CMA 1290 4-17
From the above statements, which are considered limitations of the ARR concept? (M) B. Payback method. D. Accounting rate of return method.
a. Statements 2 and 3 only. c. All the 3 statements.
b. Statements 3 and 1 only. d. Statements 1 and 2 only. RPCPA 1094

CMA EXAMINATION QUESTIONS Page 16 of 107


MANAGEMENT ADVISORY SERVICES CAPITAL BUDGETING

*. The method that measures how quickly investment pesos may be recovered is the (E) 8. The payback period rule:
a. Payback method c. Simple rate of return method A. Varies the cut-off point with the interest rate
b. Time adjusted rate of return d. Least squares method RPCPA 1074, 10/77 B. Determines a cut-off point so that all projects accepted by the NPV rule will be accepted
by the payback period rule.
*. The payback method measures (E) C. Requires an arbitrary choice of a cut-off point
a. Profitability of an investment c. Time to recover investment RPCPA 1093 D. Both A and C B&M
b. Economic life of the investment d. Cash flow from an investment
2. Which of the following capital budgeting techniques may potentially ignore part of a project's
27. The payback method measures: (E) relevant cash flows? (M)
a. how quickly investment dollars may be recovered. a. net present value c. payback period
b. the cash flow from an investment. b. internal rate of return d. profitability index Barfield
c. the economic life of an investment.
41
d. the profitability of an investment. CMA adapted . A characteristic of the payback method (before taxes) is that it (E)
a. Incorporates the time value of money.
61. The method that measures the time it will take to recoup, in the form of future cash inflows, the b. Neglects total project profitability.
total dollars invested in a project is called (E) c. Uses accrual accounting inflows in the numerator of the calculation. CMA 0694 4-17
a. the accrued accounting rate-of-return method. d. Uses the estimated expected life of the asset in the denominator of the calculation.
b. payback method.
c. internal rate-of-return method. 4. Which of the following capital budgeting techniques does not routinely rely on the assumption
d. the book-value method. Horngren that all cash flows occur at the end of the period?
a. internal rate of return c. profitability index
63. The payback method of capital budgeting approach to the investment decision highlights (E) b. net present value d. payback period Barfield
a. cash flow over the life of the investment.
b. the liquidity of the investment. 7. The payback method assumes that all cash inflows are reinvested to yield a return equal to
c. the tax savings of the depreciation amounts. a. the discount rate. c. the internal rate of return.
d. having as lengthy payback time as possible. Horngren b. the hurdle rate. d. zero. Barfield

Characteristics 39. Assuming that a project has already been evaluated using the following techniques, the
19. The technique most concerned with liquidity is (M) evaluation under which technique is least likely to be affected by an increase in the estimated
a. Payback. c. IRR. residual value of the project?
b. NPV. d. Book rate of return. L&H a. Payback period. c. NPV.
b. IRR. d. PI. L&H
2. The payback criterion for capital investment decisions
a. Is conceptually superior to the IRR criterion. 30. The evaluation of an investment having uneven cash flows using the payback method: (M)
b. Takes into consideration the time value of money. a. cannot be done.
c. Gives priority to rapid recovery of cash. b. can be done only by matching cash inflows and investment outflows on a year-by-year
d. Emphasizes the most profitable projects. L&H basis.
CMA EXAMINATION QUESTIONS Page 17 of 107
MANAGEMENT ADVISORY SERVICES CAPITAL BUDGETING

c. will product essentially the same results as those obtained through the use of discounted a. Does not consider the time value of money.
cash flow techniques. b. Consider cash flows after the payback has been reached.
d. requires the use of a sophisticated calculator or computer software. G & N 9e c. Uses discounted cash flow techniques. CMA 1295 4-1
d. Generally leads to the same decision as other methods for long-term projects.
10. Which of the following investment rules may not use all possible cash flows in its calculations?
A. Payback period. C. IRR 34. Which of the following methods FAILS to distinguish between return of investment and return
B. NPV D. All of the above B&M on investment.
a. NPV. c. Payback.
Advantage b. IRR. d. Book rate of return. L&H
*. An advantage of the payback method is (E)
a. Not based on cash flow data. c. Precise in estimate of profits. 15. Which of the following is NOT a defect of the payback method?
b. Insensitive of the life of the project. d. Easy to apply. RPCPA 1093 a. It ignores cash flow because it uses net income.
b. It ignores profitability.
*. An advantage of using the payback method of evaluating capital budgeting alternatives is that c. It ignores the present values of cash flows.
payback is d. It ignores the pattern of cash flows beyond the payback period. L&H
a. Insensitive to the life of the project considered.
b. Precise estimate of profitability. 14. Deficiencies associated with using the payback method to evaluate investment alternatives
c. Based on cash flow data. include all of the following, except that:
d. Easy to apply. RPCPA 0577, 5/80, 5/98 A. the present value of cash inflows is ignored
B. inflows of different time periods are treated equally
12. The advantage of the payback rule is: C. it may be used to select those investments yielding a quick return of cash
A. Adjustment for uncertainty of early cash flows D. cash flows after the payback period are ignored CIA adapted
B. It is simple to use
C. Does not discount cash flows 1. A major disadvantage of the payback period method is that it (E)
D. Both A and C B&M a. Is useless as a risk indicator.
b. Ignores cash flows beyond the payback period.
Disadvantage c. Does not directly account for the time value of money.
*. This technique is criticised because it fails to consider investment profitability (E) d. Statements b and c are correct. Brigham
a. Time adjusted ROI c. Average return on investment
b. Payback method d. Present value method RPCPA 1093 15. The following are disadvantages of using the payback rule except:
A. The payback rule ignores all cash flow after the cutoff date
11. Which of the following capital budgeting techniques has been criticized because it fails to B. The payback rule does not use the time value of money
consider investment profitability? (E) C. The payback period is easy to calculate and use
a. payback method c. net present value method D. The payback rule does not have the value additive property B&M
b. accounting rate of return d. internal rate of return Barfield
Formula
42
. Which one of the following statements about the payback method of investment analysis is . The cash payback formula is:
correct? The payback method a. Cost of Capital Investment / Net Income.
CMA EXAMINATION QUESTIONS Page 18 of 107
MANAGEMENT ADVISORY SERVICES CAPITAL BUDGETING

b. Average Investment / Net Annual Cash Inflow. TVM relevant irrelevant Relevant irrelevant
c. Cost of Capital Investment / Net Annual Cash Inflow.
d. Average Investment / Net Income. RPCPA 1001 44
. The capital budgeting technique known as payback period uses
a. b. c. d.
5. Assume that a project consists of an initial cash outlay of $100,000 followed by equal annual Depreciation expense Yes Yes No No
cash inflows of $40,000 for 4 years. In the formula X = $100,000/$40,000, X represents the (E) Time value of money Yes No No Yes
a. payback period for the project. c. internal rate of return for the project.
b. profitability index of the project. d. project's discount rate. Barfield BAILOUT PAYBACK
Definition
Decision Criteria *. The bailout payback period is (E)
9. The payback period rule accepts all projects for which the payback period is: a. The payback period used by firms with government insured loans.
A. Greater than the cut-off value C. Is positive b. The length of time for payback using cash flows plus the salvage value to recover the
B. Less than the cut-off value D. An integer B&M original investment
c. (a) and (b)
Payback Reciprocal d. None of the above. RPCPA 1090
43
. The payback reciprocal can be used to approximate a project’s
a. Profitability index 45
. The bailout payback method (E)
b. Net present value. A. Is used by firms with federally insured loans.
c. Accounting rate of return if the cash flow pattern is relatively stable. B. Calculates the payback period using the sum of the net cash flows and the salvage value.
d. Internal rate of return if the cash flow pattern is relatively stable. CMA 0693 4-27 C. Calculates the payback period using the difference between net cash inflow and the
salvage value.
Relevant Items D. Estimates short-term profitability. Gleim
1. In order to calculate the payback period for a project, it is necessary to know the:
A. salvage value D. net present value Use
B. useful life E. annual cash flow 46
. The bailout payback method
C. minimum desired rate of return Carter & Usry a. Incorporates the time value of money.
b. Equals the recovery period from normal operations.
1. Calculating the payback period for a capital project requires knowing which of the following? c. Eliminates the disposal value from the payback calculation.
a. Useful life of the project. d. Measures the risk if a project is terminated. CMA 1292 4-11
b. The company’s minimum required rate of return.
c. The project’s NPV.
d. The project’s annual cash flow. L&H

Irrelevant Items
*. As a capital budgeting technique, the payback period considers depreciation expenses (DE)
and time value of money (TVM) as follows: (M)
RPCPA 1095 a. b. c. d.
DE relevant irrelevant Irrelevant relevant
CMA EXAMINATION QUESTIONS Page 19 of 107
MANAGEMENT ADVISORY SERVICES CAPITAL BUDGETING

*. The method of project selection which considers the time value of money in a capital
DISCOUNTED CASH FLOW ANALYSIS budgeting decision is accomplished by computing the (E)
37. The consumption opportunity increases when an investment with positive NPV is available a. Accounting rate of return on initial investment
because: b. Payback period.
A. The investment is better than what is available in the market c. Accounting rate of return on average investment.
B. The project rate of return is less than market rate d. Discounted cash flow. RPCPA 0598
C. The market is irrelevant to the investment criteria
D. All of the above are reasons for increase in consumption B&M 2. The component of the capital investment decision that would most likely concern an
accountant is the:
Factors A. social responsibility factors D. imponderables
14. When using one of the discounted cash flow methods to evaluate the desirability of a capital B. competition E. legal restrictions
budgeting project, which of the following factors is generally not important? (E) C. time value of money Carter & Usry
a. method of financing the project under consideration
b. timing of cash flows relating to the project *. The fact that an amount of money that is to be received in the future is not equivalent to the
c. impact of the project on income taxes to be paid same amount of money to be received now is referred to as: (E)
d. amounts of cash flows relating to the project Barfield a. Present value of money. c. Future value of money.
b. Time value of money. d. Discounted value of money. RPCPA 0587
Assumptions
47
52. In a typical (conservative assumptions) after-tax discounted cash flow analysis, depreciation . What is the time value of money?
expense is assumed to accrue at A. Interest. C. Future value.
a. the beginning of the period. B. Present value. D. Annuity. Gleim
b. the middle of the period.
c. the end of the period. 12. The time value of money is explicitly recognized through the process of
d. irregular intervals over the life of the investment. Barfield a. interpolating. c. annuitizing.
b. discounting. d. budgeting. Barfield
Time Value Of Money
53. There are two reasons for discounting future cash flow. They are: 13. The time value of money is considered in long-range investment decisions by (M)
A. A dollar today is worth more than a dollar tomorrow a. assuming equal annual cash flow patterns.
B. A safe dollar is worth more than a risky one b. investing only in short-term projects.
C. The value of a dollar is changing all the time c. assigning greater value to more immediate cash flows.
D. A and B above B&M d. ignoring depreciation and tax implications of the investment. Barfield
48
29. The line that connects the maximum that one can consume this year (now) and the maximum . Basic time value of money concepts concern
one can consume next year: Gleim A. B. C. D.
A. Has a slope of (1+r) C. Has a slope of r Interest Factors Yes Yes No No
B. Has a slope of -(1+r) D. Has a slope of 1/r B&M Risk Yes No Yes No
Cost of Capital No Yes No Yes

CMA EXAMINATION QUESTIONS Page 20 of 107


MANAGEMENT ADVISORY SERVICES CAPITAL BUDGETING
49
. The present value may be calculated for discounted cash a. Cash flow based payback period. c. Payback method.
Gleim A. B. C. D. b. Capital budgeting. d. Discounted cash flow. RPCPA 0595
Inflows Yes Yes No No
Outflows Yes No Yes No *. The method of project selection which considers the value of money in a capital budgeting
Annuities Yes Yes No Yes decision is accomplished by computing the
a. Payback period.
9. The net present value and the internal rate of return methods of decision making in capital b. Accounting rate of return on initial investment.
budgeting are superior to the payback method in that they: c. Accounting rate of return on average investment.
A. consider the time value of money d. Discounted cash flow. RPCPA 0577, 10/79, 10/93
B. are easier to implement
C. consider accrual-based accounting income *. There are several evaluation techniques for determining the acceptability of an investment.
D. require less input The method that considers the time value of money in an investment budgeting decision is
E. reflect the effects of depreciation and income taxes AICPA adapted accomplished by determining the (E)
a. Cash-flow payback method. c. Average rate of return.
Application b. Accounting rate of return. d. Discounted cash flow. RPCPA 1081
11. A company is considering the purchase of a new conveyor belt system for carrying parts and
subassemblies from building to building within its plant complex. It is expected that the system *. The method of project selection which considers the time value of money in a capital
will have a useful life of at least ten years and that it will substantially reduce labor and waiting- budgeting decision is accomplished by computing the (E)
time costs. If the company's average cost of capital is about 15% and if some evaluation must a. Accounting rate of return on initial investment.
be made of cost/benefit relationships (including the effects of interest) to determine the b. Accounting rate of return on average investment.
desirability of the purchase, the most relevant quantitative technique for evaluating the c. Discounted cash flow.
investment is: d. Payback period. RPCPA 0580
A. present value (or internal rate of return) analysis
B. Program Evaluation and Review Technique (PERT) *. The modern approach used by decision-makers in evaluating investment opportunities by
C. accounting rate of return analysis comparing the original investment with the cash generation for the entire life of the project to
D. cost-volume-profit analysis come up with the rate of return on investment is (E)
E. payback analysis AICPA adapted a. Discounted cash flow method d. Profitability index
b. Accounting rate of return approach e. Cash payoff method
2. Discounted cash flow techniques for analyzing capital budgeting decisions are NOT normally c. Cash flow projections RPCPA 1077
applied to projects 50
a. Requiring no investment after the first year of life. . All of the following items are included in discounted cash flow analysis except
b. Having useful lives shorter than one year. a. Future operating cash savings.
c. That are essential to the business. b. The disposal prices of the current assets
d. Involving replacement of existing assets. L&H c. The future asset depreciation expense.
d. The tax effects of future asset depreciation. CMA 0694 4-18
Definition of Discounted Cash Flow
*. Which of the following methods measures the cash flows and outflows of a project as if they
occurred at a single point in time? (M)
CMA EXAMINATION QUESTIONS Page 21 of 107
MANAGEMENT ADVISORY SERVICES CAPITAL BUDGETING
51
. The capital budgeting model that is ordinarily considered the best model for long-range Cost of Capital
decision making is the CMA 1294 4-25 27. Hurdle rate for capital budgeting decisions is:
a. Payback model. c. Unadjusted rate of return model. A. The cost of capital C. The cost of equity
b. Accounting rate of return model. d. Discounted cash flow model. B. The cost of debt D. All of the above B&M

*. When using one of the discounted-cash-flow methods to evaluate the feasibility of a capital *. In an investment in plant the return that should keep the market price of the firm stock
budgeting project, which of the following factors generally is not important? (M) unchanged is (M) RPCPA 0577, 1077, 0588, 1093
a. The method of financing the project under consideration. a. Payback c. Net present value
b. The impact of the project on income taxes to be paid. b. Discounted rate of return d. Cost of capital
c. The timing of cash flows relating to the project.
d. The amount of cash flows relating to the project. RPCPA 0598 25. For a project such as plant investment, the return that should leave the market price of the
firm's stock unchanged is known as the
52
. Discounted cash flow concepts concern a. cost of capital. c. payback rate.
Gleim a. b. c. d. b. net present value. d. internal rate of return. Barfield
Interest Factors Yes Yes No No
Risk Yes No Yes No 16. A company with cost of capital of 15% plans to finance an investment with debt that bears 10%
interest. The rate it should use to discount the cash flows is
53
. Depreciation is incorporated explicitly in the discounted cash flow analysis of an investment a. 10%. c. 25%.
proposal because it b. 15%. d. Some other rate. L&H
A. Is a cost of operations that cannot be avoided.
B. Is a cash inflow. 1. The term cost of capital for a project depends on:
C. Reduces the cash outlay for income taxes. A. The use to which the capital is put, i.e. the project
D. Represents the initial cash outflow spread over the life of the investment. CMA 1277 5-14 B. The company's cost of capital
C. The industry cost of capital
31. Discounted cash flow methods for capital budgeting focus on (E) D. All of the above B&M
a. cash inflows. c. cash outflows.
b. operating income. d. both (a) and (c). Horngren Weighted-Average Cost of Capital
54
. A company had made the decision to finance next year’s capital projects through debt rather
Rate of Return than additional equity. The benchmark cost of capital for these projects should be (M)
Discount Rate a. The before-tax cost of new-debt financing.
18. The discount rate for safe projects is the: b. The after-tax cost of new-debt financing.
A. Market rate of return C. Market risk premium c. The cost of equity financing.
B. Risk-free rate D. None of the above B&M d. The weighted-average cost of capital. CIA 0597 IV-42

19. The discount rate for a project with a risk the same as the market risk is the: Opportunity Cost of Capital
A. Market rate of return C. Market risk premium 32. The ___________________ is the highest rate of return that can be earned from the most
B. Risk-free rate D. None of the above B&M attractive, alternative capital project available to the firm. (D)
a. accounting rate of return c. hurdle rate
CMA EXAMINATION QUESTIONS Page 22 of 107
MANAGEMENT ADVISORY SERVICES CAPITAL BUDGETING

b. internal rate of return d. opportunity cost of capital Barfield c. present value of X.


d. present value interest factor associated with r. Barfield
21. The opportunity cost of capital for a risky project is
A. The expected rate of return on a government security having the same maturity as the Discounted Cash Flow Methods
project *. A mechanized system of handling parts from one assembly line to another is being
B. The expected rate of return on a well diversified portfolio of common stocks contemplated by the Moonbeam Co. The technical evaluation indicated that the system will
C. The expected rate of return on a portfolio of securities of similar risks as the project reduce labor and waiting time costs substantially. An assessment has to be made of
D. None of the above B&M cost/benefit relationship including the effects of interest. The most relevant quantitative
technique to evaluate the project is (M)
Comprehensive a. Regression analysis. c. Time adjusted rate of return analysis.
. All of the following refer to the discount rate used by a firm in capital budgeting except (M) b. PERT-CPM. d. Payback period analysis. RPCPA 0594
a. Hurdle rate. c. Opportunity cost.
b. Required rate of return. d. Opportunity cost of capital. RPCPA 1096

Present Value Formula


94. If r is the discount rate, the formula [1/(1 + r)] refers to the
a. future value interest factor associated with r for one period.
b. present value of some future cash flow.
c. present value interest factor associated with r for one period.
d. future value interest factor for an annuity with a duration of r periods. Barfield

97. Which of the following indicates that the first cash flow is at the end of a period? (M)
Barfield a. b. c. d.
Ordinary annuity Yes Yes No No
Annuity due No Yes Yes No

Future Value Formula


95. Future value is the
a. sum of dollars-in discounted to time zero.
b. sum of dollars-out discounted to time zero.
c. difference of dollars-in and dollars-out.
d. value of dollars-in minus dollars-out for future periods adjusted for any interest-
compounding factor. Barfield

98. Assume that X represents a sum of money that Bill has available to invest in a project that will
yield a return of r. In the formula Y = X(1 + r), Y represents the
a. future value of X in one period.
b. future value interest factor associated with r.
CMA EXAMINATION QUESTIONS Page 23 of 107
MANAGEMENT ADVISORY SERVICES CAPITAL BUDGETING
55
. The discount rate (hurdle rate of return) must be determined in advance for the (E) PROFITABILITY INDEX
a. Payback period method. c. Net present value method. Definition
b. Time adjusted rate of return method. d. Internal rate of return method. 3. The profitability index
a. Does not use present values of cash flows.
56
. Amster Corporation has not yet decided on its hurdle rate for use in the evaluation of capital b. Is generally preferable to any other approach for evaluating mutually exclusive investment
budgeting projects. This lack of information will prohibit Amster from calculating a project’s alternatives.
(M) c. Produces the same ranking of investment alternatives as does the IRR criterion.
CMA 0693 4-20 a. b. c. d. d. Is a discounted cash flow method. L&H
Accounting Rate of Return No Yes No No
Net Present Value No Yes Yes Yes *. Which of the following capital budgeting techniques is computed by dividing present value of
Internal Rate of Return No Yes Yes No future inflows by the initial investment? (M)
a. Accounting rate of return c. Payback reciprocal
30. Which capital budgeting technique(s) measure all expected future cash inflows and outflows b. Time-adjusted rate of return d. Profitability index RPCPA 0589
as if they occurred at a single point in time? (E)
a. Net present value c. Payback 41. The profitability index is (E)
b. Internal rate of return d. Both (a) and (b). Horngren a. the ratio of net cash flows to the original investment.
b. the ratio of the present value of cash flows to the original investment.
2. Which of the following investment rules does not use the time value of the money concept? c. a capital budgeting evaluation technique that doesn't use discounted values.
A. The payback period d. a mandatory technique when capital rationing is used. Barfield
B. Internal rate of return
58
C. Net present value . The technique that reflects the time value of money and is calculated by dividing the present
D. All of the above use the time value concept B&M value of the future net after-tax cash inflows that have been discounted at the desired cost of
capital by the initial cash outlay for the investment is the
BREAKEVEN TIME a. Capital rationing method. c. Profitability index method.
14. Which of the following statements regarding the discounted payback period rule is true? b. Average rate of return method. d. Accounting rate of return. CMA 1290 4-14
A. The discounted payback rule uses the time value of money concept.
B. The discounted payback rule is better than the NPV rule
C. The discounted payback rule considers all cash flows
D. The discounted payback rule exhibits the value additive property B&M
57
. When evaluating projects, breakeven time is best described as (M)
a. Annual fixed costs  monthly contribution margin.
b. Project investment  annual net cash inflows.
c. The point at which cumulative cash inflows on a project equal total cash outflows.
d. The point at which discounted cumulative cash inflows on a project equal discounted total
cash outflows. CMA 0693 4-28

CMA EXAMINATION QUESTIONS Page 24 of 107


MANAGEMENT ADVISORY SERVICES CAPITAL BUDGETING
59
. The profitability index (present value index) B. Net present value method. D. Payback method. CMA 1294 4-23
a. Represents the ratio of the discounted net cash outflows to cash inflows.
61
b. Is the relationship between the net discounted cash inflows less the discounted cash . The technique that recognizes the time value of money by discounting the after-tax cash flows
outflows. for a project over its life to time period zero using the company’s minimum desired rate of
c. Is calculated by dividing the discounted profits by the cash outflows. return is the CMA 1290 4-13
d. Is the ratio of the discounted net cash inflows to discounted cash outflows. CMA 0695 4-4 a. Net present value method. c. Average rate of return method.
b. Payback method. d. Accounting rate of return method.
19. The profitability index is the ratio of
a. Total cash inflows to the cost of the investment. 22. Probabilistic estimates are most frequently used with which of the following methods of capital
b. The present value of cash inflows to the cost of the investment. expenditure evaluation?
c. The NPV of the investment to the cost of the investment. A. payback C. internal rate of return
d. The IRR to the company’s cost of capital. L&H B. present value D. accounting rate of return Carter & Usry

Assumption 3. The net present value of a proposed project represents the:


42. Which method of evaluating capital projects assumes that cash inflows can be reinvested at A. cash flows less the original investment
the discount rate? B. present value of the cash flows plus the present value of the original investment less the
a. internal rate of return c. profitability index original investment
b. payback period d. accounting rate of return Barfield C. present value of the cash flows less the original investment
D. present value of the cash flows less the cost of the old machine being replaced
Formula E. cash flows less the present value of the cash flows Carter & Usry
33. Profitability index is the ratio of:
A. Present value of cash flow to initial investment 34. The capital budgeting method which calculates the expected monetary gain or loss from a
B. Net present value cash flow to initial investment project by discounting all expected future cash inflows and outflows to the present point in time
C. Net present value of cash flow to IRR using the required rate of return is the (E)
D. Present value of cash flow to IRR B&M a. payback method.
b. accrual accounting rate-of-return method.
Benefit Cost Ratio c. sensitivity method.
34. Benefit cost ratio is defined as: d. net present value method. Horngren
A. Present value of cash flow to initial investment
B. Net present value cash flow to initial investment 70. A capital budgeting tool management can use to summarize the difference in the future net
C. Net present value of cash flow to IRR cash inflows from an intangible asset at two different points in time is referred to as (E)
D. Present value of cash flow to IRR B&M a. the accrual accounting rate-of-return method.
b. the net present value method.
NET PRESENT VALUE METHOD c. sensitivity analysis
Definition d. the payback method. Horngren
60
. The method that recognizes the time value of money by discounting the after-tax cash flows
over the life of a project, using the company's minimum desired rate of return is the Underlying Theory
A. Accounting rate of return method. C. Internal rate of return method.
CMA EXAMINATION QUESTIONS Page 25 of 107
MANAGEMENT ADVISORY SERVICES CAPITAL BUDGETING

*. The excess present value method is anchored on the theory that the future returns, expressed C. Uncertain cash flows that extend far into the future
in terms of present value, must at least be (E) D. All of the above B&M
a. Equal to the amount of investment c. More than the amount of investment
b. Less than the amount of investment RPCPA 1074 16. Which of the following capital budgeting methods has the value additive property?
A. NPV C. Payback period
62
. If a firm identifies (or creates) an investment opportunity with a present value <List A> its cost, B. IRR D. Discounted payback period B&M
the value of the firm and the price of its common stock will <List B> (M)
CIA 1195 IV-44 a. b. c. d. Assumption
List A Greater than Greater than Equal to Equal to 33. If an analyst desires a conservative net present value estimate, he/she will assume that all
List B Increase Decrease Increase Decrease cash inflows occur at (M)
a. mid year. c. year end.
Characteristics b. the beginning of the year. d. irregular intervals. Barfield
35. The net present value method of evaluating proposed investments
a. measures a project's internal rate of return. *. The common assumption in capital budgeting analysis is that cash inflows occur in lump sums
b. ignores cash flows beyond the payback period. at the end of individual years during the life of an investment project when in fact they flow
c. applies only to mutually exclusive investment proposals. more or less continuously during those years (M)
d. discounts cash flows at a minimum desired rate of return. Barfield a. Results in understated estimates of NPV.
b. Is done because present value tables for continuous flows cannot be constructed.
63
. In evaluating a capital budget project, the use of the net present value (NPV) model is c. Will result in inconsistent errors being made on estimating NPVs such that project cannot
ordinarily not affected by the (E) be evaluated reliably.
a. Method of funding the project. d. Results in higher estimate for the IRR on the investment. RPCPA 1095
b. Initial cost of the project. 65
c. Amount of added working capital needed for operations during the term of the project. . The accountant of Ronier, Inc. has prepared an analysis of a proposed capital project using
d. Project’s salvage value. CMA 1294 4-21 discounted cash flow techniques. One manager has questioned the accuracy of the results
e. Amount of the project’s associated depreciation tax allowance. because the discount factors employed in the analysis have assumed the cash flows occurred
at the end of the year when the cash flows actually occurred uniformly throughout each year.
64
. The capital budgeting technique known as net present value uses The net present value calculated by the accountant will
AICPA 1180 T-48 a. b. c. d. A. Not be in error.
B. Be slightly overstated.
Cash flow over life of project No No Yes Yes
C. Be unusable for actual decision making.
Time value of money Yes No No Yes
D. Be slightly understated but usable. CMA 1278 5-10
42. The net present value method focuses on (E) 21. The net present value method assumes that all cash inflows can be immediately reinvested at
a. cash inflows. c. cash outflows. the
b. accrual-accounting net income. d. both (a) and (c). Horngren a. cost of capital. c. internal rate of return. Barfield
b. discount rate. d. rate on the corporation's short-term debt.
41. The net present value rule is valid for:
A. Two period certain cash flows
B. Two period uncertain cash flows
CMA EXAMINATION QUESTIONS Page 26 of 107
MANAGEMENT ADVISORY SERVICES CAPITAL BUDGETING

21. The net present value method of capital budgeting assumes that cash flows are reinvested at: A. accounting rate of return D. return on investment
(E) B. net present value E. payback
a. the internal rate of return on the project. C. internal rate of return AICPA adapted
b. the rate of return on the company's debt.
c. the discount rate used in the analysis. 36. The net present value rule assumes that:
d. a zero rate of return. CMA adapted A. Borrowing and lending rate are equal C. Both A and B are true
B. Financial markets are well functioning D. None of the above are true B&M
66
. The net present value (NPV) method of investment project analysis assumes that the project’s
cash flows are reinvested at the CMA 0692 4-16, RPCPA 0596 Advantage
68
a. Computed internal rate of return. c. Discount rate used in the NPV calculation. . An advantage of the net present value method over the internal rate of return model in
b. Risk-free interest rate. d. Firm’s accounting rate of return. discounted cash flow analysis is that the net present value method
a. Computes a desired rate of return for capital projects. CMA 0695 4-1
67
. The net present value method of capital budgeting assumes that cash flows are reinvested at b. Can be used when there is no constant rate of return required for each year of the project.
a. The risk-free rate. c. The internal rate of return of the project. c. Uses a discount rate that equates the discounted cash inflows with the outflows.
b. The cost of debt. d. The discount rate used in the analysis. d. Uses discounted cash flows whereas the internal rate of return model does not.
CMA 1295 4-9
28. The net present value capital budgeting technique can be used when cash flows from period to
36. Which of the following statements is true regarding capital budgeting methods? (D) period are:
a. The Fisher rate can never exceed a company's cost of capital. AICPA adapted A. B. C. D.
b. The internal rate of return measure used for capital project evaluation has more Uniform No No Yes Yes
conservative assumptions than the net present value method, especially for projects that Uneven Yes No No Yes
generate a positive net present value.
c. The net present value method of project evaluation will always provide the same ranking 53. In situations where the required rate of return is not constant for each year of the project, it is
of projects as the profitability index method. advantageous to use (E)
d. The net present value method assumes that all cash inflows can be reinvested at the a. the adjusted rate-of-return method. c. the net present value method.
project's cost of capital. Barfield b. the internal rate-of-return method. d. sensitivity analysis. Horngren
10. The advantage of the Net Present Value method over the Internal Rate of Return method for Disadvantage
screening investment projects is that it: 69
. A disadvantage of the net present value method of capital expenditure evaluations is that it (M)
a. does not consider the time value of money a. Is calculated using sensitivity analysis.
b. implicitly assumes that the company is able to reinvest cash flows from the project at the b. Computes the true interest rate.
company’s discount rate c. Does not provide the true rate of return on investment.
c. implicitly assumes that the company is able to reinvest cash flows from the project at the d. Is difficult to apply because it uses a trial-and-error approach. CMA 1295 4-16 RPCPA
internal rate of return 0597
d. fails to consider the timing of cash flows Pol Bobadilla
Application
16. The capital budgeting method that assumes that funds are reinvested at the company's cost of 35. NPV is appropriate to use to analyze which decision relating to a joint-products company?
capital is:
CMA EXAMINATION QUESTIONS Page 27 of 107
MANAGEMENT ADVISORY SERVICES CAPITAL BUDGETING
70
a. Whether or not to sell facilities now used for additional processing of one of the joint . The discount rate ordinarily used in present value calculation is the
products. a. Federal Reserve rate.
b. Whether or not to acquire facilities needed for additional processing of one of the joint b. Treasury bill rate.
products. c. Minimum desired rate of return set by the firm.
c. Whether or not to sell facilities now used to operate the joint process. d. Prime rate. Gleim
d. All of the above. L&H
19. A firm's discount rate is typically based on (M)
Variables a. the interest rates related to the firm's bonds.
Required Rate of Return b. a project's internal rate of return.
32. Net present value is calculated using (E) c. its cost of capital.
a. the internal rate of return. d. the corporate Aa bond yield. Barfield
b. the required rate of return.
c. the rate of return required by the investment bankers. 20. In capital budgeting, a firm's cost of capital is frequently used as the (M)
d. none of the above. Horngren a. internal rate of return. c. discount rate.
b. accounting rate of return. d. profitability index. Barfield
13. The rate of return is also called:
71
A. Discount rate C. Opportunity cost of capital . When using the net present value method for capital budgeting analysis, the required rate of
B. Hurdle rate D. All of the above. B&M return is called all of the following except the
A. Risk-free rate. C. Discount rate.
39. Which of the following is NOT an appropriate term for the required rate of return? (E) B. Cost of capital. D. Cutoff rate. CMA 1292 4-16
a. Discount rate c. Cost of capital Horngren
72
b. Hurdle rate d. All of the above are appropriate terms . All of the following are the rates used in net present value analysis except for the
a. Cost of capital. c. Discount rate.
18. The interest rate used to find the present value of a future cash flow is the b. Hurdle rate. d. Accounting rate of return. CMA 0694 4-15
a. prime rate. c. cutoff rate.
b. discount rate. d. internal rate of return. Barfield Net Investment
73
. A project’s net present value, ignoring income tax considerations, is normally affected by the
54. By using the required rate of return of an equivalent security traded in the financial markets as a. Proceeds from the sale of the asset to be replaced.
a discount rate in the NPV calculations, we are: b. Carrying amount of the asset to be replaced by the project.
A. Discounting for time C. A and B above c. Amount of annual depreciation on the asset to be replaced. AICPA 0593 T-47
B. Discounting for risk D. None of the above B&M d. Amount of annual depreciation on fixed assets used directly on the project.

52. The discount rate is used for calculating the NPV is: Working Capital
A. Determined by the financial market 22. Some investment projects require that a company expand its working capital to service the
B. Found by the government greater volume of business that will be generated. Under the net present value method, the
C. Found by the CEO investment of working capital should be treated as: (M)
D. None of the above B&M a. an initial cash outflow for which no discounting is necessary.
b. a future cash inflow for which discounting is necessary.
CMA EXAMINATION QUESTIONS Page 28 of 107
MANAGEMENT ADVISORY SERVICES CAPITAL BUDGETING

c. both an initial cash outflow for which no discounting is necessary and a future cash inflow B. Taking all projects with NPVs greater than the cost of investment
for which discounting is necessary. C. Taking all projects with NPVs greater than present value of cash flow
d. irrelevant to the net present value analysis. G & N 9e D. All of the above B&M

Salvage Value 43. If the net present value for a project is zero or positive, this means (E)
31. A proposed project has an expected economic life of eight years. In the calculation of the net a. the project should be accepted.
present value (NPV) of the project, salvage value would be: b. the project should not be accepted.
A. excluded from the calculation of the NPV c. the expected rate of return is below the required rate of return.
B. included as a cash inflow at the estimated salvage value AICPA adapted d. both (a) and (c). Horngren
C. included as a cash inflow at the future amount of the estimated salvage value
D. included as a cash inflow at the present value of the estimated salvage value 32. According to the net present value rule, an investment in a project should be made if the:
A. Net present value is greater than the cost of investment
Comprehensive B. Net present value is greater than the present value of cash flows
19. How   are   the   following   used   in   the   calculation   of   the   net C. Net present value is positive
present   value   of   a   proposed   project?   Ignore   income   tax D. Net present value is negative B&M
considerations. (M)
AICPA adapted A. B. C. D. 35. Which of the following statements regarding the net present value rule and the rate of return
Depreciation expense Include Include Exclude Exclude rule is true?
Salvage value Include Exclude Include Exclude A. Accept a project if the rate of return is positive
B. Accept a project the rate of return on a risky project exceeds the risk-free rate
20. The net present value method takes into account: (M) C. Accept a project if the net present value is positive
AICPA adapted A. B. C. D. D. None of the above statements are true B&M
Cash Flow Over Life of Project No No Yes Yes
Time Value of Money Yes No No Yes 38. In using the net present value method, only projects with a zero or positive net present value
are acceptable because (E)
Formula a. the return from these projects equals or exceeds the cost of capital.
50. The present value formula for one period cash flow is: b. a positive net present value on a particular project guarantees company profitability.
A. PV = C1(1 + r) C. PV = C1/(1 + r) c. the company will be able to pay the necessary payments on any loans secured to finance
the project.
B. PV = C1/r D. None of the above B&M d. of both (a) and (b). Horngren

51. The net present value formula for one period is: 33. Which of the following statements regarding the net present value rule and the rate of return
A. NPV = PV cash flows - initial investment C. NPV = C0/[C1(1 + r)] rule is not true?
B. NPV = C0/C1 D. None of the above B&M A. Accept a project if NPV > cost of investment
B. Accept a project if NPV is positive
Decision Criteria C. Accept a project if return on investment exceeds the rate of return on an equivalent
43. The managers of a firm can maximize stockholder wealth by: investment in the financial market
A. Taking all projects with positive NPVs D. All of the above statements are true B&M
CMA EXAMINATION QUESTIONS Page 29 of 107
MANAGEMENT ADVISORY SERVICES CAPITAL BUDGETING

discount rate
17. The following statements regarding the NPV rule and the rate of return rule are true except: C. The same as the NPV value obtained by discounting nominal cash flows using the real
A. Accept a project if its NPV > 0 discount rate
B. Reject a project if its NPV < 0 D. None of the above B&M
C. Accept a project if its rate of return > 0
D. Accept a project if its rate of return > opportunity cost of capital B&M INTERNAL RATE OF RETURN
Definition
34. According to the rate of return rule an investment in a risky project should be made if: *. The discount rate that equates the present value of the expected cash flows with the cost of
A. The return on investment exceeds the risk-free rate the investment is the (E)
B. The return on investment is positive a. Net present value c. Accounting rate of return
C. The return on investments exceeds the rate of return on an equivalent investment in the b. Internal rate of return d. Payback period RPCPA 0593
financial market
75
D. None of the above statements are true B&M . The technique that incorporates the time value of money by determining the compound
interest rate of an investment such that the present value of the after-tax cash inflows over the
NPV in an Inflationary Environment life of the investment is equal to the initial investment is called the
13. Which of the following statements is true? A. Internal rate of return method. C. Profitability index method. CMA 1290 4-16
A. Nominal cash flows are discounted using nominal discount rate B. Capital asset pricing model. D. Accounting rate of return method.
B. Nominal cash flows are discounted using the real discount rate
C. Real cash flows are discounted using the nominal discount rate
D. None of the above statements are true B&M
74
. When determining net present value in an inflationary environment, adjustments should be
made to
a. Increase the discount rate, only.
b. Increase the estimated cash inflows and increase the discount rate.
c. Increase the estimated cash inflows but not the discount rate.
d. Decrease the estimated cash inflows and increase the discount rate. CMA 1293 4-21

18. Proper treatment of inflation in the NPV calculation involves:


A. Discounting nominal cash flows using the nominal discount rate
B. Discounting real cash flows using the real discount rate
C. Discounting nominal cash flows using the real discount rates
D. A and B B&M

20. The NPV value obtained by discounting nominal cash flows using the nominal discount rate is:
A. The same as the NPV value obtained by discounting real cash flows using the real
discount rate
B. The same as the NPV value obtained by discounting real cash flows using the nominal
CMA EXAMINATION QUESTIONS Page 30 of 107
MANAGEMENT ADVISORY SERVICES CAPITAL BUDGETING
76
. The internal rate of return (IRR) is the b. a trial-and-error approach d. a time line Barfield
a. Hurdle rate.
b. Rate of interest for which the net present value is greater than 1.0. 47. The capital budgeting method that calculates the discount rate at which the present value of
c. Rate of interest for which the net present value is equal to zero. expected cash inflows from a project equals the present value of expected cash outflows is the
d. Rate of return generated from the operational cash flows. CMA 0694 4-16, RPCPA 1096 (E)
a. net present value method.
24. The IRR is defined as: (E) b. accrual accounting rate-of-return method.
A. The discount rate that makes the NPV equal to zero c. payback method.
B. The difference between the cost of capital and the present value of the cash flows d. internal rate of return. Horngren
C. The discount rate used in the NPV method
D. The discount rate used in the discounted payback period method B&M 21 The internal rate of return of a capital investment(M)
a. Changes when the cost of capital changes.
46. The rate of interest that produces a zero net present value when a project's discounted cash b. Is equal to the annual net cash flows divided by one half of the project’s cost when the
operating advantage is netted against its discounted net investment is the cash flows are an annuity.
a. cost of capital. c. cutoff rate. c. Must exceed the cost of capital in order for the firm to accept the investment.
b. discount rate. d. internal rate of return. Barfield d. Is similar to the yield to maturity on a bond.
e. Statements c and d are correct. Brigham
77
. The internal rate of return for a project can be determined
A. If the internal rate of return is greater than the firm's cost of capital. Variables
B. Only if the project cash flows are constant. CMA 1293 4-12 *. The capital budgeting technique known as internal rate of return uses (E)
C. By finding the discount rate that yields a net present value of zero for the project. RPCPA 0598 a. b. c. d.
D. By subtracting the firm's cost of capital from the project's profitability index. Cash flow over entire life of project No Yes Yes No
Time value of money Yes Yes No No
78
. The internal rate of return is
a. The discount rate at which the NPV of the cash flows is zero. 80
. How are the following used in the calculation of the internal rate of return of a proposed
b. The breakeven borrowing rate for the project in question. project? Ignore income tax considerations.
c. The yield rate/effective rate of interest quoted on long-term debt and other instruments. a. b. c. d.
d. All of the answers are correct. AICPA 1181 I-39 Residual sales value of project Exclude Include Exclude Include
79
Depreciation expense Include Include Exclude Exclude
. The internal rate of return is
A. The breakeven borrowing rate for the project in question.
38. If Co. X wants to use IRR to evaluate long-term decisions and to establish a cutoff rate of
B. The yield rate/effective rate of interest quoted on long-term debt and other instruments.
return, X must be sure the cutoff rate is (E)
C. Favorable when it exceeds the hurdle rate.
a. At least equal to its cost of capital.
D. All of the answers are correct. Gleim
b. At least equal to the rate used by similar companies.
c. Greater than the IRR on projects accepted in the past.
17. When a project has uneven projected cash inflows over its life, an analyst may be forced to
d. Greater than the current book rate of return. L&H
use _______________ to find the project's internal rate of return.
a. a screening decision c. a post investment audit
CMA EXAMINATION QUESTIONS Page 31 of 107
MANAGEMENT ADVISORY SERVICES CAPITAL BUDGETING

Assumption/Disadvantage B. accounting rate of return on initial investment method


48. Which of the following capital expenditure planning and control techniques has been criticized C. payback method
because it might mistakenly imply that earnings are reinvested at the rate of return earned by D. average return on investment method
the investment? (M) E. present value method AICPA adapted
a. payback method c. net present value method
b. accounting rate of return d. internal rate of return Barfield 25. The following are some of the shortcomings of the IRR method except: (E)
A. IRR is conceptually easy to communicate
81
. The net present value (NPV) method and the internal rate of return (IRR) method are used to B. Projects can have multiple IRRs B&M
analyze capital expenditures. The IRR method, as contrasted with the NPV method, C. IRR method cannot distinguish between a borrowing project and a lending project
A. Is considered inferior because it fails to calculate compounded interest rates. D. It is very cumbersome to evaluate mutually exclusive projects using the IRR method
B. Incorporates the time value of money whereas the NPV method does not.
C. Assumes that the rate of return on the reinvestment of the cash proceeds is at the Advantage
83
indicated rate of return of the project analyzed rather than at the discount rate used. . Which of the following characteristics represent an advantage of the internal rate of return
D. Is preferred in practice because it is able to handle multiple desired hurdle rates, which is techniques over the accounting rate of return technique in evaluating a project? (M)
impossible with the NPV method. CMA 1291 4-7 I Recognition of the project’s salvage value.
II Emphasis on cash flows.
82
. A weakness of the internal rate of return (IRR) approach for determining the acceptability of III Recognition of the time value of money.
investments is that it (E) a. I only. c. II and III.
a. Does not consider the time value of money. b. I and II. d. I, II, and III. AICPA 1192 T-49
b. Is not a straightforward decision criterion.
c. Implicitly assumes that the firm is able to reinvest project cash flows at the firm’s cost of Decision Criteria
capital. 48. In capital budgeting, a project is accepted only if the internal rate of return (E)
d. Implicitly assumes that the firm is able to reinvest project cash flows at the project’s a. equals or exceeds the required rate of return.
internal rate of return. CMA 1292 4-13 b. equals or is less than the required rate of return.
c. equals or exceeds the net present value.
25. A weakness of the internal rate of return method for screening investment projects is that it: d. equals or exceeds the accrual accounting rate of return. Horngren
(M)
a. does not consider the time value of money. Comprehensive
b. implicitly assumes that the company is able to reinvest cash flows from the project at the 23. Your company is comparing internal rate of return to net present value computations as
company's discount rate. alternative criteria for evaluating potential capital investments. Which of the following best
c. implicitly assumes that the company is able to reinvest cash flows from the project at the describes these computations?
internal rate of return. A. The internal rate of return method ignores the initial cost of the investment in its
d. does not take into account all of the cash flows from a project. CMA adapted computations.
B. The net present value method ignores the company's cost of capital.
10. Which of the following capital expenditure planning and control techniques has been criticized C. The net present value method is more appropriate to use during periods of inflation.
because it might mistakenly imply that earnings are reinvested at the rate of return earned by D. The two methods will give the same rankings because they both consider the time value
the investment? of money. CIA adapted
A. internal rate of return method
CMA EXAMINATION QUESTIONS Page 32 of 107
MANAGEMENT ADVISORY SERVICES CAPITAL BUDGETING

E. The internal rate of return method assumes that the positive cash flows generated each a. The MIRR method will always arrive at the same conclusion as the NPV method.
year are reinvested at the computed rate of return for the investment being evaluated. b. The MIRR method can overcome the multiple IRR problem, while the NPV method
cannot.
11. A company is considering the purchase of a new conveyor belt system for carrying parts and c. The MIRR method uses a more reasonable assumption about reinvestment rates than the
subassemblies from building to building within its plant complex. It is expected that the system IRR method.
will have a useful life of at least ten years and that it will substantially reduce labor and waiting- d. Statements a and c are correct.
time costs. If the company's average cost of capital is about 15% and if some evaluation must e. All of the statements above are correct. Brigham
be made of cost/benefit relationships (including the effects of interest) to determine the
desirability of the purchase, the most relevant quantitative technique for evaluating the RELATIONSHIP among Payback, ARR, PI, NPV & IRR
investment is: ARR vs. IRR
A. present value (or internal rate of return) analysis 11. Which of the following is a basic difference between the IRR and the book rate of return (BRR)
B. Program Evaluation and Review Technique (PERT) criteria for evaluating investments?
C. accounting rate of return analysis a. IRR emphasizes expenses and BRR emphasizes expenditures.
D. cost-volume-profit analysis b. IRR emphasizes revenues and BRR emphasizes receipts.
E. payback analysis AICPA adapted c. IRR is used for internal investments and BRR is used for external investments.
d. IRR concentrates on receipts and expenditures and BRR concentrates on revenues and
*. Statement 1 The internal rate of return is the discount rate that equals the amount invested at expenses. L&H
a given date with the present value of the expected cash inflows from the
investment. Payback & NPV
Statement 2 If the minimum desired rate of return exceeds the internal rate of return expected 12. If a project has a payback period shorter than its life,
from a project, the project should be rejected. a. Its NPV may be negative.
Statement 3 The internal rate of return can be more easily applied to situations with uneven b. Its IRR is greater than cost of capital.
periodic cash flows than can the net present value. (M) c. It will have a positive NPV.
RPCPA 0592 a. b. c. d. d. Its incremental cash flows may not cover its cost. L&H
Statement 1 True False True False
Statement 2 True False True False Payback & IRR
Statement 3 True False False True 5. The relationship between payback period and IRR is that
a. A payback period of less than one-half the life of a project will yield an IRR lower than the
MODIFIED INTERNAL RATE OF RETURN target rate.
22 Which of the following statements is most correct? The modified IRR (MIRR) method: (M) b. The payback period is the present value factor for the IRR.
a. Always leads to the same ranking decision as NPV for independent projects. c. A project whose payback period does not meet the company’s cutoff rate for payback will
b. Overcomes the problem of multiple rates of return. not meet the company’s criterion for IRR.
c. Compounds cash flows at the cost of capital. d. None of the above. L&H
d. Overcomes the problems of cash flow timing and project size that lead to criticism of the
regular IRR method. PI and NPV
e. Statements b and c are correct. Brigham 31. An investment whose profitability index is 1.00
a. Has an IRR equal tot eh prevailing interest rate.
84
. Which of the following statements is most correct? (M) b. Returns to the company only the cash outlay for the investment.
CMA EXAMINATION QUESTIONS Page 33 of 107
MANAGEMENT ADVISORY SERVICES CAPITAL BUDGETING

c. Has a payback period equal to its useful life. c. Greater than one.
d. Has an NPV of zero. L&H d. Cannot be determined without more information. L&H

38. If a project generates a net present value of zero, the profitability index for the project will 47. A project has an IRR in excess of the cost of capital. The profitability index for this project
a. equal zero. c. equal -1. would be (M)
b. equal 1. d. be undefined. Barfield a. Less than zero.
b. Between zero and one.
40. If a project's profitability index is less than 1, the project's (E) c. Greater than one.
a. discount rate is above its cost of capital. c. payback period is infinite. d. Cannot be determined without more information. L&H
b. internal rate of return is less than zero. d. net present value is negative. Barfield
NPV and IRR
86
39. If the profitability index for a project exceeds 1, then the project's . Polo Co. requires higher rates of return for projects with a life span greater than 5 years.
a. net present value is positive. Projects extending beyond 5 years must earn a higher specified rate of return. Which of the
b. internal rate of return is less than the project's discount rate. following capital budgeting techniques can readily accommodate this requirement? (M)
c. payback period is less than 5 years. AICPA 0590 T-48 a. b. c. d.
d. accounting rate of return is greater than the project's internal rate of return. Barfield Internal Rate of Return Yes No No Yes
Net Present Value No Yes No Yes
15. An investment with a positive NPV also has
a. A positive profitability index. 49. If the discount rate that is used to evaluate a project is equal to the project's internal rate of
b. A profitability index of one. return, the project's _________ is zero.
c. A profitability index less than one. a. profitability index c. present value of the investment
d. A profitability index greater than one. L&H b. internal rate of return d. net present value Barfield
85
. If an investment project has a profitability index of 1.15, the 22. If an investment project (normal project) has an IRR equal to the cost of capital, the NPV for
a. Project’s internal rate of return is 15%. that project is: (E)
b. Project’s cost of capital is greater than its internal rate of return. A. Positive C. Zero
c. Project’s internal rate of return exceeds its net present value. B. Negative D. Unable to be determined B&M
d. Net present value of the project is positive. CMA 1293 4-11
26. If the net present value of a project is zero based on a discount rate of sixteen percent, then
PI & IRR the time-adjusted rate of return: (M)
21. If the profitability index is less than one, a. is equal to sixteen percent.
a. The IRR is less than cost of capital. c. The IRR is greater than cost of capital. b. is less than sixteen percent.
b. The IRR is the same as cost of capital. d. None of the above is true. L&H c. is greater than sixteen percent.
d. cannot be determined from the information given. G & N 9e
48. A project has an IRR less than the cost of capital. The profitability index for this project would
be 4. If the present value of the future cash flows for an investment equals the required investment,
a. Less than zero. the IRR is (D)
b. Between zero and one. a. Equal to the cutoff rate.
CMA EXAMINATION QUESTIONS Page 34 of 107
MANAGEMENT ADVISORY SERVICES CAPITAL BUDGETING
89
b. Equal to the cost of borrowed capital. . The net present value of a proposed investment is negative; therefore, the discount rate used
c. Equal to zero. must be
d. Lower than the company’s cutoff rate of return. L&H A. Greater than the project's internal rate of return.
B. Less than the project's internal rate of return.
87
. If a prospective investment has a positive net present value at a company's cost of capital of C. Greater than the firm's cost of equity.
15%, it can be concluded that D. Less than the risk-free rate. CMA 1295 4-14, RPCPA 0596
A. The accounting rate of return of the project is greater than 15%.
B. The internal rate of return of the project is equal to the accounting rate of return. 25. At a company's cost of capital of 15%, a prospective investment has a negative net present
C. The payback period of the associated asset is shorter than its life. value. Based on this information, it can be concluded that:
D. The internal rate of return of the project is greater than 15%. CIA 0R98 IV-37 A. the internal rate of return is greater than 15%
B. the payback period is shorter than the life of the asset
45. If an investment has a positive net present value, the C. the accounting rate of return is less than 15%
a. internal rate of return is higher than the discount rate. D. the accounting rate of return is greater than 15%
b. discount rate is higher than the hurdle rate of return. E. the internal rate of return is less than 15% CIA adapted
c. internal rate of return is lower than the discount rate of return.
90
d. hurdle rate of return is higher than the discount rate. Barfield . Which of the following statements is most correct? (E)
a. If a project’s internal rate of return (IRR) exceeds the cost of capital, then the project’s net
7. If an investment has a positive NPV present value (NPV) must be positive.
a. Its IRR is greater than the company’s cost of capital. b. If Project A has a higher IRR than Project B, then Project A must also have a higher NPV.
b. Cost of capital exceeds the cutoff rate of return. c. The IRR calculation implicitly assumes that all cash flows are reinvested at a rate of return
c. Its IRR is less than the company’s cutoff rate of return. equal to the cost of capital.
d. The cutoff rate of return exceeds cost of capital. L&H d. Statements a and c are correct. Brigham
91
*. Lenders Inc. is considering an investment that has a positive net present value based on its . Which of the following statements is most correct? (M)
16% hurdle rate. The internal rate of return would be (M) a. If a project with normal cash flows has an IRR which exceeds the cost of capital, then the
a. More than 16%. c. 16%. project must have a positive NPV.
b. Less than 16%. d. Zero. RPCPA 1095 b. If the IRR of Project A exceeds the IRR of Project B, then Project A must also have a
higher NPV.
88
. Neu Co. is considering the purchase of an investment that has a positive net present value c. The modified internal rate of return (MIRR) can never exceed the IRR.
based on Neu’s 12% hurdle rate. The internal rate of return would be d. Statements a and c are correct. Brigham
a. 0%. c. >12%
92
b. 12%. d. < 12% . Project A has an internal rate of return (IRR) of 15 percent. Project B has an IRR of 14
percent. Both projects have a cost of capital of 12 percent. Which of the following statements
18. An investment has a positive NPV discounting the cash flows at a 14% cost of capital. Which is most correct? (E)
statement is true? a. Both projects have a positive net present value (NPV).
a. The IRR is lower than 14%. c. The payback period is less than 14 years. b. Project A must have a higher NPV than Project B.
b. The IRR is higher than 14%. d. The book rate of return is 14%. L&H c. If the cost of capital were less than 12 percent, Project B would have a higher IRR than
Project A.
CMA EXAMINATION QUESTIONS Page 35 of 107
MANAGEMENT ADVISORY SERVICES CAPITAL BUDGETING

d. Statements a and c are correct. b. If the multiple IRR problem does not exist, any independent project acceptable by the
e. All of the statements above are correct. Brigham NPV method will also be acceptable by the IRR method.
c. If IRR = k (the cost of capital), then NPV = 0.
93
. Project A has an IRR of 15 percent. Project B has an IRR of 18 percent. Both projects have d. NPV can be negative if the IRR is positive.
the same risk. Which of the following statements is most correct? (E) e. The NPV method is not affected by the multiple IRR problem. Brigham
a. If the WACC is 10 percent, both projects will have a positive NPV, and the NPV of Project
97
B will exceed the NPV of Project A. . A project has an up-front cost of $100,000. The project’s WACC is 12 percent and its net
b. If the WACC is 15 percent, the NPV of Project B will exceed the NPV of Project A. present value is $10,000. Which of the following statements is most correct? (E)
c. If the WACC is less than 18 percent, Project B will always have a shorter payback than a. The project should be rejected since its return is less than the WACC.
Project A. b. The project’s internal rate of return is greater than 12 percent.
d. If the WACC is greater than 18 percent, Project B will always have a shorter payback than c. The project’s modified internal rate of return is less than 12 percent.
Project A. d. All of the statements above are correct. Brigham
e. If the WACC increases, the IRR of both projects will decline. Brigham
NPV, IRR & Payback
94
. Project J has the same internal rate of return as Project K. Which of the following statements 37. A project that has a negative NPV.
is most correct? (M) a. Has a payback period longer than its life.
a. If the projects have the same size (scale) they will have the same NPV, even if the two b. Has a negative profitability index.
projects have different levels of risk. c. Must be rejected.
b. If the two projects have the same risk they will have the same NPV, even if the two d. Doesn’t necessarily fit any of the above descriptions. L&H
projects are of different size.
98
c. If the two projects have the same size (scale) they will have the same discounted . Project X has an internal rate of return of 20 percent. Project Y has an internal rate of return of
payback, even if the two projects have different levels of risk. 15 percent. Both projects have a positive net present value. Which of the following
d. All of the statements above are correct. statements is most correct? (M)
e. None of the statements above is correct. Brigham a. Project X must have a higher net present value than Project Y.
b. If the two projects have the same WACC, Project X must have a higher net present value.
NPV, IRR & WACC c. Project X must have a shorter payback than Project Y.
95
. Assume a project has normal cash flows (that is, the initial cash flow is negative, and all other d. Statements b and c are correct.
cash flows are positive). Which of the following statements is most correct? (E) e. None of the statements above is correct. Brigham
a. All else equal, a project’s IRR increases as the cost of capital declines.
b. All else equal, a project’s NPV increases as the cost of capital declines. PI, NPV & IRR
c. All else equal, a project’s MIRR is unaffected by changes in the cost of capital. 22. Which of the following combinations is possible? L&H
d. Statements a and b are correct. Profitability Index NPV IRR
e. Statements b and c are correct. Brigham a. Greater than 1 Positive Equal cost of capital
b. Greater than 1 Negative Less that cost of capital
. Which of the following statements is incorrect? (M)
96
c. Less than 1 Negative Less than cost of capital
a. Assuming a project has normal cash flows, the NPV will be positive if the IRR is less than d. Less than 1 Positive Less than cost of capital
the cost of capital.
23. Which of the following combinations is NOT possible? (E) L&H
CMA EXAMINATION QUESTIONS Page 36 of 107
MANAGEMENT ADVISORY SERVICES CAPITAL BUDGETING

Profitability Index NPV IRR Payback Included Excluded Included Excluded


a. Greater than 1 Positive More than cost of capital
b. Equals 1 Zero Equals cost of capital *. If income tax considerations are ignored, how is depreciation used in the following capital
c. Less than 1 Negative Less than cost of capital budgeting techniques? (E)
d. Less than 1 Positive Less than cost of capital RPCPA 0595 a. b. c. d.
Internal rate of return Included Excluded Excluded Included
NPV, IRR, MIRR, and Payback Accounting rate of return Excluded Included Excluded Included
99
. A proposed project has normal cash flows. In other words, there is an up-front cost followed
over time by a series of positive cash flows. The project’s internal rate of return is 12 percent 30. If income tax considerations are ignored, how is depreciation expense used in the following
and its WACC is 10 percent. Which of the following statements is most correct? (E) capital budgeting techniques?
a. The project’s NPV is positive. AICPA adapted A. B. C. D.
b. The project’s MIRR is greater than 10 percent but less than 12 percent. Internal Rate of Return Excluded Excluded Included Included
c. The project’s payback period is greater than its discounted payback period. Net Present Value Excluded Included Excluded Included
d. Statements a and b are correct. Brigham
100
. If income tax considerations are ignored, how is depreciation handled by the following
COMPARISION among Payback, ARR, PI, NPV & IRR budgeting technique?
NPV vs.IRR
CMA 1293 4-17 a. b. c. d.
52. An important advantage of the net present value method of capital budgeting over the internal
rate-of-return method is (E) Internal Rate of Return Excluded Included Excluded Included
a. the net present value method is expressed as a percentage. Accounting Rate of Return Included Excluded Excluded Included
b. the net present values of individual projects can be added to determine the effects of Payback Excluded Included Included Included
accepting a combination of projects.
c. no advantage. Cash Flow
d. both (a) and (b). Horngren 24. Which   of   the   following   capital   budgeting   techniques
consider(s) cash flow over the entire life of the project? (E)
Depreciation AICPA adapted A. B. C. D.
23. (Ignore income taxes in this problem.) How is depreciation Internal Rate of Return Yes Yes No No
handled by the following capital budgeting techniques? (M) Payback Yes No Yes No
CMA adapted A. B. C. D.
Internal Rate of Return Excluded Included Excluded Included PROJECT SCREENING (Accept/Reject Decision for Independent Project)
Simple Rate of Return Included Excluded Excluded Included 59. Which of the following best represents a screening decision?
Payback Excluded Included Excluded a. determining which project has the highest net present value
b. determining if a project's internal rate of return exceeds the firm's cost of capital
19. If income tax considerations are ignored, how is depreciation expense used in the following c. determining which projects are mutually exclusive
capital budgeting techniques? d. determining which are the best projects Barfield
AICPA adapted A. B. C. D.
Internal Rate of Return Excluded Included Included Excluded

CMA EXAMINATION QUESTIONS Page 37 of 107


MANAGEMENT ADVISORY SERVICES CAPITAL BUDGETING

Security Market Line (SML) Concept 14. The NPV and IRR methods give
. Using the Security Market Line concept in capital budgeting, which of the following is correct? a. The same decision (accept or reject) for any single investment.
(M) b. The same choice from among mutually exclusive investments.
a. If the expected rate of return on a given capital project lies above the SML, the project c. Different rankings of projects with unequal lives.
should be accepted even if its beta is above the beta of the firm’s average project. d. The same rankings of projects with different required investments. L&H
b. If a project’s return lies below the SML, it should be rejected if it has a beta greater than
the firm’s existing beta but accepted if its beta is below the firm’s beta. 29. Which of the following is always true of the net present value (NPV) approach?
c. If two mutually exclusive projects’ expected returns are both above the SML, the project A. If a project is found to be acceptable under the NPV approach, it would also be acceptable
with the lower risk should be accepted. under the internal rate of return (IRR) approach.
d. If a project’s expected rate of return is greater than the expected rate of return on an B. The NPV and the IRR approaches will always rank projects in the same order.
average project, it should be accepted. Brigham C. If a project is found to be acceptable under the NPV approach, it would also be acceptable
under the payback approach. CIA adapted
23. On a graph with common stock returns on the Y axis and market returns on the X-axis, the D. The NPV and the payback approaches will always rank projects in the same order.
slope of the regression line represents the:
A. Alpha C. R-squared Questions 140 and 141 are based on the following information. CIA 0594 IV-45 & 46
B. Beta D. Adjusted beta B&M The financial management team of a company is assessing an investment proposal involving a
$100,000 outlay today. Manager one expects the project to provide cash inflows of $20,000 at the
15. The historical returns data for the past three years for Company A's stock is -6.0%, 15%, 15% end of each year for 6 years. She considers the project to be of low risk, requiring only a 10% rate
and that of the market portfolio is 10%, 10% and 16%. According to the SML, the Stock A is: of return. Manager two expects the project to provide cash inflows of $5,000 at the end of the first
A. over priced C. Correctly priced year, followed by $23,000 at the end of each year in years two through six. He considers the
B. Under priced D. Need more information B&M project to be of medium risk, requiring a 14% rate of return. Manager three expects the project to
be of high risk, providing one large cash inflow of $135,000 at the end of the sixth year. She
22. The historical returns data for the past three years for Stock B and the stock market portfolio proposes a 15% rate of return for the project.
are: Stock B:- 24%, 0%, 24%, Market Portfolios:- 10%, 12%, 20%. According to the SML the Additional Information:
stock B is: Number Discount Present Value of $1 Due at Annuity of $1 per Period
A. Overpriced C. Correctly priced of Years Rate (%) the End of n Periods (PVIF) for n Periods (PVIFA)
B. Underpriced B&M 1 10 .9091 .9091
1 14 .8772 .8772
Single Project 1 15 .8696 .8696
101
. The profitability index approach to investment analysis (M) 5 10 .6209 3.7908
A. Fails to consider the timing of project cash flows. 5 14 .5194 3.4331
B. Considers only the project's contribution to net income and does not consider cash flow 5 15 .4972 3.3522
effects. 6 10 .5645 4.3553
C. Always yields the same accept/reject decisions for independent projects as the net 6 14 .4556 3.8887
present value method. 6 15 .4323 3.7845
D. Always yields the same accept/reject decisions for mutually exclusive projects as the net 102
. According to the net present value criterion, which of the following is true?
present value method. CMA 1292 4-14, RPCPA 0596 A. Manager one will recommend that the project be accepted.
B. Manager two will recommend that the project be accepted.
CMA EXAMINATION QUESTIONS Page 38 of 107
MANAGEMENT ADVISORY SERVICES CAPITAL BUDGETING
106
C. All three managers will recommend acceptance of the project. . Barker Inc. has no capital rationing constraint and is analyzing many independent investment
D. All three managers will recommend rejection of the project. alternatives. Barker should accept all investment proposals
a. If debt financing is available for them.
103
. Which manager will assess the project as having the shortest payback period? b. That have positive cash flows.
A. Manager one. c. That provide returns greater than the after-tax cost of debt.
B. Manager two. d. That have a positive net present value. CMA 1295 4-2
C. Manager three.
D. All three managers will agree on the payback period. Independent Projects
107
. An organization is using capital budgeting techniques to compare two independent projects. It
Group Project could accept one, both, or neither of the projects. Which of the following statements is true
6. You are given a job to make a decision on project X, which is composed of three projects A, B, about the use of net-present-value (NPV) and internal-rate-of-return (IRR) methods for
and C which have NPVs of +$50, -$20 and +$100, respectively. How would you go about evaluating these two projects?
making the decision about whether to accept or reject the project? (M) a. NPV and IRR criteria will always lead to the same accept or reject decision for two
A. Accept the firm's joint project as it has a positive NPV independent projects.
B. Reject the joint project b. If the first project’s IRR is higher than the organization’s cost of capital, the first project will
C. Break up the project into its components: accept A and C and reject B be accepted but the second project will not.
D. None of the above B&M c. If the NPV criterion leads to accepting or rejecting the first project, one cannot predict
whether the IRR criterion will lead to accepting or rejecting the first project.
Unlimited Capital d. If the NPV criterion leads to accepting the first project, the IRR criterion will never lead to
104
. A company has unlimited capital funds to invest. The decision rule for the company to follow in accepting the first project. CIA 0597 IV-43
order to maximize shareholders' wealth is to invest in all projects having a(n)
108
A. Present value greater than zero. . Which of the following is always true with regard to the net present value (NPV) approach?
B. Net present value greater than zero. A. If a project is found to be acceptable under the NPV approach, it would also be acceptable
C. Internal rate of return greater than zero. CMA 1293 4-15 under the internal rate of return (IRR) approach.
D. Accounting rate of return greater than the hurdle rate used in capital budgeting analyses. B. The NPV and the IRR approaches will always rank projects in the same order.
C. If a project is found to be acceptable under the NPV approach, it would also be acceptable
105
. Future, Inc. is in the enviable situation of having unlimited capital funds. The best decision rule, under the payback approach. CIA 0586 IV-29
in an economic sense, for it to follow would be to invest in all projects in which the D. The NPV and payback approaches will always rank projects in the same order.
A. Accounting rate of return is greater than the earnings as a percent of sales.
B. Payback reciprocal is greater than the internal rate of return. PROJECT RANKING METHOD
C. Internal rate of return is greater than zero. Capital Rationing
D. Net present value is greater than zero. CMA 1278 5-12 38. Soft rationing is imposed in order to: (E)
A. Control managers' actions C. Cope with market imperfections
B. Limit runaway growth D. A and B B&M

39. Hard rationing is imposed by the: (E)


A. Market C. Management
B. Majority stockholder D. Both B and C B&M
CMA EXAMINATION QUESTIONS Page 39 of 107
MANAGEMENT ADVISORY SERVICES CAPITAL BUDGETING
111
. The rankings of mutually exclusive investments determined using the internal rate of return
Mutually Inclusive Projects method (IRR) and the net present value method (NPV) may be different when
63. If management judges one project in a mutually inclusive set to be acceptable for investment, a. The lives of the multiple projects are equal and the size of the required investments are
a. all the other projects in the set are rejected. equal.
b. only one other project in the set can be accepted. b. The required rate of return equals that IRR of each project.
c. all other projects in the set are also accepted. c. The required rate of return is higher than the IRR of each project.
d. only one project in the set will be rejected. Barfield d. Multiple projects have unequal lives and the size of the investment for each project is
different. CMA 1292 4-15, RPCPA 1096
Mutually Exclusive Projects
Net Present Value (Preferred Method) 50. Why do the NPV method and the IRR method sometimes produce different rankings of
7. In choosing from among mutually exclusive investments the manager should normally select mutually exclusive investment projects?
the one with the highest (M) A. The NPV method does not assume reinvestment of cash flows while the IRR method
a. NPV. c. Profitability index. assumes the cash flows will be reinvested at the internal rate of return.
b. IRR. d. Book rate of return. L&H B. The NPV method assumes a reinvestment rate equal to the discount rate while the IRR
method assumes a reinvestment rate equal to the internal rate of return.
57. Which of the following capital investment models would be preferred when choosing among C. The IRR method does not assume reinvestment of the cash flows while the NPV assumes
mutually exclusive alternatives? (M) the reinvestment rate is equal to the discount rate.
a. payback period c. IRR D. The NPV method assumes a reinvestment rate equal to the bank loan interest rate while
b. accounting rate of return d. NPV H&M the IRR rate method assumes a reinvestment rate equal to the discount rate. Pol Bobadilla

Profitability Index Ranking Decision


109 112
. When ranking two mutually exclusive investments with different initial amounts, management . Which mutually exclusive project would you select, if both are priced at $1,000 and your
should give first priority to the project discount rate is 15%; Project A with three annual cash flows of $1,000, or Project B, with 3
A. That generates cash flows for the longer period of time. years of zero cash flow followed by 3 years of $1,500 annually?
B. Whose net after-tax flows equal the initial investment. A. Project A.
C. That has the greater accounting rate of return. B. Project B.
D. That has the greater profitability index. CMA 1291 4-6 C. The IRRs are equal, hence you are indifferent.
D. The NPVs are equal, hence you are indifferent. Gleim
NPV & IRR
110
. The internal rate of return on an investment Independent Projects
A. Usually coincides with the company's hurdle rate. Definition
B. Disregards discounted cash flows. *. The kind of investment project which has no direct relationship with other projects and can
C. May produce different rankings from the net present value method on mutually exclusive therefore be implemented or rejected independently of others (E)
projects. a. Independent investment project
D. Would tend to be reduced if a company used an accelerated method of depreciation for b. Complimentary investment project
tax purposes rather than the straight-line method. CMA 0691 4-19 c. None of these RPCPA 0588

Examples
CMA EXAMINATION QUESTIONS Page 40 of 107
MANAGEMENT ADVISORY SERVICES CAPITAL BUDGETING

60. Below are pairs of projects. Which pair best represents independent projects? *. Several proposed capital projects which are economically acceptable may have to be ranked
a. buy computer; buy software package due to constraints in financial resources. In ranking these projects, the least pertinent is this
b. buy computer #1; buy computer #2 statement. (M)
c. buy computer; buy computer security system a. If the internal rate of return method is used in the capital rationing problem, the higher the
d. buy computer; repave parking lot Barfield rate, the better the project.
b. In selecting the required rate of return, one may either calculate the organization’s cost of
Profitability Index capital or use a rate generally acceptable in the industry.
35. Profitability index is useful under: (E) c. A ranking procedure on the basis of quantitative criteria may be established by specifying
A. Capital rationing C. Non-normal projects a minimum desired rate of return, which rate is used in calculating the net present value of
B. Mutually exclusive projects D. None of the above B&M each project.
d. If the net present value method is used, the profitability index is calculated to rank the
40. The profitability index can be used for ranking projects under: (E) projects. The lower the index, the better the project. RPCPA 1094
A. Soft capital rationing C. Capital rationing at t = 0
B. Hard capital rationing D. Both A and B B&M Profitability Index
115
. Capital budgeting methods are often divided into two classifications: project screening and
113
. The recommended technique for evaluating projects when capital is rationed and there are no project ranking. Which one of the following is considered a ranking method rather than a
mutually exclusive projects from which to choose is to rank the projects by screening method?
A. Accounting rate of return. C. Internal rate of return. A. Net present value. C. Profitability index.
B. Payback. D. Profitability index. CMA 0692 4-15 B. Time-adjusted rate of return. D. Accounting rate of return. CMA 0691 4-17
114
. The technique used to evaluate all possible capital projects of different dollar amounts and IRR
116
then rank them according to their desirability is the (M) . A company has analyzed seven new projects, each of which has its own internal rate of return.
a. Profitability index method. c. Payback method. CMA 1294 4-26 It should consider each project whose internal rate of return is _____ its marginal cost of
b. Net present value method. d. Discounted cash flow method. capital and accept those projects in _____ order of their internal rate of return.
A. Below; decreasing. C. Above; increasing.
Ranking Decision B. Above; decreasing. D. Below; increasing. CIA 0593 IV-55
37. A company is evaluating three possible investments. Information relating to the company and
the investments follow: Internal Rate of Return & Net Present Value
Fisher rate for the three projects 7% 22. The three frequently used methods for ranking investment proposals are payback, net present
Cost of capital 8% value, and internal rate of return. One of the three is defined as the interest rate that equates
Based on this information, we know that (D) the present value of expected cash flows and the cost of the project. A second method finds
a. all three projects are acceptable. the present value of expected cash flows and subtracts the initial cost of the project. The
b. none of the projects are acceptable. following terms that match these respective definitions are:
c. the capital budgeting evaluation techniques profitability index, net present value, and A. internal rate of return and net present value
internal rate of return will provide a consistent ranking of the projects. B. internal rate of return and payback
d. the net present value method will provide a ranking of the projects that is superior to the C. net present value and internal rate of return
ranking obtained using the internal rate of return method. Barfield D. net present value and payback CIA adapted

CMA EXAMINATION QUESTIONS Page 41 of 107


MANAGEMENT ADVISORY SERVICES CAPITAL BUDGETING

RISK ANALYSIS B. Use accelerated depreciation.


Risk C. Adjust the minimum desired rate of return.
Risk-free Rate D. Increase the estimates of the cash flows. CMA 1278 5-8
117
. The proper discount rate to use in calculating certainty equivalent net present value is the (E)
a. Risk-adjusted discount rate. d. Cost of equity capital. 23. To reflect greater uncertainty (greater risk) about a future cash inflow, an analyst could
b. Cost of capital. c. Risk-free rate. CMA 1292 4-19 a. increase the discount rate for the cash flow.
b. decrease the discounting period for the cash flow.
118
. The following data are related to the cash flows of a risky capital-budgeting alternative: c. increase the expected value of the future cash flow before it is discounted.
Col. 1 Col. 2 Col. 3 d. extend the acceptable length for the payback period. Barfield
Period Expected Cash Flows Certainty Equivalent Factors
1 1,000 .85 Risk Factor
121
2 1,000 .75 . For capital budgeting purposes, management would select a high hurdle rate of return for
3 1,000 .70 certain projects because management
The discount rates available for this analysis are: risk-free rate = 5%, cost of capital = 10%, a. Wants to use equity funding exclusively.
and risk-adjusted discount rate = 15%. How should these cash flows be discounted using the b. Believes too many proposals are being rejected.
certainty-equivalent method (CE) and the risk-adjusted discount rate method (RADR)? c. Believes bank loans are riskier than capital investments.
CE RADR d Wants to factor risk into its consideration of projects. CMA 1294 4-22
A. (Col.2xCol.3) at 10% (Col.2xCol.3) at 5%
B. (Col.2xCol.3) at 5% (Col.2xCol.3) at 10% 16. In a discounted cash flow analysis, which of the following would not be consistent with
C. (Col.2xCol.3) at 10% Col.2 at 15% adjusting a project's cash flows to account for higher-than-normal risk?
D. (Col.2xCol.3) at 5% Col.2 at 15% a. increasing the expected amount for cash outflows
b. increasing the discounting period for expected cash inflows
CIA 0586 IV-32
c. increasing the discount rate for cash outflows
119 d. decreasing the amount for expected cash inflows Barfield
. A firm has negotiated a contract with the government and has locked in the payment it will
receive in each of the future years from this project. However, the firm's costs for this project
Different Risk Levels
are uncertain. How should the certainty-equivalent (CE) approach be applied in this situation? 122
. When the risks of the individual components of a project’s cash flows are different, an
A. Discount cash inflows using cost of capital and CE values of cost using cost of capital.
acceptable procedure to evaluate these cash flows is to
B. Discount cash inflows using cost of capital and CE values of cost using risk-free rate.
a. Divide each cash flow by the payback period.
C. Determine net cash inflows using CE values of cost and discount using cost of capital.
b. Compute the net present value of each cash flow using the firm’s cost of capital.
D. Determine net cash inflows using CE values of cost and discount using risk-free rate.
c. Compare the internal rate of return from each cash flow to its risk. CMA 1295 4-6
CIA 0586 IV-34
d. Discount each cash flow using a discount rate that reflects the degree of risk.
Uncertainties
120 Risk-Adjusted Discount Rates
. Carco, Inc. wants to use discounted cash flow techniques when analyzing its capital 123
. Risk in a revenue-producing project can best be adjusted for by(E)
investment projects. The company is aware of the uncertainty involved in estimating future
a. Ignoring it.
cash flows. A simple method some companies employ to adjust for the uncertainty inherent in
b. Adjusting the discount rate upward for increasing risk.
their estimates is to
c. Adjusting the discount rate downward for increasing risk.
A. Prepare a direct analysis of the probability of outcomes.
CMA EXAMINATION QUESTIONS Page 42 of 107
MANAGEMENT ADVISORY SERVICES CAPITAL BUDGETING

d. Picking a risk factor equal to the average discount rate. e. Increase the cost of capital used to evaluate the project to reflect the higher risk of the
e. Reducing the NPV by 10 percent for risky projects. Brigham project. Brigham
126
Risk and Project Selection . Downingtown Industries has an overall (composite) WACC of 10 percent. This cost of capital
. If a company uses the same discount rate for evaluating all projects, which of the following reflects the cost of capital for a Downingtown project with average risk; however, there are
results is likely? (M) large differences among the projects. The company estimates that low-risk projects have a
a. Accepting poor, high-risk projects. d. Accepting no projects. cost of capital of 8 percent and high-risk projects have a cost of capital of 12 percent. The
b. Rejecting good, low-risk projects. e. Statements a and b are correct. company is considering the following projects:
c. Accepting only good, low risk projects. Brigham Project Expected Return Risk
A 15% High
. If a typical U. S. company uses the same discount rate to evaluate all projects, the firm will B 12 Average
most likely become (M) C 11 High
a. Riskier over time, and its value will decline. D 9 Low
b. Riskier over time, and its value will rise. E 6 Low
c. Less risky over time, and its value will rise. Which of the projects will the company select? (E)
d. Less risky over time, and its value will decline. a. A and B. d. A, B, C, and D.
e. There is no reason to expect its risk position or value to change over time as a result of its b. A, B, and C. e. A, B, C, D, and E.
use of a single discount rate. Brigham c. A, B, and D. Brigham
124
. A company estimates that an average-risk project has a WACC of 10 percent, a below- 127
. Mega Inc., a large conglomerate with operating divisions in many industries, uses risk-adjusted
average risk project has a WACC of 8 percent, and an above-average risk project has a discount rates in evaluating capital investment decisions. Consider the following statements
WACC of 12 percent. Which of the following independent projects should the company concerning Mega's use of risk-adjusted discount rates.
accept? (E) I. Mega may accept some investments with internal rates of return less than Mega's overall
a. Project A has average risk and an IRR = 9 percent. average cost of capital.
b. Project B has below-average risk and an IRR = 8.5 percent. II. Discount rates vary depending on the type of investment.
c. Project C has above-average risk and an IRR = 11 percent. III. Mega may reject some investments with internal rates of return greater than the cost of
d. All of the projects above should be accepted. capital.
e. None of the projects above should be accepted. Brigham IV. Discount rates may vary depending on the division.
125
Which of the above statements are correct?
. A firm is considering the purchase of an asset whose risk is greater than the current risk of the A. I and III only. C. II, III, and IV only.
firm, based on any method for assessing risk. In evaluating this asset, the decision maker B. II and IV only. D. I, II, III, and IV. CMA Samp Q4-5
should(E)
a. Increase the IRR of the asset to reflect the greater risk. 128
. Kemp Consolidated has two divisions of equal size: a computer division and a restaurant
b. Increase the NPV of the asset to reflect the greater risk. division. Stand-alone restaurant companies typically have a cost of capital of 8 percent, while
c. Reject the asset, since its acceptance would increase the risk of the firm. stand-alone computer companies typically have a 12 percent cost of capital. Kemp’s
d. Ignore the risk differential if the asset to be accepted would comprise only a small fraction restaurant division has the same risk as a typical restaurant company, and its computer
of the total assets of the firm. division has the same risk as a typical computer company. Consequently, Kemp estimates that
its composite corporate cost of capital is 10 percent. The company’s consultant has suggested
CMA EXAMINATION QUESTIONS Page 43 of 107
MANAGEMENT ADVISORY SERVICES CAPITAL BUDGETING

that they use an 8 percent hurdle rate for the restaurant division and a 12 percent hurdle rate b. 13 percent; the firm’s cost of capital should not be adjusted when evaluating outflow only
for the computer division. However, Kemp has chosen to ignore its consultant, and instead, projects.
chooses to assign a 10 percent cost of capital to all projects in both divisions. Which of the c. 16 percent; since A is more risky, its cash flows should be discounted at a higher rate,
following statements is most correct? (M) because this correctly penalizes the project for its high risk.
a. While Kemp’s decision to not risk adjust its cost of capital will lead it to accept more d. Somewhere between 10 percent and 16 percent, with the answer depending on the
projects in its computer division and fewer projects in its restaurant division, this should riskiness of the relevant inflows.
not affect the overall value of the company. e. Indeterminate, or, more accurately, irrelevant, because for such projects we would simply
b. Kemp’s decision to not risk adjust means that it is effectively subsidizing its restaurant select the process that meets the requirements with the lowest required investment.
division, which means that its restaurant division is likely to become a larger part of the Brigham
overall company over time.
c. Kemp’s decision to not risk adjust means that the company will accept too many projects Methods of Analyzing Risk
in the computer business and too few projects in the restaurant business. This will lead to 131
. Which of the following is not a method for analyzing risk in capital budgeting? (E)
a reduction in the overall value of the company. a. Sensitivity analysis.
d. Statements a and b are correct. Brigham b. Beta, or CAPM, analysis.
c. Monte Carlo simulation.
129
. The Barabas Company has an equal amount of low-risk projects, average-risk projects, and d. Scenario analysis.
high-risk projects. Barabas estimates that the overall company’s WACC is 12 percent. This is e. All of the statements above are methods of analyzing risk in capital budgeting. Brigham
also the correct cost of capital for the company’s average-risk projects. The company’s CFO
132
argues that, even though the company’s projects have different risks, the cost of capital for . Which of the following statements is correct? (M)
each project should be the same because the company obtains its capital from the same a. Sensitivity analysis is incomplete because it fails to consider the range of likely values of
sources. If the company follows the CFO’s advice, what is likely to happen over time? (M) key variables as reflected in their probability distributions.
a. The company will take on too many low-risk projects and reject too many high-risk b. In comparing two projects using sensitivity analysis, the one with the steeper lines would
projects. be considered less risky, because a small error in estimating a variable, such as unit
b. The company will take on too many high-risk projects and reject too many low-risk sales, would produce only a small error in the project’s NPV.
projects. c. The primary advantage of simulation analysis over scenario analysis is that scenario
c. Things will generally even out over time, and therefore, the risk of the firm should remain analysis requires a relatively powerful computer, coupled with an efficient financial
constant over time. planning software package, whereas simulation analysis can be done using a PC with a
d. Statements a and c are correct. Brigham spreadsheet program or even a calculator.
d. Sensitivity analysis is a risk analysis technique that considers both the sensitivity of NPV
130
. The Oneonta Chemical Company is evaluating two mutually exclusive pollution control to changes in key variables and the likely range of variable values. Brigham
systems. Since the company’s revenue stream will not be affected by the choice of control
systems, the projects are being evaluated by finding the PV of each set of costs. The firm’s Simulation and Sensitivity Analysis
133
required rate of return is 13 percent, and it adds or subtracts 3 percentage points to adjust for . A company is deciding whether to purchase an automated machine to manufacture one of its
project risk differences. System A is judged to be a high-risk project (it might end up costing products. Expected net cash flows from this decision depend on several factors, interactions
much more to operate than is expected). System A’s risk-adjusted cost of capital is(M) among those factors and the probabilities associated with different levels of those factors. The
a. 10 percent; this might seem illogical at first, but it correctly adjusts for risk where outflows, method that the company should use to evaluate the distribution of net cash flows from this
rather than inflows, are being discounted. decision and changes in net cash flows resulting from changes in levels of various factors is
a. Simulation and sensitivity analysis. c. Correlation analysis.
CMA EXAMINATION QUESTIONS Page 44 of 107
MANAGEMENT ADVISORY SERVICES CAPITAL BUDGETING

b. Linear programming. d. Differential analysis. CIA 1194 III-61 A. Sensitivity analysis. C. Cost behavior analysis. CMA 1286 5-4
B. Risk analysis. D. Return on investment analysis.
Simulation Analysis
134
. A firm is evaluating a large project. It desires to develop not only the best guess of the 62. Sensitivity analysis is
outcome of the project, but also a list (or distribution) of outcomes that might occur. This firm a. an appropriate response to uncertainty in cash flow projections.
would best achieve its objective by using b. useful in measuring the variance of the Fisher rate.
A. The net-present-value (NPV) approach for capital budgeting. c. typically conducted in the post investment audit.
B. The profitability-index approach for capital budgeting. d. useful to compare projects requiring vastly different levels of initial investment. Barfield
C. Simulation as applied to capital budgeting.
139
D. The internal-rate-of-return (IRR) approach for capital budgeting. CIA 0589 IV-51 . Sensitivity analysis is used in capital budgeting to
A. Estimate a project's internal rate of return.
135
. A statistical technique used to evaluate possible rates of return for a capital budgeting project B. Determine the amount that a variable can change without generating unacceptable
is results.
A. Regression analysis. C. Markov chain analysis. C. Simulate probabilistic customer reactions to a new product.
B. Simulation analysis. D. Gantt charting. CMA 0689 5-15 D. Identify the required market share to make a new product viable and produce acceptable
results. CMA 1293 4-16
136
. Which of the following statements is most correct? (E)
a. Sensitivity analysis is a good way to measure market risk because it explicitly takes into 24. In capital budgeting, sensitivity analysis is used
account the effects of diversification. a. To determine whether an investment is profitable.
b. One advantage of sensitivity analysis relative to scenario analysis is it explicitly takes into b. To see how a decision would be affected by changes in variables.
account the probability of certain effects occurring, whereas scenario analysis does not c. To test the relationship of the IRR and NPV.
take into account probabilities. d. To evaluate mutually exclusive investments. L&H
c. Simulation analysis is a computerized version of scenario analysis that uses continuous
probability distributions of the input variables. 17. Which of the following makes investments more desirable than they had been?
d. Statements a and b are correct. a. An increase in income tax rate.
e. All of the statements above are correct. Brigham b. An increase in interest rates.
c. An increase in the number of years over which assets must be depreciated.
Sensitivity Analysis d. None of the above. L&H
137
. Sensitivity analysis, if used with capital projects (M)
a. Is used extensively when cash flows are known with certainty. 28. Which statement could express the results of a sensitivity analysis of an investment decision?
b. Measures the change in the discounted cash flows when using the discounted payback a. The NPV of the project is $50,000.
method rather than the net present value method. b. A 5% decline in volume will make the project unprofitable.
c. Is a “what-if” technique that asks how a given outcome will change if the original estimates c. This project ranks third out of the five available.
of the capital budgeting model are changed. d. This project does not meet the cutoff rate of return. L&H
d. Is a technique used to rank capital expenditure requests. CMA 0695 4-2, RPCPA 0596
36. If X Co. expects to get a one-year bank loan to help cover the initial financing of capital project
138
. A manager wants to know the effect of a possible change in cash flows on the net present Q, the analysis of Q should (D)
value of a project. The technique used for this purpose is a. Offset the loan against any investment in inventory or receivables required by the project.
CMA EXAMINATION QUESTIONS Page 45 of 107
MANAGEMENT ADVISORY SERVICES CAPITAL BUDGETING

b. Show the loan as an increase in the investment. d. increase the project's internal rate of return. Barfield
c. Show the loan as a cash outflow in the second year of the project’s life.
d. Ignore the loan. L&H Depreciation and Savings on Cash Operating Costs
41. If depreciation of a new asset exceeds its savings in cash operating costs, which of the
54. A "what-if" technique that examines how a result will change if the original predicted data are following is true?
not achieved or if an underlying assumption changes is called (E) a. The project is usually unacceptable.
a. sensitivity analysis. c. internal rate-of-return analysis. Horngren b. The annual after-tax cash flow on the new asset will be greater than the savings in cash
b. net present value analysis. d. adjusted rate-of-return analysis. operating costs.
c. The project has a negative NPV.
Monte Carlo simulation d. All of the above. L&H
140
. Monte Carlo simulation(M)
a. Can be useful for estimating a project’s stand-alone risk. Sale of Old Plant Assets
b. Is capable of using probability distributions for variables as input data instead of a single 26. Because of idle capacity, a company is considering two assets for sale. They are identical in
numerical estimate for each variable. all respects except that asset A has a higher tax basis than asset B. Only one need be sold
c. Produces both an expected NPV (or IRR) and a measure of the riskiness of the NPV or now and the market price is the same for both assets. Which of the following is true?
IRR. a. The cash flow is greater from selling asset A.
d. Statements a and b are correct. b. The cash flow is greater from selling asset B.
e. All of the statements above are correct. Brigham c. The cash flow is the same no matter which one is sold.
d. It is not possible to determine how the cash flows from sale of the assets will differ. L & H
Net Investment
64. All other factors equal, which of the following would affect a project's internal rate of return, net Sale of Old Plant Asset at a Loss
present value, and payback period? (M) 51. When a profitable corporation sells an asset at a loss, the after-tax cash flow on the sale will
a. an increase in the discount rate c. an increase in the initial cost of the project (D)
b. a decrease in the life of the project d. all of the above Barfield a. exceed the pre-tax cash flow on the sale.
b. be less than the pre-tax cash flow on the sale.
Tax Effect on Transactions c. be the same as the pre-tax cash flow on the sale.
Change in Depreciation Rate d. increase the corporation's overall tax liability. Barfield
27. If the tax law were changed so that owners of apartment buildings had to depreciate them over
50 years instead of the current 31.5 years, 50. As the marginal tax rate goes up, the benefit from the depreciation tax shield
a. Rents would rise. a. decreases.
b. Rents would fall because annual depreciation charges would fall. b. increases.
c. Rents would stay about the same. c. stays the same. Barfield
d. More people would invest in apartment buildings. L&H d. can move up or down depending on whether the firm's cost of capital is high or low.

47. For a profitable company, an increase in the rate of depreciation on a specific project could Payback Period
a. increase the project's profitability index. In general
b. increase the project's payback period. 6. All other factors equal, a large number is preferred to a smaller number for all capital project
c. decrease the project's net present value. evaluation measures except (E)
CMA EXAMINATION QUESTIONS Page 46 of 107
MANAGEMENT ADVISORY SERVICES CAPITAL BUDGETING

a. net present value. c. internal rate of return. Accounting Rate of Return


b. payback period. d. profitability index. Barfield 25. Investment A has a book rate of return of 26%, investment B one of 18%. From this
information we can conclude
26. Risk can be controlled in capital budgeting situations by assuming a: a. That investment A has a higher NPV than B.
A. high accounting rate of return C. high net income b. That investment A has a higher IRR than B.
B. large net present value D. short payback period CIA adapted c. That investment A has a shorter payback period than B.
d. None of the above. L&H
3. In comparing two projects, the _______ is often used to evaluate the relative riskiness of the
projects. (D) Discount Rate
a. payback period c. internal rate of return Effect of Change in Acceptability of Projects
b. net present value d. discount rate Barfield 10. All other things being equal, as cost of capital increases
a. More capital projects will probably be acceptable.
Effect of Cash Flow b. Fewer capital projects will probably be acceptable.
28. An investment project that requires a present investment of $210,000 will have cash inflows of c. The number of capital projects that are acceptable will change, but the direction of the
"R" dollars each year for the next five years. The project will terminate in five years. Consider change is not determinable just by knowing the direction of the change in cost of capital.
the following statements (ignore income tax considerations): d. The company will probably want to borrow money rather than issue stock. L&H
I. If "R" is less than $42,000, the payback period exceeds the life of the project.
II. If "R" is greater than $42,000, the payback period exceeds the life of the project. Effect on Payback Period, Profitability Index & Accounting Rate of Return
III. If "R" equals $42,000, the payback period equals the life of the project. (M) *. Payback period (PP), profitability index (PI), and simple accounting rate of return (SARR) are
Which statement(s) is (are) true? some of the capital budgeting techniques. What is the effect of an increase in the cost of
a. Only I and II. c. Only II and III. capital on these techniques? (M)
b. Only I and III. d. I, II, and III. G & N 9e RPCPA 0594 a. b. c. d.
PP Increase No change No change Decrease
Relative Profitability PI Decrease Decrease Increase No change
24. Investment A has a payback period of 5.4 years, investment B one of 6.7 years. From this SARR Increase No change Decrease No change
information we can conclude
a. That investment A has a higher NPV than B. Present Value Factors
b. That investment A has a higher IRR than B. Effect of Discount Rate
c. That investment A’s book rate of return is higher than B’s. 30. As the discount rate increases
d. None of the above. L&H a. Present value factors increase.
b. Present value factors decrease.
9. If investment A has a payback period of three years and investment B has a payback period of c. Present value factors remain constant.
four years, then d. It is impossible to tell what happens to present value factors. L&H
a. A is more profitable than B.
b. A is less profitable than B. Effect of Length of an Annuity
c. A and B are equally profitable. Barfield 31. As the length of an annuity increases
d. the relative profitability of A and B cannot be determined from the information given. a. Present value factors increase.
b. Present value factors decrease.
CMA EXAMINATION QUESTIONS Page 47 of 107
MANAGEMENT ADVISORY SERVICES CAPITAL BUDGETING

c. Present value factors remain constant. a. will increase the present value of future cash flows.
d. It is impossible to tell what happen to present value factors. L&H b. will have no effect on net present value.
c. will reduce the present value of future cash flows.
96. All other things being equal, as the time period for receiving an annuity lengthens, d. is one method of compensating for reduced risk. G & N 9e
a. the related present value factors increase.
b. the related present value factors decrease. 22. Which of the following changes would not decrease the present value of the future
c. the related present value factors remain constant. Barfield depreciation deductions on a specific depreciable asset? (D)
d. it is impossible to tell what happens to present value factors from the information given. a. a decrease in the marginal tax rate
b. a decrease in the discount rate
Net Present Value c. a decrease in the rate of depreciation
Factors Affecting Net Present Value d. an increase in the life expectancy of the depreciable asset Barfield
54. The after-tax net present value of a project is affected by
a. tax-deductible cash flows. c. accounting accruals. 17. Suppose an investment has cash inflows of R dollars at the end of each year for two years.
b. non-tax-deductible cash flows. d. all of the above. Barfield The present value of these cash inflows using a 12% discount rate will be: (M)
a. greater than under a 10% discount rate.
17. Which of the following events will increase the NPV of an investment involving a new product? b. less than under a 10% discount rate.
a. An increase in the income tax rate. c. equal to that under a 10% discount rate. G & N 9e
b. An increase in the expected per-unit variable cost of the product. d. sometimes greater than under a 10% discount rate and sometimes less; it depends on R.
c. An increase in the expected annual unit volume of the product.
d. A decrease in the expected salvage value of equipment. L&H 24. A firm is evaluating a project that has a net present value of $0 when a discount rate of 8% is
used. A discount rate of 10% will result in
55. A project's after-tax net present value is increased by all of the following except a. a negative net present value
a. revenue accruals. c. depreciation deductions. b. a positive net present value
b. cash inflows. d. expense accruals. Barfield c. a net present value of $0
d. The question cannot be answered based upon the information provided. H&M
Change in Discount Rate
24. A change in the discount rate used to evaluate a specific project will affect the project's 27. A firm is evaluating a project that has a net present value of $0 when a discount rate of 8% is
a. life. c. net present value. used. A discount rate of 6% will result in
b. payback period. d. total cash flows. Barfield a. a negative net present value
b. a positive net present value
141
. The net present value (NPV) of a project has been calculated to be $215,000. Which one of c. a net present value of $0
the following changes in assumptions would decrease the NPV? (M) d. The question cannot be answered based upon the information provided. H&M
a. Decrease the estimated effective income tax rate.
b. Decrease the initial investment amount. 12. Which of the following would decrease the net present value of a project?
c. Extend the project life and associated cash inflows. A. A decrease in the income tax rate.
d. Increase the discount rate. CMA 1295 4-7 B. A decrease in the initial investment.
C. An increase in the useful life of the project.
16. An increase in the discount rate: (M) D. An increase in the discount rate. Pol Bobadilla
CMA EXAMINATION QUESTIONS Page 48 of 107
MANAGEMENT ADVISORY SERVICES CAPITAL BUDGETING

22. Two new products, X and Y, are alike in every way except that the sales of X will start low and
142
. Other things held constant, which of the following would increase the NPV of a project being rise throughout its life, while those of Y will be the same each year. Total volumes over their
considered? (E) five-year lives will be the same, as will selling prices, unit variable costs, cash fixed costs, and
a. A shift from MACRS to straight-line depreciation. investment. The NPV of product X
b. Making the initial investment in the first year rather than spreading it over the first three a. Will be less than that of product Y. c. Will be greater than that of product Y.
years. b. Will be the same as that of product Y. d. None of the above. L&H
c. A decrease in the discount rate associated with the project.
d. An increase in required net operating working capital. Income Tax Rate
e. All of the statements above will increase the project’s NPV. Brigham 6. Which of the following events is most likely to reduce the expected NPV of an investment?
a. The major competitor for the product to be manufactured with the machinery being
*. You have determined the profitability of a planned project by finding the present value of all the considered for purchase has been rated “unsatisfactory” by a consumer group.
cash flows from that project. Which of the following would cause the project to look less b. The interest rate on long-term debt declines.
appealing, that is, have a lower present value? (M) c. The income tax rate is raised by the Congress.
a. The discount rate increases. d. Congress approves the use of faster depreciation than was previously available. L&H
b. The cash flows are extended over a longer period of time.
c. The investment cost decreases without affecting the expected income and life of the Expected Returns
143
project. RPCPA 0595 . Stock C has a beta of 1.2, while Stock D has a beta of 1.6. Assume that the stock market is
d. The cash flows are accelerated and the project life is correspondingly shortened. efficient. Which of the following statements is most correct? (E)
a. The required rates of return of the two stocks should be the same.
*. Velasquez & Co. is considering an investment proposal for P10 million yielding a net present b. The expected rates of return of the two stocks should be the same.
value of P450,000. The project has a life of 7 years with salvage value of P200,000. The c. Each stock should have a required rate of return equal to zero.
company uses a discount rate of 12%. Which of the following would decrease the net present d. The NPV of each stock should equal its expected return.
value? (M) e. The NPV of each stock should equal zero. Brigham
a. Extend the project life and associated cash inflows.
b. Increase discount rate to 15%. NPV profiles
144
c. Decrease the initial investment amount to P9.0 million. . If the net present value profiles for two mutually exclusive capital projects are shaped as in the
d. Increase the salvage value. RPCPA 0597 graph below, which of the following statements is true?
$
Effect of Salvage Value
34. The salvage value of an old lathe is zero. If instead, the salvage value of the old lathe was
$20,000, what would be the impact on the net present value of the proposal to purchase a new
lathe? (M) NPV Profile
a. It would increase the net present value of the proposal. for Project 2 Cost of
b. It would decrease the net present value of the proposal. Capital %
c. It would not affect the net present value of the proposal.
d. Potentially it could increase or decrease the net present value of the new lathe. Barfield NPV Profile for Project 1
a. Project 2 has a higher internal rate of return that Project 1.
Timing of Cash Flow Stream b. Project 1 has a higher internal rate of return than Project 2.
CMA EXAMINATION QUESTIONS Page 49 of 107
MANAGEMENT ADVISORY SERVICES CAPITAL BUDGETING
146
c. Project 1 has a higher net present value than Project 2. . Cherry Books is considering two mutually exclusive projects. Project A has an internal rate of
d. Project 2 has a higher net present value than Project 1. return of 18 percent, while Project B has an internal rate of return of 30 percent. The two
projects have the same risk, the same cost of capital, and the timing of the cash flows is
43 Projects A and B have the same expected lives and initial cash outflows. However, one similar. Each has an up-front cost followed by a series of positive cash flows. One of the
project’s cashNPV
flows are larger in the early years, while the other project has larger cash flows projects, however, is much larger than the other. If the cost of capital is 16 percent, the two
in the later years. The two NPV profiles are given below: projects have the same net present value (NPV); otherwise, their NPVs are different. Which of
the following statements is most correct? (E)
a. If the cost of capital is 12 percent, Project B will have a higher NPV.
A
b. If the cost of capital is 17 percent, Project B will have a higher NPV.
c. Project B is larger than Project A.
d. Statements a and c are correct. Brigham
B

5. Projects L and S each have an initial cost of $10,000, followed by a series of positive cash
k inflows. Project L has total, undiscounted cash inflows of $16,000, while S has total
undiscounted inflows of $15,000. Further, at a discount rate of 10 percent, the two projects
have identical NPVs. Which project’s NPV will be more sensitive to changes in the discount
rate? (Hint: Projects with steeper NPV profiles are more sensitive to discount rate changes.)
(M)
Which of the following statements is most correct? (E) a. Project S.
a. Project A has the smaller cash flows in the later years. b. Project L.
b. Project A has the larger cash flows in the later years. c. Both projects are equally sensitive to changes in the discount rate since their NPVs are
c. We require information on the cost of capital in order to determine which project has larger equal at all costs of capital.
early cash flows. d. Neither project is sensitive to changes in the discount rate, since both have NPV profiles
d. The NPV profile graph is inconsistent with the statement made in the problem. which are horizontal.
e. None of the statements above is correct. Brigham e. The solution cannot be determined unless the timing of the cash flows is known. Brigham
145
. Projects A and B both have normal cash flows. In other words, there is an up-front cost 6. Two mutually exclusive projects each have a cost of $10,000. The total, undiscounted cash
followed over time by a series of positive cash flows. Both projects have the same risk and a flows from Project L are $15,000, while the undiscounted cash flows from Project S total
WACC equal to 10 percent. However, Project A has a higher internal rate of return than Project $13,000. Their NPV profiles cross at a discount rate of 10 percent. Which of the following
B. Assume that changes in the WACC have no effect on the projects’ cash flow levels. Which statements best describes this situation? (M)
of the following statements is most correct? (E) a. The NPV and IRR methods will select the same project if the cost of capital is greater
a. Project A must have a higher net present value than Project B. than 10 percent; for example, 18 percent.
b. If Project A has a positive NPV, Project B must also have a positive NPV. b. The NPV and IRR methods will select the same project if the cost of capital is less than 10
c. If Project A’s WACC falls, its internal rate of return will increase. percent; for example, 8 percent.
d. If Projects A and B have the same NPV at the current WACC, Project B would have a c. To determine if a ranking conflict will occur between the two projects the cost of capital is
higher NPV if the WACC of both projects was lower. Brigham needed as well as an additional piece of information.
d. Project L should be selected at any cost of capital, because it has a higher IRR.
e. Project S should be selected at any cost of capital, because it has a higher IRR. Brigham
CMA EXAMINATION QUESTIONS Page 50 of 107
MANAGEMENT ADVISORY SERVICES CAPITAL BUDGETING

However, if they have identical cash flow patterns, then the one with the steeper profile
147
. A company is comparing two mutually exclusive projects with normal cash flows. Project P probably has the lower initial cost. Brigham
has an IRR of 15 percent, while Project Q has an IRR of 20 percent. If the WACC is 10 e. If the two projects both have a single outlay at t = 0, followed by a series of positive cash
percent, the two projects have the same NPV. Which of the following statements is most inflows, and if their NPV profiles cross in the lower left quadrant, then one of the projects
correct? (M) should be accepted, and both would be accepted if they were not mutually exclusive.
a. If the WACC is 12 percent, both projects would have a positive NPV.
b. If the WACC is 12 percent, Project Q would have a higher NPV than Project P. Cross-Over Rate or Fisher Rate
149
c. If the WACC is 8 percent, Project Q would have a lower NPV than Project P. . Project A and Project B are mutually exclusive projects with equal risk. Project A has an
d. All of the statements above are correct. Brigham internal rate of return of 12 percent, while Project B has an internal rate of return of 15 percent.
The two projects have the same net present value when the cost of capital is 7 percent. (In
148
. Project C and Project D are two mutually exclusive projects with normal cash flows and the other words, the “crossover rate” is 7 percent.) Which of the following statements is most
same risk. If the WACC were equal to 10 percent, the two projects would have the same correct? (E)
positive NPV. However, if the WACC < 10%, Project C has a higher NPV, whereas if the a. If the cost of capital is 10 percent, each project will have a positive net present value.
WACC > 10%, Project D has a higher NPV. On the basis of this information, which of the b. If the cost of capital is 6 percent, Project B has a higher net present value than Project A.
following statements is most correct? (M) c. If the cost of capital is 13 percent, Project B has a higher net present value than Project A.
a. Project D has a higher IRR, regardless of the cost of capital. d. Statements a and b are correct.
b. If the WACC < 10%, Project C has a higher IRR. e. Statements a and c are correct. Brigham
c. If the WACC < 10%, Project D’s MIRR is less than its IRR.
150
d. Statements a and c are correct. Brigham . Sacramento Paper is considering two mutually exclusive projects. Project A has an internal
rate of return (IRR) of 12 percent, while Project B has an IRR of 14 percent. The two projects
7. Your assistant has just completed an analysis of two mutually exclusive projects. You must have the same risk, and when the cost of capital is 7 percent the projects have the same net
now take her report to a board of directors meeting and present the alternatives for the board’s present value (NPV). Assume each project has an initial cash outflow followed by a series of
consideration. To help you with your presentation, your assistant also constructed a graph with inflows. Given this information, which of the following statements is most correct? (E)
NPV profiles for the two projects. However, she forgot to label the profiles, so you do not know a. If the cost of capital is 13 percent, Project B’s NPV will be higher than Project A’s NPV.
which line applies to which project. Of the following statements regarding the profiles, which b. If the cost of capital is 9 percent, Project B’s NPV will be higher than Project A’s NPV.
one is most reasonable? (D) c. If the cost of capital is 9 percent, Project B’s modified internal rate of return (MIRR) will be
a. If the two projects have the same investment cost, and if their NPV profiles cross once in less than its IRR.
the upper right quadrant, at a discount rate of 40 percent, this suggests that a NPV versus d. Statements a and c are correct.
IRR conflict is not likely to exist. e. All of the statements above are correct. Brigham
b. If the two projects’ NPV profiles cross once, in the upper left quadrant, at a discount rate
of minus 10 percent, then there will probably not be a NPV versus IRR conflict, Profitability Index
irrespective of the relative sizes of the two projects, in any meaningful, practical sense *. What is the effect of changes in cash inflows, investment cost and cash outflows on profitability
(that is, a conflict which will affect the actual investment decision). (present value) index (PI) (M)
c. If one of the projects has a NPV profile which crosses the X-axis twice, hence the project a. PI will increase with an increase in cash inflows, a decrease in investment cost, or a
appears to have two IRRs, your assistant must have made a mistake. decrease in cash outflows.
d. Whenever a conflict between NPV and IRR exist, then, if the two projects have the same b. PI will increase with an increase in cash inflows, an increase in investment cost, or an
initial cost, the one with the steeper NPV profile probably has less rapid cash flows. increase in cash outflows.

CMA EXAMINATION QUESTIONS Page 51 of 107


MANAGEMENT ADVISORY SERVICES CAPITAL BUDGETING

c. PI will decrease with an increase in cash inflows, a decrease in investment cost, or a


decrease in cash outflows. 36. Which of the following factors increase NPV and IRR?
d. PI will decrease with an increase in cash outflows, an increase in investment cost, or an a. An upward revision in expected annual cash flows.
increase in cash inflows. RPCPA 0594 b. An upward revision of expected life.
c. An upward revision of the residual value of the long-lived assets being acquired for the
Internal Rate of Return project.
IRR of Zero d. All of the above. L&H
40. If the IRR on an investment is zero,
a. Its NPV is positive. NPV, IRR, and MIRR
b. Its annual cash flows equal its required investment. 17 Assume a project has normal cash flows (that is, the initial cash flow is negative, and all other
c. It is generally a wise investment. cash flows are positive). Which of the following statements is most correct? (M)
d. Its cash flows decrease over its life. L&H a. All else equal, a project’s IRR increases as the cost of capital declines.
b. All else equal, a project’s NPV increases as the cost of capital declines.
Timing of Cash Flow Stream c. All else equal, a project’s MIRR is unaffected by changes in the cost of capital.
151
. Everything else being equal, the internal rate of return (IRR) of an investment project will be d. Statements a and b are correct. Brigham
lower if (M)
a. The investment cost is lower. NPV & Market Value of Stocks
152
b. Cash inflows are received later in the life of the project. . If a firm identifies (or creates) an investment opportunity with a present value <List A> its
c. Cash inflows are larger. cost, the value of the firm and the price of its common shares will <List B>.
d. The project has a shorter payback period. CIA 0595 IV-37 CIA 1195 IV-44 A. B. C. D.
List A Greater than Greater than Equal to Equal to
Cash Outflow and Cash Inflow List B Increase Decrease Increase Decrease
43. If the total cash inflows associated with a project exceed the total cash outflows associated
with the project, the project's (D) 153
. The economic value of the firm will rise following an increase in
a. net present value is greater than zero. A. Net cash flow. C. Unsystematic risk.
b. internal rate of return is greater than zero. B. Systematic risk. D. The discount rate. CIA 0591 IV-47
c. profitability index is greater than 1.
d. payback period is acceptable. Barfield Comprehensive
33. Which of the following is true of an investment?
NPV and IRR a. The lower the cost of capital, the higher the NPV.
Change in Sales and Cost of Capital b. The lower the cost of capital, the higher the IRR.
*. You are engaged by the Baquis Co. to evaluate the introduction of a new product line with an c. The longer the project’s life, the shorted its payback period.
innovative packaging. You computed the net present value (NPV) and internal rate of return d. The higher the project’s NPV, the shorter its life. L&H
(IRR). If your client would reduce the estimate for its sales of the new product and increase
the projected cost of capital, what would be the impact of these revisions on NPV and IRR? 154
. Lieber Technologies is considering two potential projects, Project X and Project Y. In assessing
(M) RPCPA 1094 the projects’ risk, the company has estimated the beta of each project and has also conducted
a. NPV will increase, IRR will increase. c. NPV will increase, IRR will decrease. a simulation analysis. Their efforts have produced the following numbers:
b. NPV will decrease, IRR will increase. d. NPV will decrease, IRR will decrease. Project X Project Y
CMA EXAMINATION QUESTIONS Page 52 of 107
MANAGEMENT ADVISORY SERVICES CAPITAL BUDGETING

Expected NPV $350,000 $350,000


Standard deviation (NPV) $100,000 $150,000 3. If a firm uses the same company cost of capital for evaluating all projects, which of the
Estimated project beta 1.4 0.8 following is likely?
Estimated correlation of Cash flows are not highly Cash flows are highly A. Accepting poor low risk projects. C. Both A and B
project’s cash flows with the correlated with the cash correlated with the cash B. Rejecting good high risk projects D. Neither A nor B B&M
cash flows of the Company’s flows of the existing flows of the existing
existing projects. projects. projects. 4. Using the company cost of capital to evaluate a project is:
A. Always correct
Which of the following statements is most correct? (E) B. Always incorrect
a. Project X has a higher level of stand-alone risk relative to Project Y. C. Correct for projects that are about as risky as the average of the firm's other assets
b. Project X has a higher level of corporate risk relative to Project Y. D. None of the above B&M
c. Project X has a higher level of market risk relative to Project Y.
d. Statements b and c are correct. Brigham Project Analysis
1. Project analysis includes the following procedures:
PROJECT EVALUATION & SELECTION A. Sensitivity analysis C. Monte Carlo simulation
Categories of Projects B. Break-even analysis D. All of the above B&M
5. A firm might categorize its projects into:
A. Cost improvement projects D. Speculative ventures Break-even Analysis
B. Expansion projects (existing business) E. All of the above 16. Firms often calculate a project's break-even sales using book earnings. Generally, break-even
C. New products projects B&M sales based on NPV is:
A. Higher than the one calculated using book earnings
6. Which of the following types of projects have the highest risk? B. Lower than the one calculated using book earnings
A. Speculation ventures C. Expansion of existing business B&M C. Equal to the one calculated using book earnings
B. New products D. Cost improvement, (known technology) D. None of the above B&M

7. Which of the following type of projects has the lowest risk? Decision Tree
A. Speculation ventures C. Expansion of existing business 32. Which of the following statements applied to decision trees?
B. New products D. Cost improvement B&M A. They are simple to construct and analyze
B. They should include all possible future events and decisions
8. Which of the following type of projects has average risk? C. They help the financial manager to assess the value of options to abandon or expand the
A. Speculation ventures C. Expansion of existing business project
B. New products D. Cost improvement B&M D. All of the above B&M

Cost of Capital Consideration Simulation Models


2. If a firm uses the same company cost of capital for evaluating all projects, which of the 2. Simulation models are useful:
following is likely? A. To understand the project better C. To assess the project risk
A. Rejecting good low risk projects C. Both A and B B. To forecast expected cash flows D. All of the above B&M
B. Accepting poor high risk projects D. Neither A nor B B&M
CMA EXAMINATION QUESTIONS Page 53 of 107
MANAGEMENT ADVISORY SERVICES CAPITAL BUDGETING

23. Generally, the simulation models for projects are developed using a:
A. Computer C. Pair of dice 24. Monte Carlo simulation is likely to be most useful:
B. Roulette wheel D. Pack of cards B&M A. If small amounts of funds are at stake
B. If large amounts of funds are at stake
28. The hardest and most important part of a simulation is: C. If moderate amounts of funds are at stake
A. Simulating the cash flows D. Regardless of amount at stake B&M
B. Specifying the inter-dependencies
C. Specifying probabilities 25. Monte Carlo simulation is likely to be most useful:
D. Specifying the numbers on the roulette wheel B&M A. For simple problems
B. For problems of moderate complexity
20. Which of the following simulation outputs is likely to be most useful and easy to interpret? The C. For very complex problems
output shows the distribution(s) of the project: D. Regardless of the problem's complexity B&M
A. Earnings C. Cash flows
B. Internal rate of return D. Profits B&M 26. There are three steps involved in Monte Carlo simulations. One of the following is not one of
them:
21. The following statements about simulation models are true except: A. Modeling the project C. Modeling the strategy
A. Simulation models enable the financial manager to analyze risky projects without B. Specifying probabilities D. Simulating the cash flows B&M
estimating the approximate cost of capital
B. Simulation models are complex and expensive to develop 27. The following is not among the steps involved in the Monte Carlo method:
C. Simulation models are specific to the project and every project requires a new simulation A. Modeling the project
model B. Specifying the numbers on the roulette wheel
D. Simulation models usually ignore opportunities to expand or abandon the project B & M C. Specifying probabilities
D. Simulating the cash flows B&M
22. The following statements about simulation models are true except:
A. Simulation models enable the financial manager to analyze what would happen if the 29. The pharmaceutical companies have used the following method to analyze investments in
uncertainty about any of the variables were reduced R&D (research and development) of new drugs:
B. Simulation models take into account the interdependencies between different time periods A. Monte Carlo Simulation C. Sensitivity analysis
C. Simulation models are easy to understand and communicate B. Decision trees D. None of the above B&M
D. Simulation models enable the financial manager to visualize how outcomes may be
affected if the project is modified B&M 30. The pharmaceutical companies face three types of uncertainty. They are the following except:
A. Scientific and clinical C. Bureaucratic
Monte Carlo Simulation B. Production and distribution D. Market success B&M
19. Monte Carlo simulation is a tool for considering the:
A. Effect of changing one variable on the NPV of the project 31. According to the simulation model used by Merck and Company, the following types of
B. Effect of changing a limited number of plausible combination of variables on the NPV of variables are used in their model except:
the project A. Research and development
C. Effect of changing all possible combinations of variables on the NPV of the project B. Manufacturing variables
D. None of the above B&M C. Marketing variables
CMA EXAMINATION QUESTIONS Page 54 of 107
MANAGEMENT ADVISORY SERVICES CAPITAL BUDGETING

D. FDA (Food and Drug Administration) variables B&M

Basis for Decision


3. After the completion of project analysis, the final decision on the project would be from:
A. Sensitivity analysis C. Decision trees
B. Break-even analysis D. NPV B&M

CMA EXAMINATION QUESTIONS Page 55 of 107


MANAGEMENT ADVISORY SERVICES CAPITAL BUDGETING

CMA EXAMINATION QUESTIONS Page 56 of 107


MANAGEMENT ADVISORY SERVICES CAPITAL BUDGETING

stream, that is, one or more initial cash outflows (the investment) followed by a series of
Independent Projects cash inflows.
155
. Moynihan Motors has a cost of capital of 10.25 percent. The firm has two normal projects of b. If a conflict exists between the NPV and the IRR, the conflict can always be eliminated by
equal risk. Project A has an internal rate of return of 14 percent, while Project B has an dropping the IRR and replacing it with the MIRR.
internal rate of return of 12.25 percent. Which of the following statements is most correct? (E) c. There will be a meaningful (as opposed to irrelevant) conflict only if the projects’ NPV
a. Both projects have a positive net present value. profiles cross, and even then, only if the cost of capital is to the left of (or lower than) the
b. If the projects are mutually exclusive, the firm should always select Project A. discount rate at which the crossover occurs.
c. If the crossover rate (that is, the rate at which the Project’s NPV profiles intersect) is 8 d. All of the statements above are correct. Brigham
percent, Project A will have a higher net present value than Project B.
158
d. Statements a and b are correct. . Jurgensen Medical is considering two mutually exclusive projects with the following
e. Statements a and c are correct. Brigham characteristics:
 The two projects have the same risk and the same cost of capital.
156
. A company estimates that its weighted average cost of capital (WACC) is 10 percent. Which of  Both projects have normal cash flows. Specifically, each has an up-front cost followed by
the following independent projects should the company accept? (M) a series of positive cash flows.
a. Project A requires an up-front expenditure of $1,000,000 and generates a net present  If the cost of capital is 12 percent, Project X’s IRR is greater than its MIRR.
value of $3,200.  If the cost of capital is 12 percent, Project Y’s IRR is less than its MIRR.
b. Project B has a modified internal rate of return of 9.5 percent.  If the cost of capital is 10 percent, the two Project’s have the same NPV.
c. Project C requires an up-front expenditure of $1,000,000 and generates a positive internal Which of the following statements is most correct? (M)
rate of return of 9.7 percent. a. Project X’s IRR is greater than 12 percent.
d. Project D has an internal rate of return of 9.5 percent. Brigham b. Project Y’s IRR is less than 12 percent.
157
c. If the cost of capital is 8 percent, Project X has a lower NPV than Project Y.
. Project A has an internal rate of return of 18 percent, while Project B has an internal rate of d. All of the statements above are correct.
return of 16 percent. However, if the company’s cost of capital (WACC) is 12 percent, Project e. None of the statements above is correct. Brigham
B has a higher net present value. Which of the following statements is most correct? (D)
a. The crossover rate for the two projects is less than 12 percent. Ranking methods
b. Assuming the timing of the two projects is the same, Project A is probably of larger scale 159
. Which of the following statements is correct? (M)
than Project B. a. Because discounted payback takes account of the cost of capital, a project’s discounted
c. Assuming that the two projects have the same scale, Project A probably has a faster payback is normally shorter than its regular payback.
payback than Project B. b. The NPV and IRR methods use the same basic equation, but in the NPV method the
d. Statements a and b are correct. discount rate is specified and the equation is solved for NPV, while in the IRR method the
e. Statements b and c are correct. Brigham NPV is set equal to zero and the discount rate is found.
c. If the cost of capital is less than the crossover rate for two mutually exclusive projects’
Mutually Exclusive Projects NPV profiles, a NPV/IRR conflict will not occur.
11. Assume that you are comparing two mutually exclusive projects. Which of the following d. If you are choosing between two projects which have the same life, and if their NPV
statements is most correct? (M) profiles cross, then the smaller project will probably be the one with the steeper NPV
a. The NPV and IRR rules will always lead to the same decision unless one or both of the profile.
projects are “non-normal” in the sense of having only one change of sign in the cash flow e. If the cost of capital is relatively high, this will favor larger, longer-term projects over
smaller, shorter-term alternatives because it is good to earn high rates on larger amounts
CMA EXAMINATION QUESTIONS Page 57 of 107
MANAGEMENT ADVISORY SERVICES CAPITAL BUDGETING

over longer periods. Brigham a. There can never be a conflict between NPV and IRR decisions if the decision is related to
a normal, independent project, that is, NPV will never indicate acceptance if IRR indicates
160
. In comparing two mutually exclusive projects of equal size and equal life, which of the rejection.
following statements is most correct? (M) b. To find the MIRR, we first compound CFs at the regular IRR to find the TV, and then we
a. The project with the higher NPV may not always be the project with the higher IRR. discount the TV at the cost of capital to find the PV.
b. The project with the higher NPV may not always be the project with the higher MIRR. c. The NPV and IRR methods both assume that cash flows are reinvested at the cost of
c. The project with the higher IRR may not always be the project with the higher MIRR. capital. However, the MIRR method assumes reinvestment at the MIRR itself.
d. Statements a and c are correct. d. If you are choosing between two projects which have the same cost, and if their NPV
e. All of the statements above are correct. Brigham profiles cross, then the project with the higher IRR probably has more of its cash flows
coming in the later years. Brigham
Ranking conflicts e. A change in the cost of capital would normally change both a project’s NPV and its IRR.
26 Which of the following statements is most correct? (E)
162
a. The NPV method assumes that cash flows will be reinvested at the cost of capital while . In an operational audit of the finance department, the auditor observed that the department
the IRR method assumes reinvestment at the IRR. always used proper quantitative techniques based on sound economic assumptions to
b. The NPV method assumes that cash flows will be reinvested at the risk-free rate while the evaluate proposed alternative capital investments. However, management did not always
IRR method assumes reinvestment at the IRR. choose the investment option with the most favorable quantitative assessment. In fact,
c. The NPV method assumes that cash flows will be reinvested at the cost of capital while sometimes management opted for what appeared to be the third or fourth most favorable
the IRR method assumes reinvestment at the risk-free rate. investment. The chief financial officer indicated that management ultimately makes a
d. The NPV method does not consider the inflation premium. subjective decision as to which investment is best regardless of which investment option looks
e. The IRR method does not consider all relevant cash flows, and particularly cash flows best according to the quantitative analysis. Which of the following statements is most
beyond the payback period. Brigham accurate?
A. The approach is justifiable if the economic results of capital investments are highly
Comprehensive uncertain.
18 Which of the following statements is most correct? (D) B. The approach is an irrational, intuitive decision process.
a. When dealing with independent projects, discounted payback (using a payback C. The approach results in the organization not maximizing its profits.
requirement of 3 or less years), NPV, IRR, and modified IRR always lead to the same D. The approach is an example of the bounded rationality model of decision making whereby
accept/reject decisions for a given project. managers simplify problems.
b. When dealing with mutually exclusive projects, the NPV and modified IRR methods CIA 1195 II-4
always rank projects the same, but those rankings can conflict with rankings produced by
the discounted payback and the regular IRR methods. COMPREHENSIVE
c. Multiple rates of return are possible with the regular IRR method but not with the modified *. In capital budgeting decision, the following are relevant statements except: (E)
IRR method, and this fact is one reason given by the textbook for favoring MIRR (or a. Since resources are scarce, all capital expenditures must be ranked according to priority.
modified IRR) over IRR. b. The company must be able to define what falls under this category, whether they are for
d. Statements a and c are correct. expansion, for replacements, or for improvements in operations.
e. None of the statements above is correct. Brigham c. Capital investments are short-term commitments of resources, and they are decided in the
same process as operating expenses/
Decision-Making d. A careful analysis of the economic and non-economic reasons or justifications for these
161
. Which of the following statements is correct? (D) investments must be made to arrive at the appropriate decision. RPCPA 0593
CMA EXAMINATION QUESTIONS Page 58 of 107
MANAGEMENT ADVISORY SERVICES CAPITAL BUDGETING

a. The IRR method is appealing to some managers because it produces a rate of return
*. Which of the following statements is correct? (E) upon which to base decisions rather than a dollar amount like the NPV method.
a. One key shortcoming of discounted cash flow method is that they ignore the recovery of b. The discounted payback method solves all the problems associated with the payback
original investment. method.
b. Although a cash outlay for noncurrent asset such as a machine would be considered in a c. For independent projects, the decision to accept or reject will always be the same using
capital budgeting analysis, a cash outlay for working capital item such as inventory would either the IRR method or the NPV method.
not be considered. d. Statements a and c are correct.
c. To be acceptable, a project’s time adjusted rate of return cannot be less than the e. All of the statements above are correct. Brigham
company’s cost of capital.
165
d. If the net present value of an investment is zero, then the project should be rejected since . Which of the following statements is most correct? (M)
it is not providing any return on investment. RPCPA 1095 a. One of the disadvantages of choosing between mutually exclusive projects on the basis of
the discounted payback method is that you might choose the project with the faster
*. Which of the following statements is False? (M) payback period but with the lower total return.
a. The net present value (NPV) of a project with cash flows that come in relatively slowly is b. Multiple IRRs can occur in cases when project cash flows are normal, but they are more
more sensitive to changes in the discount rate than is the NPV of a project with cash flows common in cases where project cash flows are nonnormal.
that come in rapidly. c. When choosing between mutually exclusive projects, managers should accept all projects
b. Other things held constant, a decrease in the cost of capital (discount rate) will cause an with IRRs greater than the weighted average cost of capital.
increase in a project’s internal rate of return. d. Statements a and b are correct.
c. The IRR method can be used in place of the NPV method for all independent projects e. All of the statements above are correct. Brigham
because the two methods then result in identical decisions.
166
d. The NPV method is preferred over the IRR method because the NPV method’s . Normal projects C and D are mutually exclusive. Project C has a higher net present value if
reinvestment rate assumption is the correct assumption. RPCPA 0595 the WACC is less than 12 percent, whereas Project D has a higher net present value if the
WACC exceeds 12 percent. Which of the following statements is most correct? (M)
163
. Which of the following is most correct? (M) a. Project D has a higher internal rate of return.
a. The NPV and IRR rules will always lead to the same decision in choosing between b. Project D is probably larger in scale than Project C.
mutually exclusive projects, unless one or both of the projects are “non-normal” in the c. Project C probably has a faster payback.
sense of having only one change of sign in the cash flow stream. d. Statements a and c are correct.
b. The Modified Internal Rate of Return (MIRR) compounds cash outflows at the cost of e. All of the statements above are correct. Brigham
capital.
c. Conflicts between NPV and IRR rules arise in choosing between two mutually exclusive MODIFIED ACCELERATED COST RECOVERY SYSTEM (MACRS)
projects (that each have normal cash flows) when the cost of capital exceeds the MACRS accelerated depreciation rates should be given for many of these problems. These rates
crossover point (that is, the point at which the NPV profiles cross). are provided in the text in Appendix 12A.
d. The discounted payback method overcomes the problems that the payback method has
with cash flows occurring after the payback period. ACRS
e. None of the statements above is correct. Brigham 15. The system for recovering the cost of capital expenditures through federal income tax
deductions that was required for tangible, depreciable property placed in service after 1980 is
164
. Which of the following statements is most correct? (M) known as:
A. MACRS C. ACRS
CMA EXAMINATION QUESTIONS Page 59 of 107
MANAGEMENT ADVISORY SERVICES CAPITAL BUDGETING

B. 200% declining balance D. 150% declining balance Carter & Usry 18. An example of 7-year property under MACRS is:
A. automobiles C. light trucks
MACRS B. most manufacturing machinery D. small aircraft Carter & Usry
16. Under the Tax Reform Act of 1986, the system that increased the number of property classes
and lengthened the recovery periods of most kinds of depreciable property is known as: 19. An example of 27.5-year property under MACRS is:
A. MACRS C. ACRS A. residential rental property C. nonresidential buildings
B. 200% declining balance D. 150% declining balance Carter & Usry B. commercial aircraft D. railroad cars Carter & Usry

21. When computing depreciation deductions under the MACRS system, taxpayers must: (M) 20. With respect to income taxes, the principal advantage of MACRS over straight-line
a. use the half-year convention under which taxpayers are allowed to take only a half year's depreciation is that
depreciation in the first year of an asset's life. a. Total taxes will be lower under MACRS.
b. use the half-year convention under which taxpayers are allowed to take only a half year's b. Taxes will be constant from year to year under MACRS.
depreciation in the last year of an asset's life. c. Taxes will be lower in the earlier years under MACRS.
c. use the half-year convention under which taxpayers are allowed to take only a half year's d. Taxes will decline in future years under MACRS. L&H
depreciation in the first and last years of an asset's life.
d. calculate depreciation for partial periods using the exact number of days if the asset is 4. Companies using MACRS for tax purposes and straight-line depreciation for financial reporting
acquired at some time other than the beginning or end of the fiscal year. G & N 9e purposes usually find that the relationship between the tax basis and book value of their assets
is
20. Under MACRS, the depreciation on tangible personal property is computed as if the property a. The tax basis is lower than book value.
were placed into service at the: b. The tax basis is higher than book value.
A. beginning of the year C. midpoint of the year c. The tax basis is the same as book value.
B. end of the year D. midpoint of the month Carter & Usry d. None of the above. L&H

21. Under MACRS, the depreciation on real property is computed as if the property were placed 5. A company that wants to use MACRS for tax purposes must
into service at the: a. Request permission from the IRS.
A. beginning of the year C. midpoint of the year b. Acquire new assets to or near the middle of the year.
B. end of the year D. midpoint of the month Carter & Usry c. Ignore salvage value in calculating depreciation.
d. Do none of the above. L&H
16. Classifying an asset in a MACRS life category is based on
a. Useful life estimated by the company. 38. A company evaluates a project using straight-line depreciation over its 10-year estimated
b. Asset depreciation range (ADR) guidelines. useful life and then reevaluates it using a 7-year MACRS class life. The second analysis will
c. The cost of the asset. show
d. Any of the above factors. L&H a. A lower IRR for the project.
b. The same NPV and IRR for the project.
17. An example of 5-year property under MACRS is: c. A higher NPV for the project.
A. most manufacturing machinery C. commercial aircraft d. Lower total cash flows over the 10 years. L&H
B. railroad cars D. light trucks Carter & Usry

CMA EXAMINATION QUESTIONS Page 60 of 107


MANAGEMENT ADVISORY SERVICES CAPITAL BUDGETING

13. If a company uses a five-year MACRS period to depreciate assets instead of a 10-year life B. MACRS producing less total depreciation than straight line.
with straight-line depreciation, C. Equal total tax payments, after discounting for the time value of money.
a. The NPV of the investment is higher. D. MACRS producing more total depreciation than straight line.
b. The IRR of the investment is lower.
c. There is no difference in either NPV or IRR. MACRS vs. Optional Straight-line Method
d. Total cash flows over the useful life would be lower. L&H 18. In a capital budgeting decision, the use of MACRS tables as compared to the optional straight-
line method will result in: (M)
167
. Flex Corporation is studying a capital acquisition proposal in which newly acquired assets will a. equal total depreciation for both methods.
be depreciated using the straight-line method. Which one of the following statements about the b. more total depreciation for the MACRS tables method.
proposal would be incorrect if a switch is made to the Modified Accelerated Cost Recovery c. more total depreciation for the optional straight-line method. CMA adapted
System (MACRS)? CMA 0693 4-29 d. less depreciation for the MACRS tables method in the early years of asset life.
A. The net present value will increase. C. The payback period will be shortened.
B. The internal rate of return will increase. D. The profitability index will decrease. 19. The use of the MACRS tables instead of the optional straight-line method of depreciation has
the effect of: (M)
. Which of the following statement completions is incorrect? For a profitable firm, when MACRS
168
a. raising the hurdle rate necessary to justify the project.
accelerated depreciation is compared to straight-line depreciation, MACRS accelerated b. decreasing the net present value of the project.
allowances produce (M) c. increasing the present value of the depreciation tax shield.
a. Higher depreciation charges in the early years of an asset’s life. d. increasing the cash outflows at the beginning of the project. CMA adapted
b. Larger cash flows in the earlier years of an asset’s life.
c. Larger total undiscounted profits from the project over the project’s life. 20. Which of the following is correct? (M)
d. Smaller accounting profits in the early years, assuming the company uses the same a. Use of the MACRS tables requires that salvage value be deducted in computing
depreciation method for tax and book purposes. depreciation deductions.
e. None of the statements above. (All of the statements above are correct.) Brigham b. Use of the optional straight-line method requires that salvage value not be considered in
computing depreciation deductions.
Questions 74 and 75 are based on the following information. CMA 0694 4-13 & 14 c. The use of both MACRS tables and the optional straight-line method requires that salvage
The tax impact of equipment depreciation affects capital budgeting decisions. Currently, the value be deducted in computing depreciation deductions.
Modified Accelerated Cost Recovery System (MACRS) is used as the depreciation method for d. None of the above are true. G & N 9e
most assets for tax purposes.
Taxes on gain on sale
169 171
. The MACRS method of depreciation for assets with 3, 5, 7, and 10-year recovery periods is . St. John’s Paper is considering purchasing equipment today that has a depreciable cost of $1
most similar to which one of the following depreciation methods used for financial reporting million. The equipment will be depreciated on a MACRS 5-year basis, which implies the
purposes? following depreciation schedule:
A. Straight-line. C. Sum-of-the-years'-digits. Year MACRS Depreciation Rate
B. Units-of-production. D. 200% declining-balance. 1 0.20
2 0.32
170
. When employing the MACRS method of depreciation in a capital budgeting decision, the use 3 0.19
of MACRS as compared with the straight-line method of depreciation will result in 4 0.12
A. Equal total depreciation for both methods. 5 0.11
CMA EXAMINATION QUESTIONS Page 61 of 107
MANAGEMENT ADVISORY SERVICES CAPITAL BUDGETING

6 0.06 years. The Unlimited’s marginal tax rate is 40 percent. What risk-adjusted discount rate will
Assume that the company sells the equipment after three years for $400,000 and the equate the NPV of Project B to that of Project A? (M)
company’s tax rate is 40 percent. What would be the tax consequences resulting from the sale a. 15% d. 20%
of the equipment? (E) b. 16% e. 12%
a. There are no tax consequences. c. 18% Brigham
b. The company would have to pay $44,000 in taxes.
174
c. The company would have to pay $160,000 in taxes. . California Mining is evaluating the introduction of a new ore production process. Two alter-
d. The company would receive a tax credit of $124,000. natives are available. Production Process A has an initial cost of $25,000, a 4-year life, and a
e. The company would receive a tax credit of $48,000. Brigham $5,000 net salvage value, and the use of Process A will increase net cash flow by $13,000 per
year for each of the 4 years that the equipment is in use. Production Process B also requires
Risk-adjusted discount rate an initial investment of $25,000, will also last 4 years, and its expected net salvage value is
172
. Dick Boe Enterprises, an all-equity firm, has a corporate beta coefficient of 1.5. The financial zero, but Process B will increase net cash flow by $15,247 per year. Management believes
manager is evaluating a project with an IRR of 21 percent, before any risk adjustment. The that a risk-adjusted discount rate of 12 percent should be used for Process A. If California
risk-free rate is 10 percent, and the required rate of return on the market is 16 percent. The Mining is to be indifferent between the two processes, what risk-adjusted discount rate must
project being evaluated is riskier than Boe’s average project, in terms of both beta risk and be used to evaluate B? (D)
total risk. Which of the following statements is most correct? (E) a. 8% d. 14%
a. The project should be accepted since its IRR (before risk adjustment) is greater than its b. 10% e. 16%
required return. c. 12% Brigham
b. The project should be rejected since its IRR (before risk adjustment) is less than its re-
quired return. New project NPV
175
c. The accept/reject decision depends on the risk-adjustment policy of the firm. If the firm’s . Given the following information, calculate the NPV of a proposed project: Cost = $4,000;
policy were to reduce a riskier-than-average project’s IRR by 1 percentage point, then the estimated life = 3 years; initial decrease in accounts receivable = $1,000, which must be
project should be accepted. restored at the end of the project’s life; estimated salvage value = $1,000; earnings before
d. Riskier-than-average projects should have their IRRs increased to reflect their added taxes and depreciation = $2,000 per year; method of depreciation = MACRS; tax rate = 40
riskiness. Clearly, this would make the project acceptable regardless of the amount of the percent; and cost of capital = 18 percent. (MACRS table required) (M)
adjustment. a. $1,137 d. $ 804
e. Projects should be evaluated on the basis of their total risk alone. Thus, there is b. -$ 151 e. $ 544
insufficient information in the problem to make an accept/reject decision. Brigham c. $ 137 Brigham
176
Risk-adjusted discount rate . Mars Inc. is considering the purchase of a new machine that will reduce manufacturing costs
173
. The Unlimited, a national retailing chain, is considering an investment in one of two mutually by $5,000 annually. Mars will use the MACRS accelerated method to depreciate the machine,
exclusive projects. The discount rate used for Project A is 12 percent. Further, Project A costs and it expects to sell the machine at the end of its 5-year operating life for $10,000. The firm
$15,000, and it would be depreciated using MACRS. It is expected to have an after-tax expects to be able to reduce net operating working capital by $15,000 when the machine is
salvage value of $5,000 at the end of 6 years and to produce after-tax cash flows (including installed, but required net operating working capital will return to the original level when the
depreciation) of $4,000 for each of the 6 years. Project B costs $14,815 and would also be machine is sold after 5 years. Mars’ marginal tax rate is 40 percent, and it uses a 12 percent
depreciated using MACRS. B is expected to have a zero salvage value at the end of its 6-year cost of capital to evaluate projects of this nature. If the machine costs $60,000, what is the
life and to produce after-tax cash flows (including depreciation) of $5,100 each year for 6 project’s NPV? (MACRS table required) (M)
a. -$15,394 d. -$21,493
CMA EXAMINATION QUESTIONS Page 62 of 107
MANAGEMENT ADVISORY SERVICES CAPITAL BUDGETING

b. -$14,093 e. -$46,901  This equipment will be depreciated according to the following depreciation schedule:
c. -$58,512 Brigham Year MACRS Depreciation Rate
1 0.33
177
. Stanton Inc. is considering the purchase of a new machine that will reduce manufacturing 2 0.45
costs by $5,000 annually and increase earnings before depreciation and taxes by $6,000 3 0.15
annually. Stanton will use the MACRS method to depreciate the machine, and it expects to sell 4 0.07
the machine at the end of its 5-year operating life for $10,000 before taxes. Stanton’s marginal  The equipment will have no salvage value after four years.
tax rate is 40 percent, and it uses a 9 percent cost of capital to evaluate projects of this type. If  If MacDonald goes ahead with the new business inventories will rise by $500,000 at t = 0,
the machine’s cost is $40,000, what is the project’s NPV? (MACRS table required) (M) and its accounts payable will rise by $200,000 at t = 0. This increase in net operating
a. $1,014 d. $ 817 working capital will be recovered at t = 4.
b. $2,292 e. $5,040  The new business is expected to have an economic life of four years. The business is
c. $7,550 Brigham expected to generate sales of $3 million at t = 1, $4 million at t = 2, $5 million at t = 3, and
$2 million at t = 4. Each year, operating costs excluding depreciation are expected to be
178
. Maple Media is considering a proposal to enter a new line of business. In reviewing the 75 percent of sales.
proposal, the company’s CFO is considering the following facts:  The company’s tax rate is 40 percent.
 The new business will require the company to purchase additional fixed assets that  The company’s weighted average cost of capital is 10 percent.
will cost $600,000 at t = 0. For tax and accounting purposes, these costs will be  The company is very profitable, so any accounting losses on this project can be used to
depreciated on a straight-line basis over three years. (Annual depreciation will be reduce the company’s overall tax burden.
$200,000 per year at What is the expected net present value (NPV) of the new business? (M)
t = 1, 2, and 3.) a. $ 740,298 d. -$1,961,833
 At the end of three years, the company will get out of the business and will sell the b. -$1,756,929 e. –$5,919,974
fixed assets at a salvage value of $100,000. c. -$1,833,724 Brigham
 The project will require a $50,000 increase in net operating working capital at t = 0,
180
which will be recovered at t = 3. . Rio Grande Bookstores is considering a major expansion of its business. The details of the
 The company’s marginal tax rate is 35 percent. proposed expansion project are summarized below:
 The new business is expected to generate $2 million in sales each year (at t = 1, 2,  The company will have to purchase $500,000 in equipment at t = 0. This is the
and 3). The operating costs excluding deprecia-tion are expected to be $1.4 million per depreciable cost.
year.  The project has an economic life of four years.
 The project’s cost of capital is 12 percent.  The cost can be depreciated on a MACRS 3-year basis, which implies the following
What is the project’s net present value (NPV)? (M) depreciation schedule:
a. $536,697 d. $ 56,331 Year MACRS Depreciation Rate
b. $ 86,885 e. $561,609 1 0.33
c. $ 81,243 Brigham 2 0.45
3 0.15
179
. MacDonald Publishing is considering entering a new line of business. In analyzing the 4 0.07
potential business, their financial staff has accumulated the following information:  At t = 0, the project requires that inventories increase by $50,000 and accounts payable
 The new business will require a capital expenditure of $5 million at t = 0. This expenditure increase by $10,000. The change in net operating working capital is expected to be fully
will be used to purchase new equipment. recovered at t = 4.
CMA EXAMINATION QUESTIONS Page 63 of 107
MANAGEMENT ADVISORY SERVICES CAPITAL BUDGETING

 The project’s salvage value at the end of four years is expected to be $0. At the end of four years the company expects to be able to sell the equipment for an after-tax
 The company forecasts that the project will generate $800,000 in sales the first two years salvage value of $10,000. The company is in the 40 percent tax bracket. The company has an
(t = 1 and 2) and $500,000 in sales during the last two years (t = 3 and 4). after-tax cost of capital of 11 percent. Because there is more uncertainty about the salvage
 Each year the project’s operating costs excluding depreciation are expected to be 60 value, the company has chosen to discount the salvage value at 12 percent. What is the net
percent of sales revenue. present value (NPV) of purchasing the equipment? (D)
 The company’s tax rate is 40 percent. a. $ 9,140.78 d. $22,853.90
 The project’s cost of capital is 10 percent. b. $16,498.72 e. $28.982.64
What is the net present value (NPV) of the proposed project? (M) c. $20,564.23 Brigham
a. $159,145 d. $150,776
183
b. $134,288 e. -$257,060 . Lugar Industries is considering an investment in a proposed project that requires an initial
c. $162,817 Brigham expenditure of $100,000 at t = 0. This expenditure can be depreciated at the following annual
rates:
181
. Foxglove Corp. is faced with an investment project. The following information is associated Year MACRS Depreciation Rate
with this project: 1 0.20
Year Net Income* MACRS Depreciation Rate 2 0.32
1 $50,000 0.33 3 0.19
2 60,000 0.45 4 0.12
3 70,000 0.15 5 0.11
4 60,000 0.07 6 0.06
*Assume no interest expenses and a zero tax rate. The project has an economic life of six years. The project’s revenues are forecasted to be
The project involves an initial investment of $100,000 in equipment that falls in the 3-year $90,000 a year. The project’s operating costs (not including depreciation) are forecasted to be
MACRS class and has an estimated salvage value of $15,000. In addition, the company $50,000 a year. After six years, the project’s estimated salvage value is $10,000. The
expects an initial increase in net operating working capital of $5,000 that will be recovered in company’s WACC is 10 percent, and its corporate tax rate is 40 percent. What is the project’s
Year 4. The cost of capital for the project is 12 percent. What is the project’s net present net present value (NPV)? (D)
value? (Round your final answer to the nearest whole dollar.) (D) a. $31,684 d. $38,840
a. $153,840 d. $168,604 b. $33,843 e. $45,453
b. $159,071 e. $182,344 c. $34,667 Brigham
c. $162,409 Brigham 184
. Mills Mining is considering an expansion project. The proposed project has the following
182
. Pierce Products is deciding whether it makes sense to purchase a new piece of equipment. features:
The equipment costs $100,000 (payable at t = 0). The equipment will provide before-tax cash  The project has an initial cost of $500,000. This is also the amount that can be
inflows of $45,000 a year at the end of each of the next four years (t = 1, 2, 3, and 4). The depreciated using the following depreciation schedule:
equipment can be depreciated according to the following schedule: Year MACRS Depreciation Rate
Year MACRS Depreciation Rate 1 0.33
1 0.33 2 0.45
2 0.45 3 0.15
3 0.15 4 0.07
4 0.07
CMA EXAMINATION QUESTIONS Page 64 of 107
MANAGEMENT ADVISORY SERVICES CAPITAL BUDGETING
187
 If the project is undertaken, at t = 0 the company will need to increase its inventories . Real Time Systems Inc. is considering the development of one of two mutually exclusive new
by $50,000, and its accounts payable will rise by $10,000. This net operating working computer models. Each will require a net investment of $5,000. The cash flows for each
capital will be recovered at the end of the project’s life (t = 4). project are shown below:
 If the project is undertaken, the company will realize an additional $600,000 in sales Year Project A Project B
over each of the next four years (t = 1, 2, 3, and 4). The company’s operating cost (not 1 $2,000 $3,000
including depreciation) will equal $400,000 a year. 2 2,500 2,600
 The company’s tax rate is 40 percent. 3 2,250 2,900
 At t = 4, the project’s economic life is complete, but it will have a salvage value of Model B, which will use a new type of laser disk drive, is considered a high-risk project, while
$50,000. Model A is an average-risk project. Real Time adds 2 percentage points to arrive at a risk-
 The project’s WACC = 10 percent. adjusted cost of capital when evaluating high-risk projects. The cost of capital used for
What is the project’s net present value (NPV)? (D) average-risk projects is 12 percent. Which of the following statements regarding the NPVs for
a. $11,122.87 d. $68,336.86 Models A and B is most correct? (M)
b. $50,330.14 e. $80,035.52 a. NPVA = $380; NPVB = $1,815 c. NPVA = $380; NPVB = $1,590
c. $54,676.59 Brigham b. NPVA = $197; NPVB = $1,590 d. NPVA = $5,380; NPVB = $6,590 Brigham

Risk-adjusted NPV NPV and risk-adjusted discount rate


185
. Virus Stopper Inc., a supplier of computer safeguard systems, uses a cost of capital of 12 188
. Garcia Paper is deciding whether to build a new plant. The proposed project would have an
percent to evaluate average-risk projects, and it adds or subtracts 2 percentage points to up-front cost (at t = 0) of $30 million. The project’s cost can be depreciated on a straight-line
evaluate projects of more or less risk. Currently, two mutually exclusive projects are under basis over three years. Consequently, the depreciation expense will be $10 million in each of
consideration. Both have a cost of $200,000 and will last 4 years. Project A, a riskier-than- the first three years, t = 1, 2, and 3. Even though the project is depreciated over three years,
average project, will produce annual end-of-year cash flows of $71,104. Project B, a less-than- the project has an economic life of five years.
average-risk project, will produce cash flows of $146,411 at the end of Years 3 and 4 only. The project is expected to increase the company’s sales by $20 million. Sales will remain at
Virus Stopper should accept(M) this higher level for each year of the project (t = 1, 2, 3, 4, and 5). The operating costs, not
a. B with a NPV of $10,001. including depreciation, equal 60 percent of the increase in annual sales. The project’s interest
b. Both A and B because both have NPVs greater than zero. expense is $5 million per year and the company’s tax rate is 40 percent. The company is very
c. B with a NPV of $8,042. profitable, so any accounting losses on this project can be used to reduce the company’s
d. A with a NPV of $7,177. overall tax burden. The project does not require any additions to net operating working capital.
e. A with a NPV of $15,968. Brigham The company estimates that the project’s after-tax salvage value at t = 5 will be $1.2 million.
The project is of average risk, and, therefore, the CFO has decided to discount the operating
186
. An all-equity firm is analyzing a potential project that will require an initial, after-tax cash outlay cash flows at the company’s overall WACC of 10 percent. However, the salvage value is more
of $50,000 and after-tax cash inflows of $6,000 per year for 10 years. In addition, this project uncertain, so the CFO has decided to discount it at 12 percent. What is the net present value
will have an after-tax salvage value of $10,000 at the end of Year 10. If the risk-free rate is 6 (NPV) of the proposed project? (D)
percent, the return on an average stock is 10 percent, and the beta of this project is 1.50, what a. $11.86 million d. -$12.55 million
is the project’s NPV? (M) b. $14.39 million e. -$ 1.18 million
a. $13,210 d. -$ 6,158 c. -$26.04 million Brigham
b. $ 4,905 e. -$12,879
c. $ 7,121 Brigham Discounting risky outflows

CMA EXAMINATION QUESTIONS Page 65 of 107


MANAGEMENT ADVISORY SERVICES CAPITAL BUDGETING
189
. Alabama Pulp Company (APC) can control its environmental pollution using either “Project Old a. Projects: A, B, C, D, E d. Projects: A, D
Tech” or “Project New Tech.” Both will do the job, but the actual costs involved with Project b. Projects: B, C, D, E e. Projects: B, C, D
New Tech, which uses unproved, new state-of-the-art technology, could be much higher than c. Projects: B, D Brigham
the expected cost levels. The cash outflows associated with Project Old Tech, which uses
standard proven technology, are less risky. (They are about as uncertain as the cash flows Scenario analysis
associated with an average project.) APC’s cost of capital for average-risk projects is normally 191
. Klott Company encounters significant uncertainty with its sales volume and price in its primary
set at 12 percent, and the company adds 3 percent for high-risk projects but subtracts 3 product. The firm uses scenario analysis in order to determine an expected NPV, which it then
percent for low-risk projects. The two projects in question meet the criteria for high and uses in its budget. The base-case, best-case, and worst-case scenarios and probabilities are
average risk, but the financial manager is concerned about applying the normal rule to such provided in the table below. What is Klott’s expected NPV, standard deviation of NPV, and
cost-only projects. You must decide which project to recommend, and you should recommend coefficient of variation of NPV? (M)
the one with the lower PV of costs. What is the PV of costs of the better project? (M) Probability of Unit Sales Sales Price NPV
Cash Outflows Outcome Volume (In Thousands)
Years: 0 1 2 3 4 Worst case 0.30 6,000 $3,600 -$6,000
Project New Tech 1,500 315 315 315 315 Base case 0.50 10,000 4,200 +13,000
Project Old Tech 600 600 600 600 600 Best case 0.20 13,000 4,400 +28,000
a. 2,521 d. 2,543 a. Expected NPV = $35,000; σNPV = 17,500; CVNPV = 2.00
b. 2,399 e. 2,422 b. Expected NPV = $35,000; σNPV = 11,667; CVNPV = 0.33
c. 2,457 Brigham c. Expected NPV = $10,300; σNPV = 12,083; CVNPV = 1.17
d. Expected NPV = $13,900; σNPV = 8,476; CVNPV = 0.61
Risky projects e. Expected NPV = $10,300; σNPV = 13,900; CVNPV = 1.35 Brigham
190
. Cochran Corporation has a weighted average cost of capital of 11 percent for projects of
average risk. Projects of below-average risk have a cost of capital of 9 percent, while projects Questions 50 thru 53 are based on the following information Brigham
of above-average risk have a cost of capital equal to 13 percent. Projects A and B are mutually The president of Real Time Inc. has asked you to evaluate the proposed acquisition of a new
exclusive, whereas all other projects are independent. None of the projects will be repeated. computer. The computer’s price is $40,000, and it falls into the MACRS 3-year class. Purchase of
The following table summarizes the cash flows, internal rate of return (IRR), and risk of each of the computer would require an increase in net operating working capital of $2,000. The computer
the projects. would increase the firm’s before-tax revenues by $20,000 per year but would also increase
Year Project A Project B Project C Project D Project E operating costs by $5,000 per year. The computer is expected to be used for three years and then
0 -$200,000 -$100,000 -$100,000 -$100,000 -$100,000 be sold for $25,000. The firm’s marginal tax rate is 40 percent, and the project’s cost of capital is 14
1 66,000 30,000 30,000 30,000 40,000 percent. (MACRS table required)
2 66,000 30,000 30,000 30,000 25,000
192
3 66,000 40,000 30,000 40,000 30,000 . What is the net investment required at t = 0? (E)
4 66,000 40,000 40,000 50,000 35,000 a. -$42,000 d. -$37,600
b. -$40,000 e. -$36,600
IRR 12.110% 14.038% 10.848% 16.636% 11.630% c. -$38,600
Project Below Below Average Above Above
193
Risk Average Average Average Average . What is the operating cash flow in Year 2? (M)
Which projects will the firm select for investment? (M) a. $ 9,000 d. $13,453
b. $10,240 e. $16,200
CMA EXAMINATION QUESTIONS Page 66 of 107
MANAGEMENT ADVISORY SERVICES CAPITAL BUDGETING

c. $11,687 b. -$ 562 e. $1,034


c. $ 0
194
. What is the total value of the terminal year non-operating cash flows at the end of Year 3? (M)
a. $18,120 d. $25,000
b. $19,000 e. $27,000 ANSWER EXPLANATIONS
c. $21,000
195
. What is the project’s NPV? (M)
a. $2,622 d. $5,712
b. $2,803 e. $6,438
c. $2,917

Questions 54 thru 57 are based on the following information. Brigham


You have been asked by the president of your company to evaluate the proposed acquisition of a
new special-purpose truck. The truck’s basic price is $50,000, and it will cost another $10,000 to
modify it for special use by your firm. The truck falls into the MACRS 3-year class, and it will be
sold after three years for $20,000. Use of the truck will require an increase in net operating working
capital (spare parts inventory) of $2,000. The truck will have no effect on revenues, but it is
expected to save the firm $20,000 per year in before-tax operating costs, mainly labor. The firm’s
marginal tax rate is 40 percent. (MACRS table required)
196
. What is the net investment in the truck? (That is, what is the Year 0 net cash flow?) (E)
a. -$50,000 d. -$62,000
b. -$52,600 e. -$65,000
c. -$55,800
197
. What is the operating cash flow in Year 1? (M)
a. $17,820 d. $20,121
b. $18,254 e. $21,737
c. $19,920
198
. What is the total value of the terminal year non-operating cash flows at the end of Year 3? (M)
a. $10,000 d. $16,000
b. $12,000 e. $18,000
c. $15,680
199
. The truck’s cost of capital is 10 percent. What is its NPV? (M)
a. -$1,547 d. $ 562
CMA EXAMINATION QUESTIONS Page 67 of 107
1
. Answer (D) is correct. Capital budgeting is the process of planning expenditures for long-lived assets. It involves
choosing among investment proposals using a ranking procedure. Evaluations are based on various measures involving
rate of return on investment.
Answer (A) is incorrect because capital budgeting involves long-term investment needs, not immediate operating needs.
Answer (B) is incorrect because strategic planning establishes long-term goals in the context of relevant factors in the
firm's environment. Answer (C) is incorrect because cash budgeting determines operating cash flows. Capital budgeting
evaluates the rate of return on specific investment alternatives.
2
. Answer (D) is correct. Capital budgeting is a long-term planning process for investments. This process begins with
the identification of capital needs, that is, of projects required to achieve organizational goals. The next step is to search
for specific investments. The third step is to acquire and analyze information about the potential choices. The fourth step
is to select specific investments after considering both qualitative and quantitative factors. The fifth step is to finance the
undertakings. The final step is implementation and monitoring.
Answer (A) is incorrect because analyzing capital addition proposals is a step that is subsequent to identifying capital
addition projects and other capital needs. Answer (B) is incorrect because making expenditure decisions is a step
subsequent to identifying capital addition projects and other capital needs. Answer (C) is incorrect because analyzing
and evaluating all promising alternatives is a step that is subsequent to identifying capital addition projects and other
capital needs.
3
. Answer (C) is correct. Capital budgeting is concerned with long-range decisions, such as whether to add a product
line, to build new facilities, or to lease or buy equipment. Any decision regarding cash inflows and outflows over a period
of more than 1 year probably needs capital budgeting analysis.
Answer (A) is incorrect because capital budgeting is useful for all long-range decision making. Answer (B) is incorrect
because capital budgeting is not useful for short-range decisions. Answer (D) is incorrect because it is a nonsense
answer.
4
. Answer (D) is correct. The capital budgeting process is a method of planning the efficient expenditure of the firm's
resources on capital projects. Such planning is essential in view of the rising costs of scarce resources.
Answer (A) is incorrect because capital budgeting may also be used for analysis of multiple profitable alternatives and of
lease-or-buy decisions. Answer (B) is incorrect because capital budgeting permits analysis of adding or discontinuing
product lines or facilities and of lease-or-buy decisions. Answer (C) is incorrect because the lease-or-buy decision is just
one specific example of an appropriate use of capital budgeting techniques.
5
. Answer (D) is correct. Capital budgeting is the process of planning expenditures for investments on which the
returns are expected to occur over a period of more than 1 year. Thus, capital budgeting concerns the acquisition or
disposal of long-term assets and the financing ramifications of such decisions. The adoption of a new method of
allocating nontraceable costs to product lines has no effect on a company's cash flows, does not concern the acquisition
of long-term assets, and is not concerned with financing. Hence, capital budgeting is irrelevant to such a decision.
Answer (A) is incorrect because a new aircraft represents a long-term investment in a capital good. Answer (B) is
incorrect because a major advertising program is a high cost investment with long-term effects. Answer (C) is incorrect
because a star quarterback is a costly asset who is expected to have a substantial effect on the team's long-term
profitability.
6
. Answer (C) is correct. The investment tax credit is of no concern because it no longer exists. The 1986 Tax Reform
Act eliminated the investment tax credit.
Answer (A) is incorrect because the availability of any necessary financing should be considered even though the net
present value method indicates that the project is acceptable. Answer (B) is incorrect because the probability of near-
term technological changes to the manufacturing process should be considered even though the net present value
method indicates that the project is acceptable. Answer (D) is incorrect because maintenance requirements, warranties,
and availability of service arrangements should be considered even though the net present value method indicates that
the project is acceptable.
7
. Estimating cash flows Answer: b Diff: M
8
. Answer (D) is correct. The investment in a new project includes more than the initial cost of new capital equipment.
In addition, funds must be provided for increases in receivables and inventories. This investment in working capital is
treated as an initial cost of the investment that will be recovered in full at the end of the project's life.
Answer (A) is incorrect because the investment in working capital will be needed throughout the life of the investment.
Answer (B) is incorrect because cash will be needed to fund the investments in receivables and inventory. Answer (C) is
incorrect because the initial investment should be treated as an initial cash outflow, but one that will be recovered at the
end of the project.
9
. Relevant cash flows Answer: b Diff: E N

Sunk costs are never included in project cash flows, so statement a is false.
Externalities are always included, so statement b is true. Since the weighted
average cost of capital includes the cost of debt, and this is the discount
rate used to evaluate project cash flows, interest expense should not be
included in project cash flows. Therefore, statement c is false.
10
. Relevant cash flows Answer: d Diff: E N

Sunk costs should be ignored, but externalities and opportunity costs should
be included in the project evaluation. Therefore, the correct choice is
statement d.

11
. Answer (B) is correct. Tax depreciation is relevant to cash flow analysis because it affects the amount of income
taxes that must be paid. However, book depreciation is not relevant because it does not affect the amount of cash
generated by an investment.
Answer (A) is incorrect because it is a true statement relating to capital budgeting. Answer (C) is incorrect because it is
a true statement relating to capital budgeting. Answer (D) is incorrect because it is a true statement relating to capital
budgeting.
12
. Relevant cash flows Answer: c Diff: E
The correct answer is c. Sunk costs should be excluded from the analysis, and
interest expense is incorporated in the WACC and not the cash flows.
13
. Cash flows and accounting measures Answer: d Diff: M
14. Relevant cash flows Answer:
d Diff: E
Statements a and c are correct; therefore, statement d is the correct answer.
Net cash flow = Net income + depreciation; therefore, depreciation affects
operating cash flows. Sunk costs should be disregarded when making investment
decisions, while opportunity costs should be considered when making
investment decisions, as they represent the best alternative use of an asset.

15

. Relevant cash flows Answer: c Diff: E


16
. Relevant cash flows Answer: d Diff: M
Statements b and c are correct; therefore, statement d is the correct answer.
The $3 million spent on researching the technology is a sunk cost.
17
. Relevant cash flows Answer: d Diff: M N

Statement a is a sunk cost and sunk costs are never included in the capital
budgeting analysis. Therefore, statement a is not included. Statement b is an
opportunity cost and should be included in the capital budgeting analysis.
Statement c is the cannibalization of existing products, which will cause the
company to forgo cash flows and profits in another division. Therefore, it is
included in the capital budgeting analysis. Therefore, the correct answer is
statement d.
18
. Relevant cash flows Answer: d Diff: M
Statements b and c are correct; therefore, statement d is the correct answer.
The cost of clearing the land is a sunk cost and should not be considered in
the analysis. The expected impact of the new store on the existing store
should be considered. In addition, the opportunity to lease the land
represents an opportunity cost of opening a new store on the land and should
be considered.
19

. Relevant cash flows Answer: d Diff: M


Statements a and c are correct; therefore, statement d is correct.
Externalities and opportunity costs should be considered, while sunk costs
should not be included in the analysis.
20
. Cash flow estimation Answer: d Diff: M
Statement d is true--the forgone rent is an “opportunity cost” which should
be charged to the project under consideration. Note that Statements a and b
are both false—the cash flows should not take account of interest, because
financial costs are dealt with by discounting at the WACC. If interest were
deducted to find cash flows, then this cost would be “double counted,” and
the NPV would be downward biased. Ignoring interest when determining cash
flows produces no bias in the NPV whatever. Note also that externalities can
be either positive or negative—they tend to be negative if the new project is
a substitute for existing products, but positive if the new project is
complementary to the firm’s other products.
21
. Incremental cash flows Answer: d Diff: M
22
. Incremental cash flows Answer: d Diff: M
23
. Answer (B) is correct. A tax shield is something that will protect income against taxation. Thus, a depreciation tax
shield is a reduction in income taxes due to a company's being allowed to deduct depreciation against otherwise taxable
income.
Answer (A) is incorrect because a tax shield is not a cash flow, but a means of reducing outflows for income taxes.
Answer (C) is incorrect because cash is not provided by recording depreciation; the shield is a result of deducting
depreciation from taxable revenues. Answer (D) is incorrect because depreciation is recognized as an expense even if it
has no tax benefit.
24
. Answer (C) is correct. A depreciation deduction will reduce a firm's tax by the amount of the deduction times the
marginal tax rate. A dollar deducted is offset against the firm's last (marginal) dollar of income.
Answer (A) is incorrect because the savings will apply at the marginal rate. Without the deduction, income would be
higher and therefore subject to the marginal tax rate. Answer (B) is incorrect because the marginal rate is relevant to tax
savings from depreciation. Answer (D) is incorrect because one minus the firm's marginal tax rate times the
depreciation amount describes the effect on income, not taxes.
25
. Answer (C) is correct. Accelerated depreciation results in greater depreciation in the early years of an asset's life
compared with the straight-line method. Thus, accelerated depreciation results in lower income tax expense in the early
years of a project and higher income tax expense in the later years. By effectively deferring taxes, the accelerated
method increases the present value of the depreciation tax shield.
Answer (A) is incorrect because the hurdle rate can be reached more easily as a result of the increased present value of
the depreciation tax shield. Answer (B) is incorrect because the greater depreciation tax shield increases the NPV.
Answer (D) is incorrect because greater initial depreciation reduces the cash outflows for the taxes, but has no effect on
the initial cash outflows.
26
. Answer (D) is correct. According to microeconomic theory, a firm should produce until its marginal revenue equals
its marginal cost. In capital budgeting terms, marginal revenue is the marginal rate of return on investment, and marginal
cost is the company's marginal cost of capital (MCC). Hence, the firm should continue to invest until the cost of the last
investment equals the return.
Answer (A) is incorrect because the firm must balance cost and return. Minimizing MCC or average cost of capital (ACC
is minimized when it equals MCC) ignores possible returns. Answer (B) is incorrect because the firm must balance cost
and return. Minimizing MCC or average cost of capital (ACC is minimized when it equals MCC) ignores possible returns.
Answer (C) is incorrect because the rate of return on total assets is an average return. Setting MCC equal to this rate
may result in acceptance of poor investments.
27
. REQUIRED: The determinant of the optimal capital budget.
DISCUSSION: (A) In economics, a basic principle is that a firm should increase output until marginal cost equals
marginal revenue. Similarly, the optimal capital budget is determined by calculating the point at which marginal cost of
capital (which increases as capital requirements increase) and marginal efficiency of investment (which decreases if the
most profitable projects are accepted first) intersect.
Answer (B) is incorrect because the intersection of average marginal cost with average projected rates of return when
the largest (not most profitable) projects are accepted first offers no meaningful capital budgeting conclusion. Answer
(C) is incorrect because the optimal capital budget may exclude profitable projects as lower cost capital goes first to
projects with higher rates of return. Answer (D) is incorrect because accepting projects with rates of return lower than
the cost of capital is not rational.
28
. Find the WACCs using both John’s and Becky’s methods. (WACC = k s because there is no debt).

John’s WACC for Division B based on overall company’s beta:


k = kRF + RPM(b)
k = 5% + 5%(1.2)
k = 5% + 6%
k = 11%.

Therefore, John would only choose Project 1, because it is the only project whose IRR exceeds its cost of capital.
Consequently, the firm’s capital budget (based on John’s WACC) is only $400 million.

Becky’s WACC for Division B:


k = kRF + RPM(b)
k = 5% + 5%(0.9)
k = 5% + 4.5%
k = 9.5%.

Becky would choose projects 1, 2, 3, and 4 because all of these projects have an IRR that exceeds the Division’s 9.5
percent cost of capital. Based on Becky’s WACC, the firm’s capital budget would be $1,270 million ($400 + $300 + $250
+ $320). Therefore, the firm’s capital budget based on Becky’s WACC is $870 million ($1,270 - $400) larger than the one
based on John’s WACC.
29
. The option to abandon will increase expected cash flow and decrease risk. If a firm has the option to
abandon a project, it will choose to do so only when things look bad (negative NPV). Thus, abandoning a
project eliminates the low/negative cash flows. Therefore, statement b is correct.
30
. By having the ability to wait and see you reduce the risk of the project. Therefore, statement a is false. The greater
the uncertainty, the more value there is in waiting for additional information before going on with a project. Therefore,
statement b is false. Statement c is not necessarily true. By waiting to do a project you may lose strategic advantages
associated with being the first competitor to enter a new line of business, which may alter the cash flows. Since
statements a, b, and c are false, the correct choice is statement e.
31
. Statements a, b, c, and d are all examples of different types of real options. A flexibility option permits the firm to
alter operations depending on how conditions change during the life of the project. Typically, either inputs or outputs, or
both, can be changed. Statement a is an example of an investment timing option, while statement b is an example of an
abandonment option. Statement c is an example of a flexibility option, while statement d is an example of a growth
option. Therefore, statement c is the correct choice.
32
. By failing to consider both abandonment and growth options, the firm’s capital budget would be too small. In both
cases, the firm might reject what might otherwise be profitable projects if these options had been considered. Therefore,
the correct choice is statement a.
33
. Answer (D) is correct. The accounting rate of return (unadjusted rate of return or book value rate of return) equals
accounting net income divided by the required initial or average investment. The accounting rate of return ignores the
time value of money.
Answer (A) is incorrect because the net present value is the sum of the present values of all the cash inflows and
outflows associated with an investment. Answer (B) is incorrect because the discounted payback method calculates the
payback period by determining the present values of the future cash flows. Answer (C) is incorrect because the internal
rate of return is the discount rate at which the NPV is zero.
34
. Answer (D) is correct. The accounting rate of return (also called the unadjusted rate of return or book value rate of
return) measures investment performance by dividing the accounting net income by the average investment in the
project. This method ignores the time value of money.
Answer (A) is incorrect because the bail-out payback method measures the length of the payback period when the
periodic cash inflows are combined with the salvage value. Answer (B) is incorrect because the internal rate of return
method determines the rate at which the NPV is zero. Answer (C) is incorrect because the profitability index is the ratio
of the present value of future net cash inflows to the initial cash investment.
35
. Answer (D) is correct. The accounting rate of return is calculated by dividing the annual after-tax net income from a
project by the book value of the investment in that project. The time value of money is ignored.
Answer (A) is incorrect because the average rate of return method does not divide by the average investment cost.
Answer (B) is incorrect because the internal rate of return incorporates the time value of money into the calculation. The
IRR is the discount rate that results in a net present value of zero. Answer (C) is incorrect because the capital asset
pricing model is a means of determining the cost of capital.
36
. Answer (B) is correct. The accounting rate of return (also called the unadjusted rate of return or book value rate of
return) is calculated by dividing the increase in accounting net income by the required investment. Sometimes the
denominator is the average investment rather than the initial investment. This method ignores the time value of money
and focuses on income as opposed to cash flows.
Answer (A) is incorrect because the IRR is the rate at which the net present value is zero. Thus, it incorporates time
value of money concepts, whereas the accounting rate of return does not. Answer (C) is incorrect because the
accounting rate of return is similar to the divisional performance measure of return on investment. Answer (D) is
incorrect because the accounting rate of return ignores the time value of money.
37
. Answer (D) is correct. The accounting rate of return (ARR) is based on financial statements prepared on the
accrual basis. The formula to compute the ARR is:
Expected increase in annual net income
ARR =
Initial (or average) investment
Both the revenue over life of project and depreciation expense are used in the calculation of the ARR. Depreciation
expense over the project’s life and other expenses directly associated with the project under consideration including
income tax effects are subtracted from revenue over life of the project to determine net income over life of project. Net
income over the project’s life is then divided by the economic life to determine annual net income, the numerator of the
ARR formula. This is a weakness of the ARR method because it does not consider actual cash flows or the time value
of money.
38
. Answer (B) is correct. The accounting rate of return uses undiscounted net income (not cash flows) to determine a
rate of profitability. Annual after-tax net income is divided by the average book value (or the initial value) of the
investment in assets. Answer (C) is incorrect because the payback period is the time required to complete the return of
the original investment. This method gives no consideration to the time value of money or to returns after the payback
period.
Answer (A) is incorrect because the internal rate of return is the rate at which NPV is zero. The minimum desired rate of
return is not used in the discounting. Answer (D) is incorrect because the NPV method computes the discounted
present value of future cash inflows to determine whether it is greater than the initial cash outflow.
39
. Answer (B) is correct. The payback method measures the number of years required to complete the return of the
original investment. This measure is computed by dividing the net investment by the average expected cash inflows to
be generated, resulting in the number of years required to recover the original investment. The payback method gives
no consideration to the time value of money, and there is no consideration of returns after the payback period.
Answer (A) is incorrect because the discounted cash flow method computes a rate of return. Answer (C) is incorrect
because the net present value method is based on discounted cash flows; the length of time to recover an investment is
not the result. Answer (D) is incorrect because the net present value method is based on discounted cash flows; the
length of time to recover an investment is not the result.
40
. Answer (B) is correct. The usual payback formula divides the initial investment by the constant net annual cash
inflow. The payback method is unsophisticated in that it ignores the time value of money, but it is widely used because of
its simplicity and emphasis on recovery of the initial investment.
Answer (A) is incorrect because the net present value method first discounts the future cash flows to their present value.
Answer (C) is incorrect because the profitability index method divides the present value of the future net cash inflows by
the initial investment. Answer (D) is incorrect because the accounting rate of return divides the annual net income by the
average investment in the project.
41
. Answer (B) is correct. The payback method calculates the number of years required to complete the return of the
original investment. This measure is computed by dividing the net investment required by the average expected cash
flow to be generated, resulting in the number of years required to recover the original investment. Payback is easy to
calculate but has two principal problems: it ignores the time value of money, and it gives no consideration to returns after
the payback period. Thus, it ignores total project profitability.
Answer (A) is incorrect because the payback method does not incorporate the time value of money. Answer (C) is
incorrect because the payback method uses the net investment in the numerator of the calculation. Answer (D) is
incorrect because payback uses the net annual cash inflows in the denominator of the calculation.
42
. Answer (A) is correct. The payback method calculates the amount of time required to complete the return of the
original investment, i.e., the time it takes for a new asset to pay for itself. Although the payback method is easy to
calculate, it has inherent problems. The time value of money and returns after the payback period are not considered.
Answer (B) is incorrect because the payback method ignores cash flows after payback. Answer (C) is incorrect because
the payback method does not use discounted cash flow techniques. Answer (D) is incorrect because the payback
method may lead to different decisions.
43
. Answer (D) is correct. The payback reciprocal (1 ÷ payback) has been shown to approximate the internal rate of
return (IRR) when the periodic cash flows are equal and the life of the project is at least twice the payback period.
Answer (A) is incorrect because the payback reciprocal is not related to the profitability index. Answer (B) is incorrect
because the payback reciprocal approximates the IRR, which is the rate at which the NPV is $0. Answer (C) is incorrect
because the accounting rate of return is based on accrual-income based figures, not on discounted cash flows.
44
. Answer (C) is correct. The payback period is computed by dividing the initial investment by the annual net cash
inflow. Depreciation expense is not subtracted from cash inflow; ony the income taxes which are cause by the
depreciation deduction are substracted. One of the weaknesses of the payback period is that is ignores the time value
of time.
45
. Answer (B) is correct. The bailout payback period is the length of time required for the sum of the cumulative net
cash inflow from an investment and its salvage value to equal the original investment. The bailout payback method
measures the risk to the investor if the investment must be abandoned. The shorter the period, the lower the risk.
Answer (A) is incorrect because the use of the bailout payback method is not limited to firms with federally insured
loans. Answer (C) is incorrect because the payback period is calculated by summing the net cash inflow and the
salvage value. Answer (D) is incorrect because the bailout payback method does not estimate short-term profitability.
46
. Answer (D) is correct. The payback period equals the net investment divided by the average expected cash flow,
resulting in the number of years required to recover the original investment. The bailout payback incorporates the
salvage value of the asset into the calculation. It determines the length of the payback period when the periodic cash
inflows are combined with the salvage value. Hence, the method measures risk. The longer the payback period, the
more risky the investment.
Answer (A) is incorrect because the bailout payback method does not consider the time value of money. Answer (B) is
incorrect because the bailout payback includes salvage value as well as cash flow from operations. Answer (C) is
incorrect because the bailout payback incorporates the disposal value in the payback calculation.
47
. Answer (A) is correct. Time value of money means that, because of the interest factor, money held today is worth
more than the same amount of money to be received in the future. Interest is paid for the use of money, i.e., on debts, in
a normal business transaction. This payment compensates the lender for not being able to use the money for current
consumption.
Answer (B) is incorrect because present value is the value today, net of the interest factor, of one or more payments to
be made in the future. Answer (C) is incorrect because future value is the value some time in the future of a deposit
today or of a series of deposits. Answer (D) is incorrect because an annuity is generally a series of equal payments at
equal intervals of time.
48
. Answer (A) is correct. The time value of money is concerned with two issues: (1) the investment value of money,
and (2) the risk (uncertainty) inherent in any executory agreement. Thus, a dollar today is worth more than a dollar in the
future, and the longer one waits for a dollar, the more uncertain the receipt is. The cost of capital involves a specific
application of the time value of money principles. It is not a basic concept thereof.
Answer (B) is incorrect because risk is a basic time value of money concept. Cost of capital is not. Answer (C) is
incorrect because the interest effect is a basic time value of money concept. Answer (D) is incorrect because the interest
effect and risk are basic time value of money concepts. Cost of capital is not.
49
. Answer (A) is correct. The present value concept may be applied both to dollars-in (inflows) and to dollars-out
(outflows). Thus, individual cash inflows and cash outflows or a series thereof (an annuity) may be discounted to time
zero (the present). Net present value is the sum of discounted cash inflows minus any discounted cash outflows. Net
present value may be either positive or negative.
Answer (B) is incorrect because a present value may be calculated for discounted cash outflows. Answer (C) is incorrect
because a present value may be calculated for discounted cash inflows or a series thereof (an annuity). Answer (D) is
incorrect because a present value may be calculated for discounted cash inflows or outflows.
50
. Answer (C) is correct. Discounted cash flow analysis, using either the internal rate of return (IRR) or the net present
value (NPV) method, is based on the time value of cash inflows and outflows. All future operating cash savings are
considered as well as the tax effects on cash flows of future depreciation charges. The cash proceeds of future asset
disposals are likewise a necessary consideration. Depreciation expense is a consideration only to the extent that it
affects the cash flows for taxes. Otherwise, depreciation is excluded from the analysis because it is a noncash expense.
Answer (A) is incorrect because future operating cash savings is a consideration in discounted cash flow analysis.
Answer (B) is incorrect because the current asset disposal price is a consideration in discounted cash flow analysis.
Answer (D) is incorrect because the tax effects of future asset depreciation is a consideration in discounted cash flow
analysis.
51
. Answer (D) is correct. The capital budgeting methods that are generally considered the best for long-range decision
making are the internal rate of return and net present value methods. These are both discounted cash flow methods.
Answer (A) is incorrect because the payback method gives no consideration to the time value of money or to returns
after the payback period. Answer (B) is incorrect because the accounting rate of return does not consider the time value
of money. Answer (C) is incorrect because the unadjusted rate of return does not consider the time value of money.
52
. DISCUSSION: (A) The time value of money is concerned with two issues: (1) the investment value of money, and
(2) the risk (uncertainty) inherent in any executory agreement. Thus, a dollar today is worth more than a dollar in the
future, and the longer one waits for a dollar, the more uncertain the receipt is.
Answers (B), (C) and (D) are incorrect because risk and interest factors are concepts underlying the time value of
money.
53
. Answer (C) is correct. Depreciation is a noncash expense that is deductible for federal income tax purposes. Hence,
it directly reduces the cash outlay for income taxes and is explicitly incorporated in the capital budgeting model.
Answer (A) is incorrect because depreciation is not a cost of operations in the capital budgeting model. Also,
depreciation can be avoided by not making investments. Answer (B) is incorrect because depreciation is an allocation of
historical cost and as such is not a cash inflow, but it may reduce cash outflows for taxes. Answer (D) is incorrect
because periodic depreciation is determined by spreading the depreciation base, i.e., the cost of the asset minus
salvage value, not the initial cash outflow, over the life of the investment.
54
. REQUIRED: The benchmark cost of capital.
DISCUSSION: (D) A weighted average of the costs of all financing sources should be used, with the weights
determined by the usual financing proportions. The terms of any financing raised at the time of initiating a particular
project do not represent the cost of capital for the firm. When a firm achieves its optimal capital structure, the weighted-
average cost of capital is minimized.
Answers (A), (B), and (C) are incorrect because the cost of capital is a composite, or weighted average, of all financing
sources in their usual proportions. The cost of capital should also be calculated on an after-tax basis.
55
. REQUIRED: To determine when the discount rate (hurdle rate) must be determined before a capital budgeting
method can be used.
Answer (C) is correct. The net present value method calculates the expected net monetary gain or loss from a project
by discounting all expected future cash inflows and outflows to the present using some predetermined minimum desired
rate of return (hurdle rate).
Answer (A) is incorrect because the payback method measures the time it will take to recoup, in the form of cash inflows
from operations, the initial dollars invested in a project. The payback period does not consider the time value of money.
Answers (B) and (D) are incorrect. The time adjusted rate of return method is also called the internal rate of return
method. This method computes the rate of interest at which the present value of expected cash inflows from a project
equals the present value of expected cash outflows of the project. Here, the discount rate is not determined in advance
but is the end result of the calculation.
56
. Answer (D) is correct. A hurdle rate is not necessary in calculating the accounting rate of return. That return is
calculated by dividing the net income from a project by the investment in the project. Similarly, a company can calculate
the internal rate of return (IRR) without knowing its hurdle rate. The IRR is the discount rate at which the net present
value is $0. However, the NPV cannot be calculated without knowing the company's hurdle rate. The NPV method
requires that future cash flows be discounted using the hurdle rate.
Answer (A) is incorrect because the accounting rate of return and the IRR but not the NPV can be calculated without
knowing the hurdle rate. Answer (B) is incorrect because the accounting rate of return and the IRR but not the NPV can
be calculated without knowing the hurdle rate. Answer (C) is incorrect because the accounting rate of return and the IRR
but not the NPV can be calculated without knowing the hurdle rate.
57
. Answer (D) is correct. Breakeven time evaluates the rapidity of new product development. The usual calculation
determines the period beginning with project approval that is required for the discounted cumulative cash inflows to
equal the discounted cumulative cash outflows. However, it may also be calculated as the point at which discounted
cumulative cash inflows on a project equal discounted total cash outflows. The concept is similar to the payback period,
but it is more sophisticated because it incorporates the time value of money. It also differs from the payback method
because the period covered begins at the outset of a project, not when the initial cash outflow occurs.
Answer (A) is incorrect because it is related to breakeven point, not breakeven time. Answer (B) is incorrect because the
payback period equals investment divided by annual undiscounted net cash inflows. Answer (C) is incorrect because the
payback period is the period required for total undiscounted cash inflows to equal total undiscounted cash outflows.
58
. Answer (C) is correct. The profitability index is another term for the excess present value index. It measures the
ratio of the present value of future net cash inflows to the original investment. In organizations with unlimited capital
funds, this index will produce no conflicts in the decision process. If capital rationing is necessary, the index will be an
insufficient determinant. The capital available as well as the dollar amount of the net present value must both be
considered.
Answer (A) is incorrect because capital rationing is not a technique but rather a condition that characterizes capital
budgeting when insufficient capital is available to finance all profitable investment opportunities. Answer (B) is incorrect
because the average rate of return method does not divide the future cash flows by the cost of the investment. Answer
(D) is incorrect because the accounting rate of return does not recognize the time value of money.
59
. Answer (D) is correct. The profitability index, also known as the excess present value index, is the ratio of the
present value of future net cash inflows to the initial net cash investment (discounted cash outflows). This tool is a
variation of the NPV method that facilitates comparison of different-sized investments.
Answer (A) is incorrect because the cash inflows are also discounted in the profitability index. Answer (B) is incorrect
because the numerator is the discounted net cash inflows. Answer (C) is incorrect because the profitability index is
based on cash flows, not profits.
60
. Answer (B) is correct. The net present value (NPV) method computes the discounted present value of future cash
inflows to determine whether they are greater than the initial cash outflow. The discount rate (cost of capital or hurdle
rate) must be known to discount the future cash inflows. If the NPV is positive (present value of future cash inflows
exceeds initial cash outflow), the project should be accepted. If the NPV is negative, the project should be rejected.
Answer (A) is incorrect because the accounting rate of return uses net income (not cash flows) to determine a rate of
profitability. Answer (C) is incorrect because the internal rate of return is the rate at which NPV is zero. The minimum
desired rate of return is not used. Answer (D) is incorrect because the payback method measures the time required to
complete the return of the original investment. It gives no consideration to the time value of money or to returns after the
payback period.
61
. Answer (A) is correct. The net present value method discounts future cash flows to the present value using some
arbitrary rate of return, which is presumably the firm's cost of capital. The initial cost of the project is then deducted from
the present value. If the present value of the future cash flows exceeds the cost, the investment is considered to be
acceptable.
Answer (B) is incorrect because the payback method does not recognize the time value of money. Answer (C) is
incorrect because the average rate of return method does not use the firm's cost of capital as a discount rate. Answer
(D) is incorrect because the accounting rate of return method does not recognize the time value of money.
62
. REQUIRED: The effect on the value of the firm and its stock price of investment opportunities.
DISCUSSION: (A) Investments with present values in excess of their costs (positive NPVs) that can be identified or
created by the capital budgeting activities of the firm will have a positive impact on firm value and on the price of the
common stock of the firm. Accordingly, the more effective capital budgeting is, the higher the stock price.
Answer (B) is incorrect because positive NPV investments will increase, not decrease firm value and share price.
Answers (C) and (D) are incorrect because investments with present values equal to their costs have a zero NPV and
neither increase nor decrease firm value and share price.
63
. Answer (A) is correct. The NPV method computes the present value of future cash inflows to determine whether
they are greater than the initial cash outflow. Future cash inflows include any salvage value on facilities. Included in the
initial investment are the cost of new equipment and other facilities, and additional working capital needed for operations
during the term of the project. The discount rate (cost of capital or hurdle rate) must be known to discount the future
cash inflows. If the NPV is positive, the project should be accepted. The method of funding a project is a decision
separate from that of whether to invest.
Answer (B) is incorrect because the initial costs of the project, including additional working capital needs, are necessary
to determine the NPV. Answer (C) is incorrect because the initial costs of the project, including additional working capital
needs, are necessary to determine the NPV. Answer (D) is incorrect because the project's salvage value is a future cash
inflow to be discounted.
64
. REQUIRED: The variable(s) considered in the NPV calculation.
DISCUSSION: (D)The NPV is the difference between the present value of the future cash flows from the project
discounted at an appropriate interest rate and the initial investment. If the NPV is zero or greater, the investment may
be economically rational. The method is a technique for ranking investment proposals. Consequently, the time value of
the cash flows over the life of the project is considered.
Answers (A), (B), and (C) are incorrect because the time value of the cash flows over the life of the project is
considered.
65
. Answer (D) is correct. The effect of assuming cash flows occur at the end of the year simply understates the present
values of the future cash flows; in reality, they probably occur on the average at mid-year.
Answer (A) is incorrect because cash flows in investment decisions do not all occur at the end of each year. Answer (B)
is incorrect because discounting cash flows approximately 6 months longer understates rather than overstates. Answer
(C) is incorrect because the effect of using the year-end assumption produces a slight conservatism in the model but
does not render the results unusable.
66
. Answer (C) is correct. The NPV method is used when the discount rate (cost of capital) is specified. It assumes that
cash flows from the investment can be reinvested at the particular project's cost of capital (the discount rate).
Answer (A) is incorrect because the internal rate of return method assumes that cash flows are reinvested at the internal
rate of return. Answer (B) is incorrect because the NPV method assumes that cash flows are reinvested at the NPV
discount rate. Answer (D) is incorrect because the NPV method assumes that cash flows are reinvested at the NPV
discount rate.
67
. Answer (D) is correct. The NPV method assumes that periodic cash inflows earned over the life of an investment
are reinvested at the company's cost of capital (i.e., the discount rate used in the analysis). This is contrary to the
assumption under the internal rate of return method, which assumes that cash inflows are reinvested at the internal rate
of return. As a result of this difference, the two methods will occasionally give different rankings of investment
alternatives.
Answer (A) is incorrect because the NPV method assumes that cash inflows are reinvested at the discount rate used in
the NPV calculation. Answer (B) is incorrect because the NPV method assumes that cash inflows are reinvested at the
discount rate used in the NPV calculation. Answer (C) is incorrect because the internal rate of return method assumes a
reinvestment rate equal to the rate of return on the project.
68
. Answer (B) is correct. The NPV method calculates the present values of estimated future net cash inflows and
compares the total with the net cost of the investment. The cost of capital must be specified. If the NPV is positive, the
project should be accepted. The IRR method computes the interest rate at which the NPV is zero. The IRR method is
relatively easy to use when cash inflows are the same from one year to the next. However, when cash inflows differ from
year to year, the IRR can be found only through the use of trial and error. In such cases, the NPV method is usually
easier to apply. Also, the NPV method can be used when the rate of return required for each year varies. For example, a
company might want to achieve a higher rate of return in later years when risk might be greater. Only the NPV method
can incorporate varying levels of rates of return.
Answer (A) is incorrect because the IRR method calculates a rate of return. Answer (C) is incorrect because the IRR is
the rate at which NPV is zero. Answer (D) is incorrect because both methods discount cash flows.
69
. Answer (C) is correct. The NPV is broadly defined as the excess of the present value of the estimated net cash
inflows over the net cost of the investment. A discount rate has to be stipulated by the person conducting the analysis. A
disadvantage is that it does not provide the true rate of return for an investment, only that the rate of return is higher than
a stipulated discount rate (which may be the cost of capital).
Answer (A) is incorrect because the ability to perform sensitivity analysis is an advantage of the NPV method. Answer
(B) is incorrect because the NPV method does not compute the true interest rate. Answer (D) is incorrect because the
IRR method, not the NPV method, uses a trial-and-error approach when cash flows are not identical from year to year.
70
. Answer (C) is correct. The discount rate most often used in present value calculations is the minimum desired rate
of return as set by management. The NPV arrived at in this calculation is a first step in the decision process. It indicates
how the project's return compares with the minimum desired rate of return.
Answer (A) is incorrect because the Federal Reserve rate may be considered; however, the firm will set its minimum
desired rate of return in view of its needs. Answer (B) is incorrect because the Treasury bill rate may be considered;
however, the firm will set its minimum desired rate of return in view of its needs. Answer (D) is incorrect because the
prime rate may be considered; however, the firm will set its minimum desired rate of return in view of its needs.
71
. Answer (A) is correct. The rate used to discount future cash flows is sometimes called the cost of capital, the
discount rate, the cutoff rate, or the hurdle rate. A risk-free rate is the rate available on risk-free investments such as
government bonds. The risk-free rate is not equivalent to the cost of capital because the latter must incorporate a risk
premium.
Answer (B) is incorrect because the rate used under the NPV method is the company's cost of capital. Answer (C) is
incorrect because the NPV method discounts future cash flows to their present values. Answer (D) is incorrect because
the cost of capital is often called a cutoff rate. Investments yielding less than the cost of capital should not be made.
72
. Answer (D) is correct. The NPV is the excess of the present values of the estimated cash inflows over the net cost
of the investment. The discount rate used is sometimes the cost of capital or other hurdle rate designated by
management. This rate is also called the required rate of return. The accounting rate of return is never used in NPV
analysis because it ignores the time value of money; it is computed by dividing the accounting net income by the
investment.
Answer (A) is incorrect because cost of capital is a synonym for the rate used in NPV analysis. Answer (B) is incorrect
because hurdle rate is a synonym for the rate used in NPV analysis. Answer (C) is incorrect because discount rate is a
synonym for the rate used in NPV analysis.
73
. REQUIRED: The matter affecting a project’s net present value.
DISCUSSION: (A) To compute a project’s net present value, the initial investment is subtracted from the present value
of the after-tax cash flows. The proceeds from the sale of the asset to be replaced reduces the initial investment.
Answer (B) is incorrect because the carrying amount of the asset to be replaced affects the gain or loss on the sale.
Answer (C) is incorrect because the amount of annual depreciation on the asset to be replaced affects the carrying
value. Answer (D) is incorrect because annual depreciation of other assets, even if used directly, does not affect the
project’s net present value.
74
. Answer (B) is correct. In an inflationary environment, nominal future cash flows should increase to reflect the
decrease in the value of the unit of measure. Also, the investor should increase the discount rate to reflect the increased
inflation premium arising from the additional uncertainty. Lenders will require a higher interest rate in an inflationary
environment.
Answer (A) is incorrect because future cash flows should also increase. Answer (C) is incorrect because the discount
rate should be increased to take into consideration future uncertainty and the risk premium that lenders will require in an
inflationary environment. Answer (D) is incorrect because cash flows should increase in an inflationary environment.
75
. Answer (A) is correct. The internal rate of return (IRR) is the discount rate at which the present value of the cash
inflows equals the present values of the cash outflows (including the original investment). Thus, the NPV of the project is
zero at the IRR. The IRR is also the maximum borrowing cost the firm can afford to pay for a specific project. The IRR is
similar to the yield rate/effective rate quoted in the business media.
Answer (B) is incorrect because the capital asset pricing model is a means of determining the cost of capital. Answer (C)
is incorrect because the profitability index is not an interest rate. Answer (D) is incorrect because the accounting rate of
return is not based on present values.
76
. Answer (C) is correct. The IRR is the interest rate at which the present value of the expected future cash inflows is
equal to the present value of the cash outflows for a project. Thus, the IRR is the interest rate that will produce a net
present value (NPV) equal to zero. The IRR method assumes that the cash flows will be reinvested at the internal rate of
return.
Answer (A) is incorrect because the hurdle rate is a concept used to calculate the NPV of a project; it is determined by
management prior to the analysis. Answer (B) is incorrect because the IRR is the rate of interest at which the NPV is
zero. Answer (D) is incorrect because the IRR is a means of evaluating potential investment projects.
77
. Answer (C) is correct. The IRR is a capital budgeting technique that calculates the interest rate that yields a net
present value equal to $0. It is the interest rate that will discount the future cash flows to an amount equal to the initial
cost of the project. Thus, the higher the IRR, the more favorable the ranking of the project.
Answer (A) is incorrect because the cost of capital is not used in the calculation of the IRR. Answer (B) is incorrect
because the IRR can be determined regardless of the constancy of the cash flows. However, it is more difficult to
calculate when cash flows are not constant because a trial-and-error approach must be used. Answer (D) is incorrect
because there is no relationship between IRR and the profitability index.
78
. REQUIRED: The true statement about internal rate of return.
DISCUSSION: (D) The internal rate of return (IRR) is the discount rate at which the present value of the cash flows
equals the original investment. Thus, the NPV of the project is zero at the IRR. The IRR is also the maximum borrowing
cost the firm could afford to pay for a specific project. The IRR is similar to the yield rate/effective rate quoted in the
business media.
Answers (A), (B), and (C) are incorrect because the IRR is the discount rate at which the NPV of the cash flows is zero,
the breakeven borrowing rate for projects, and the yield rate/effective rate of interest quoted on long-term debt and other
instruments.
79
. Answer (D) is correct. The internal rate of return (IRR) is the discount rate at which the present value of the cash
flows equals the original investment. Thus, the NPV of the project is zero at the IRR. The IRR is also the maximum
borrowing cost the firm could afford to pay for a specific project. The IRR is similar to the yield rate/effective rate quoted
in the business media.
Answer (A) is incorrect because the IRR is the discount rate at which the NPV of the cash flows is zero, the breakeven
borrowing rate for the project in question, the yield rate/effective rate of interest quoted on long-term debt and other
instruments, and favorable when it exceeds the hurdle rate. Answer (B) is incorrect because the IRR is the discount rate
at which the NPV of the cash flows is zero, the breakeven borrowing rate for the project in question, the yield
rate/effective rate of interest quoted on long-term debt and other instruments, and favorable when it exceeds the hurdle
rate. Answer (C) is incorrect because the IRR is the discount rate at which the NPV of the cash flows is zero, the
breakeven borrowing rate for the project in question, the yield rate/effective rate of interest quoted on long-term debt and
other instruments, and favorable when it exceeds the hurdle rate.
80
. Answer (D) is correct. The internal rate of return of a proposed project includes the residual sales value of a project
but not the depreciation expense. This is true because the residual sales value represents a future cash flow whereas
depreciation expense (ignoring income tax considerations) provides no cash inflow or outflow.
Answers (A), (B), and (C) are incorrect because they contain the wrong combination of responses.
81
. Answer (C) is correct. Under the internal rate of return (IRR) method, the interest rate is computed that will exactly
match the present value of the future net inflows with the initial cost of the investment. The IRR method assumes that
cash flows will be reinvested at the IRR. Thus, if the project's funds are not reinvested at the internal rate of return, the
ranking calculations obtained under the IRR method may be in error. The net present value method gives a better grasp
of the problem in many decision situations because the reinvestment is assumed to be at the cost of capital.
Answer (A) is incorrect because the IRR does calculate compounded interest rates. Answer (B) is incorrect because
both methods incorporate the time value of money. Answer (D) is incorrect because sensitivity analysis can be used
with NPV to handle multiple desired hurdle rates.
82
. Answer (D) is correct. The IRR is the rate at which the discounted future cash flows equal the net investment (NPV
= 0). One disadvantage of the method is that inflows from the early years are assumed to be reinvested at the IRR. This
assumption may not be sound. Investments in the future may not earn as high a rate as is currently available.
Answer (A) is incorrect because the IRR method considers the time value of money. Answer (B) is incorrect because the
IRR provides a straightforward decision criterion. Any project with an IRR greater than the cost of capital is acceptable.
Answer (C) is incorrect because the IRR method implicitly assumes reinvestment at the IRR; the NPV method implicitly
assumes reinvestment at the cost of capital.
83
. REQUIRED: Advantage of the IRR over the accounting rate of return.
DISCUSSION: (C) The IRR is the interest rate that equalizes the present value of future cash flows with the initial cost
of the investment. The accounting rate of return is calculated by dividing the increase in accounting net income by the
required investment. However, it ignores the time value of money and does not emphasize cash flows.
Answers (A), (B), and (D) are incorrect because both techniques recognize the project’s salvage value.
84
. Statement c is correct; the other statements are false. MIRR and NPV can conflict for mutually
exclusive projects if the projects differ in size. NPV does not suffer from the multiple IRR problem.
85
. Answer (D) is correct. The profitability index is the ratio of the present value of future net cash inflows to the initial
net cash investment. It is a variation of the NPV method that facilitates comparison of different-sized investments. A
profitability index greater than 1.0 indicates a profitable investment, or one that has a positive net present value.
Answer (A) is incorrect because the IRR is the discount rate at which the NPV is $0, which is also the rate at which the
profitability index is 1.0. The IRR cannot be determined solely from the index. Answer (B) is incorrect because, if the
index is 1.15 and the discount rate is the cost of capital, the NPV is positive, and the IRR must be higher than the cost of
capital. Answer (C) is incorrect because the IRR is a discount rate, whereas the NPV is an amount.
86
. (d) The internal rate of return method determines the rate of return at which the present value of the cash flows will
exactly equal the investment outlay. It will indicate the rate of return earned over the life of the project. The net present
value method determines the present vale of all future cash flows at a selected discount rate. If the NPV of the cash
flows is positive, the return earned b the project is higher than the selected rate. Both methods will provide the
information needed to decide if a project’s rate of return will meet Polo co.’s requirement.
87
. Answer (D) is correct. The IRR is defined as the rate at which the NPV is zero. Accordingly, if the NPV is positive at
a cost of capital of 15%, the rate (the IRR) required to reduce the NPV to zero must exceed 15%.
Answer (A) is incorrect because the accounting rate of return (net income ÷ investment) ignores the time value of
money. It is not determinable from the given information. Answer (B) is incorrect because the accounting rate of return
(net income ÷ investment) ignores the time value of money. It is not determinable from the given information. Answer (C)
is incorrect because the payback period ignores the time value of money. It is not determinable from the given
information.
88
. Answer (C) is correct. The relationship between the NPV method and the IRR method can be summarized as
follows:
NPVIRRNPV > 0IRR > Discount rateNPV = 0IRR = Discount rateNPV < 0IRR < Discount rateSince the problem states
that Neu Co. has a positive net present value on the investment, then the internal rate of return would be > 12%.
89
. Answer (A) is correct. The higher the discount rate, the lower the NPV. The IRR is the discount rate at which the
NPV is zero. Consequently, if the NPV is negative, the discount rate used must exceed the IRR.
Answer (B) is incorrect because, if the discount rate is less than the IRR, the NPV is positive. Answer (C) is incorrect
because the NPV measures the difference between a company's discount rate and the IRR. Answer (D) is incorrect
because the relationship between the discount rate and the risk-free rate is not a factor in investment analysis under the
NPV method.
90
. Statement a is correct; the other statements are false. If the projects are mutually exclusive, then
project B may have a higher NPV even though Project A has a higher IRR. IRR is calculated assuming cash
flows are reinvested at the IRR, not the cost of capital.
91
. The correct answer is a; the other statements are false. The IRR is the discount rate at which a
project’s NPV is zero. If a project’s IRR exceeds the firm’s cost of capital, then its NPV must be positive,
since NPV is calculated using the firm’s cost of capital to discount project cash flows.
92
.Statement a is true; projects with IRRs greater than the cost of capital will have a positive NPV. Statement b is false
because you know nothing about the relative magnitudes of the projects. Statement c is false because the IRR is
independent of the cost of capital. Therefore, the correct choice is statement a.
93
. The correct statement is b; the other statements are false. Since Project A’s IRR is 15%, at a WACC of 15% NPVA =
0; however, Project B would still have a positive NPV. Given the information in a, we can’t conclude which project’s NPV
is going to be greater. Since we are given no details about each project’s cash flows we cannot conclude anything about
payback. Finally, IRR is independent of the discount rate, that is, IRR stays the same no matter what the WACC is.
94
. Statement a is false. The projects could easily have different NPVs based on different cash flows and costs of
capital. Statement b is false. NPV is dependent upon the size of the project. Think about the NPV of a $3 project versus
the NPV of a $3 million project. Statement c is false. NPV is dependent on a project’s risk. Therefore, the correct choice
is statement e.
95
. A project’s NPV increases as the cost of capital declines. A project’s IRR is independent of its cost of
capital, while a project’s MIRR is dependent on the cost of capital since the terminal value in the MIRR
equation is compounded at the cost of capital.
96
. Statement a is the incorrect statement. NPV is positive if IRR is greater than the cost of capital.
97
. Statement b is correct; the other statements are incorrect. Statement a is incorrect; if the NPV > 0, then the return
must be > 12%. Statement c is incorrect; if NPV > 0, then MIRR > WACC.
98
. Statement e is correct; the other statements are incorrect. Statement a is incorrect; the two projects’ NPV profiles
could cross, consequently, a higher IRR doesn’t guarantee a higher NPV. Statement b is incorrect; it the two projects’
NPV profiles cross, Y could have a higher NPV. Statement c is incorrect; we don’t have enough information.
99
. Statement a is true because the IRR exceeds the WACC. Statement b is also true because the MIRR assumes that
the inflows are reinvested at the WACC, which is less than the IRR. Statement c is false. For a normal project, the
discounted payback is always longer than the regular payback because it takes longer for the discounted cash flows to
cover the purchase price. So, statement d is the best choice.
100
. Answer (A) is correct. If taxes are ignored, depreciation is not a consideration in any of the methods based on cash
flows because it is a non-cash expense. Thus, the internal rate of return, net present value, and payback methods would
not consider depreciation because these methods are based on cash flows. However, the accounting rate of return is
based on net income as calculated on an income statement. Because depreciation is included in the determination of
accrual accounting net income, it would affect the calculation of the accounting rate of return.
Answer (B) is incorrect because the IRR and the payback period are based on cash flows. Depreciation is not needed in
their calculation. However, the accounting rate of return cannot be calculated without first deducting depreciation.
Answer (C) is incorrect because the IRR and the payback period are based on cash flows. Depreciation is not needed in
their calculation. However, the accounting rate of return cannot be calculated without first deducting depreciation.
Answer (D) is incorrect because the IRR and the payback period are based on cash flows. Depreciation is not needed in
their calculation. However, the accounting rate of return cannot be calculated without first deducting depreciation.
101
. Answer (C) is correct. The profitability index is the ratio of the present value of future net cash inflows to the initial
net cash investment. It is a variation of the net present value (NPV) method and facilitates the comparison of different-
sized investments. Because it is based on the NPV method, the profitability index will yield the same decision as the
NPV for independent projects. However, decisions may differ for mutually exclusive projects of different sizes.
Answer (A) is incorrect because the profitability index, like the NPV method, discounts cash flows based on the cost of
capital. Answer (B) is incorrect because the profitability index is cash based. Answer (D) is incorrect because the NPV
and the profitability index may yield different decisions if projects are mutually exclusive and of different sizes.
102
. Answer (D) is correct. All three managers will reject the project. Manager one will calculate a NPV of -$12,894 [-
$100,000 + ($20,000 x 4.3553 PVIFA for six periods at 10%)]. Manager two will calculate a NPV of -$26,349 {- $100,000
+ ($5,000 x .8772 PVIF for one period at 14%) + [($23,000 x 3.4331 PVIFA for five periods at 14%) x .8772 PVIF for one
period at 14%]}. Manager three will calculate a net present value of -$41,640 [-$100,000 + ($135,000 x .4323 PVIF for
six periods at 15%)].
Answer (A) is incorrect because all three managers will calculate a negative NPV, and none will recommend
acceptance. Answer (B) is incorrect because all three managers will calculate a negative NPV, and none will recommend
acceptance. Answer (C) is incorrect because all three managers will calculate a negative NPV, and none will
recommend acceptance.
103
. Answer (A) is correct. The payback period is the number of periods it takes before the cash flows from the project
repay the original investment outlay. This can be expressed as net investment divided by the average expected cash
flow. Manager one expects inflows of $20,000 per year, so it will take exactly 5 years for the project to repay the original
$100,000 invested. Manager two will calculate a payback period of more than 5 years. Only $5,000 is expected at the
end of year one, followed by inflows of $23,000 at the end of each year in years two through six. At the end of year five,
only $97,000 will have been received, based on these expectations. Manager three will calculate a payback period of 6
years. She estimates one inflow of $135,000 at the end of year six.
Answer (B) is incorrect because manager one will calculate a 5-year payback period, which is shorter than the periods
determined by managers two and three. Answer (C) is incorrect because manager one will calculate a 5-year payback
period, which is shorter than the periods determined by managers two and three. Answer (D) is incorrect because all
three managers will derive a different payback period for the project.
104
. Answer (B) is correct. If the net present value (NPV) of an investment is positive, the project should be accepted
(unless projects are mutually exclusive). If the NPV is negative, the investment should be rejected.
Answer (A) is incorrect because the present value of future net cash inflows must be compared with the initial cash
outlay to determine whether a project is acceptable. Answer (C) is incorrect because an IRR may be greater than zero
but less than a firm's cost of capital, in which case the project would not be profitable. Answer (D) is incorrect because
the accounting rate of return is not based on cash flows and is irrelevant to a company's hurdle rate.
105
. Answer (D) is correct. Given unlimited funds, all projects with a net present value greater than zero should be
invested in. Thus, it would be profitable to invest in any company where the rate of return is greater than the cost of
capital.
Answer (A) is incorrect because neither the accounting rate of return nor the earnings as a percent of sales is useful in
capital budgeting. The accounting rate of return is accounting net income over the required investment; it ignores the
time value of money. Earnings as a percent of sales ignores the amount of required investment. Answer (B) is incorrect
because the payback criterion for capital budgeting is not efficient or effective. Answer (C) is incorrect because the
problem states that there are unlimited capital funds but does not indicate what the cost of capital is. Accordingly,
projects can only be invested in when the internal rate of return is greater than cost of capital, i.e., the net present value
is greater than zero.
106
. Answer (D) is correct. A company should accept any investment proposal, unless some are mutually exclusive, that
has a positive net present value or an internal rate of return greater than the company's cost of capital.
Answer (A) is incorrect because the mere availability of financing is not the only consideration; more important is the
cost of the financing, which must be less than the rate of return on the proposed investment. Answer (B) is incorrect
because an investment with positive cash flows may be a bad investment due to the time value of money; cash flows in
later years are not as valuable as those in earlier years. Answer (C) is incorrect because returns should exceed the
weighted-average cost of capital, which includes the cost of equity capital as well as the cost of debt capital.
107
. REQUIRED: The true statement about the NPV and IRR methods.
DISCUSSION: (A) The NPV criterion is that the NPV is positive, and the IRR criterion is that the cost of capital is less
than the IRR. When the cost of capital is less than the IRR, the NPV is positive. When it exceeds the IRR, the NPV is
negative. Accordingly, when two projects are independent, the NPV and IRR criteria will always lead to the same accept
or reject decision.
Answer (B) is incorrect because, if the second project’s IRR is higher than the first project’s, the organization would
accept the second project based on the IRR criterion. Answers (C) and (D) are incorrect because, if the projects are
independent, the NPV and IRR criteria indicates the same decision.
108
. Answer (A) is correct. Although the NPV method and the IRR method may rank projects differently, if a project is
found acceptable under the NPV approach, it will also be acceptable under the internal rate of return approach.
Answer (B) is incorrect because the two approaches may rank projects differently (the IRR assumes that reinvestment
will be at the discount rate, which is frequently not possible). Answer (C) is incorrect because the payback approach
does not consider the time value of money. Therefore, a project may be ranked differently than it would be under the
NPV approach or may be acceptable under the payback approach but not the NPV or IRR approaches. Answer (D) is
incorrect because the payback approach does not consider the time value of money. Therefore, a project may be ranked
differently than it would be under the NPV approach or may be acceptable under the payback approach but not the NPV
or IRR approaches.
109
. Answer (D) is correct. The profitability (excess present value) index facilitates the comparison of investments that
have different initial costs. The profitability index equals the present value of future net cash inflows divided by the initial
cash investment. The investment with the greater profitability index will be the preferred investment. However, if
investments are mutually exclusive, the net present value method may be the better way of ranking projects. The excess
present value index indicates the best return per dollar invested but does not consider the alternative possibilities for
unused funds. Thus, the smaller of the mutually exclusive projects may have the higher index, but the incremental
investment in the larger project may make it the better choice. For example, an $8,000,000 project may be a better use
of funds than a combination of a $6,000,000 project with a higher index and the best alternative use of the remaining
$2,000,000.
Answer (A) is incorrect because the investment generating cash flows the longest may not have the best return. Answer
(B) is incorrect because, given a net present value of zero (a profitability index exactly equal to one), the investor would
be indifferent to the project. Answer (C) is incorrect because the accounting rate of return is not a good measure of
profitability. It ignores the time value of money.
110
. Answer (C) is correct. Investment projects may be mutually exclusive under conditions of capital rationing (limited
capital). In other words, scarcity of resources will prevent an entity from undertaking all available profitable activities.
Under the IRR method, an interest rate is computed such that the present value of the expected future cash flows
equals the cost of the investment (NPV = 0). The IRR method assumes that the cash flows will be reinvested at the IRR.
The NPV is the excess of the present value of the estimated net cash inflows over the net cost of the investment. The
cost of capital must be specified in the NPV method. An assumption of the NPV method is that cash flows from the
investment will be reinvested at the particular project's cost of capital. Because of the difference in the assumptions
regarding the reinvestment of cash flows, the two methods will occasionally give different answers regarding the ranking
of mutually exclusive projects. Moreover, the IRR method may rank several small, short-lived projects ahead of a large
project with a lower rate of return but with a longer life span. However, the large project might return more dollars to the
company because of the larger amount invested and the longer time span over which earnings will accrue. When faced
with capital rationing, an investor will want to invest in projects that generate the most dollars in relation to the limited
resources available and the size and returns from the possible investments. Thus, the NPV method should be used
because it determines the aggregate present value for each feasible combination of projects.
Answer (A) is incorrect because the IRR is a number computed based on the characteristics of a given project. Answer
(B) is incorrect because cash flows are discounted under the IRR method. Answer (D) is incorrect because an
accelerated depreciation method will generate larger net cash inflows in the early years of a project. To equate the
present value of these cash flows with the net investment will therefore require a higher discount rate (IRR).
111
. Answer (D) is correct. The two methods ordinarily yield the same results, but differences can occur when the
duration of the projects and the initial investments differ. The reason is that the IRR method assumes cash inflows from
the early years will be reinvested at the internal rate of return. The NPV method assumes that early cash inflows are
reinvested at the cost of capital.
Answer (A) is incorrect because the two methods will give the same results if the lives and required investments are the
same. Answer (B) is incorrect because if the required rate of return equals the IRR (i.e., the cost of capital is equal to the
IRR), the two methods would yield the same decision. Answer (C) is incorrect because if the required rate of return is
higher than the IRR, both methods would yield a decision not to acquire the investment.
112
. Answer (A) is correct. Project A's NPV is calculated as follows:
$1,000 x 2.2832$2,283.20 - Original cost(1,000.00) NPV$1,283.20The second project's NPV is:
$1,500 x (3.7845 - 2.2832) $2,251.95 - Original cost(1,000.00) NPV$1,251.95Since A has a slightly higher NPV, it
should be selected.
Answer (B) is incorrect because Project A has a slightly higher NPV and IRR. Answer (C) is incorrect because Project A
has a slightly higher IRR. Answer (D) is incorrect because Project A has a slightly higher NPV.
113
. Answer (D) is correct. The profitability index (PI) is often used to decide among investment alternatives when more
than one is acceptable. The profitability index is the ratio of the present value of future net cash inflows to the initial net
cash investment. The PI, although a variation of the net present value method, facilitates comparison of different-sized
investments.
Answer (A) is incorrect because the accounting rate of return is a poor technique. It ignores the time value of money.
Answer (B) is incorrect because the payback method ignores the time value of money and long-term profitability.
Answer (C) is incorrect because the internal rate of return is not effective when alternative investments have different
lives.
114
. Answer (A) is correct. The profitability index is the ratio of the present value of future net cash inflows to the initial
cash investment; that is, the figures are those used to calculate the net present value (NPV), but the numbers are
divided rather than subtracted. This variation of the NPV method facilitates comparison of different-sized investments. It
provides an optimal ranking in the absence of capital rationing.
Answer (B) is incorrect because the profitability index method is a discounted cash flow method. Answer (C) is incorrect
because the payback method gives no consideration to the time value of money or to returns after the payback period.
Answer (D) is incorrect because the profitability index method and the NPV method are discounted cash flow methods.
However, the profitability index method is the variant that purports to calculate a return per dollar of investment.
115
. Answer (C) is correct. The profitability index is the ratio of the present value of future net cash inflows to the initial
cash investment. This variation of the net present value method facilitates comparison of different-sized investments.
Were it not for this comparison feature, the profitability index would be no better than the net present value method.
Thus, it is the comparison, or ranking, advantage that makes the profitability index different from the other capital
budgeting tools.
Answer (A) is incorrect because the net present value (NPV > 0) is a capital budgeting tool that screens investments;
i.e., the investment must meet a certain standard to be acceptable. Answer (B) is incorrect because the time-adjusted
rate of return is a capital budgeting tool that screens investments; i.e., the investment must meet a certain standard (rate
of return) to be acceptable. Answer (D) is incorrect because the accounting rate of return is a capital budgeting tool that
screens investments; i.e., the investment must meet a certain standard (rate of return) to be acceptable.
116
. Answer (B) is correct. The IRR is the discount rate at which the net present value of a project is zero. Consequently,
if the IRR exceeds the cost of capital, the NPV calculated at the cost of capital must be positive. Projects with a positive
NPV are expected to be profitable and should be considered. Other factors being equal, projects with higher IRRs
should be accepted before those with lower IRRs.
Answer (A) is incorrect because IRRs should exceed the cost of capital, and projects should be accepted in the
descending order of their IRRs. Answer (C) is incorrect because IRRs should exceed the cost of capital, and projects
should be accepted in the descending order of their IRRs. Answer (D) is incorrect because IRRs should exceed the cost
of capital, and projects should be accepted in the descending order of their IRRs.
117
. Answer (C) is correct. Rational investors choose projects that yield the best return given some level of risk. If an
investor desires no risk, that is, an absolutely certain rate of return, the risk-free rate is used in calculating net present
value. The risk-free rate is the return on a risk-free investment such as government bonds. Certainty equivalent
adjustments involve a technique directly drawn from utility theory. It forces the decision maker to specify at what point
the firm is indifferent to the choice between a sum of money that is certain and the expected value of a risky sum.
Answer (A) is incorrect because a risk-adjusted discount rate does not represent an absolutely certain rate of return. A
discount rate is adjusted upward as the investment becomes riskier. Answer (B) is incorrect because the cost of capital
has nothing to do with certainty equivalence. Answer (D) is incorrect because the cost of equity capital does not equate
to a certainty equivalent rate.
118
. Answer (D) is correct. Under the certainty-equivalent method, expected cash flows are multiplied by a certainty
equivalent factor and discounted at the risk-free rate. Under the risk-adjusted discount rate method, expected cash flows
are discounted at the risk-adjusted discount rate.
Answer (A) is incorrect because the certainty-equivalent method uses the risk-free rate, not the cost of capital. Answer
(B) is incorrect because the risk-adjusted discount rate discounts expected cash flows at the risk-adjusted rate. Answer
(C) is incorrect because the certainty-equivalent method uses the risk-free rate, not the cost of capital.
119
. Answer (D) is correct. Under the certainty-equivalent approach, expected cash flows should be multiplied by
certainty-equivalent factors and discounted at the risk-free rate.
Answer (A) is incorrect because the risk-free rate should be used rather than the cost of capital. Answer (B) is incorrect
because the risk-free rate should be used rather than the cost of capital. Answer (C) is incorrect because the risk-free
rate should be used rather than the cost of capital.
120
. Answer (C) is correct. Uncertainty can be compensated for by adjusting the desired rate of return. If projects have
relatively uncertain returns, a higher rate should be required. A lower rate of return may be acceptable given greater
certainty. The concept is that with increased risk should come increased rewards, i.e., a higher rate of return.
Answer (A) is incorrect because preparing an analysis of probability of outcomes is not a simple method of adjustment.
Answer (B) is incorrect because accelerated depreciation should probably be used for tax purposes in every capital
project to minimize taxes in early years. Answer (D) is incorrect because uniformly increasing the estimated cash flows
and/or ignoring salvage values introduces error into the capital budgeting analysis.
121
. Answer (D) is correct. Risk analysis attempts to measure the likelihood of the variability of future returns from the
proposed investment. Risk can be incorporated into capital budgeting decisions in a number of ways, one of which is to
use a hurdle rate higher than the firm's cost of capital, that is, a risk-adjusted discount rate. This technique adjusts the
interest rate used for discounting upward as an investment becomes riskier. The expected flow from the investment
must be relatively larger or the increased discount rate will generate a negative net present value, and the proposed
acquisition will be rejected.
Answer (A) is incorrect because the nature of the funding may not be a sufficient reason to use a risk-adjusted rate. The
type of funding is just one factor affecting the risk of a project. Answer (B) is incorrect because a higher hurdle will result
in rejection of more projects. Answer (C) is incorrect because a risk-adjusted high hurdle rate is used for capital
investments with greater risk.
122
. Answer (D) is correct. Risk-adjusted discount rates can be used to evaluate capital investment options. If risks differ
among various elements of the cash flows, then different discount rates can be used for different flows.
Answer (A) is incorrect because the payback period ignores both the varying risk and the time value of money. Answer
(B) is incorrect because using the cost of capital as the discount rate does not make any adjustment for the risk
differentials among the various cash flows. Answer (C) is incorrect because risk has to be incorporated into the
company's hurdle rate to use the internal rate of return method with risk differentials.
123
. Risk adjustment Answer: b Diff: E
124
. Risk and project selection Answer: b Diff: E
125
. Accepting risky projects Answer: e Diff: E
126
. Risk and project selection Answer: c Diff: E N
You have to find out what the required rate of return on each project is. Projects that are of high risk must have a
higher required rate of return than projects that are of low risk. The following table shows the required return for
each project on the basis of its risk level.

Required return for


Project Expected Return Risk the risk level
A 15% High 12%
B 12 Average 10%
C 11 High 12%
D 9 Low 8%
E 6 Low 8%

The company will accept all projects whose expected return exceeds its required return. Therefore, it will accept
Projects A, B, and D.

127
. Answer (D) is correct. Risk analysis attempts to measure the likelihood of the variability of future returns from the
proposed investment. Risk can be incorporated into capital budgeting decisions in a number of ways, one of which is to
use a hurdle rate higher than the firm's cost of capital, that is, a risk-adjusted discount rate. This technique adjusts the
interest rate used for discounting upward as an investment becomes riskier. The expected flow from the investment
must be relatively larger, or the increased discount rate will generate a negative net present value, and the proposed
acquisition will be rejected. Accordingly, the IRR (the rate at which the NPV is zero) for a rejected investment may
exceed the cost of capital when the risk-adjusted rate is higher than the IRR. Conversely, the IRR for an accepted
investment may be less than the cost of capital when the risk-adjusted rate is less than the IRR. In this case, the
investment presumably has very little risk. Furthermore, risk-adjusted rates may also reflect the differing degrees of risk,
not only among investments, but by the same investments undertaken by different organizational subunits.
Answer (A) is incorrect because discount rates may vary with the project or with the subunit of the organization. Answer
(B) is incorrect because the company may accept some projects with IRRs less than the cost of capital, or reject some
project with IRRs greater than the cost of capital. Answer (C) is incorrect because the company may accept some
projects with IRRs less than the cost of capital, or reject some project with IRRs greater than the cost of capital.
128
. By Kemp not making the risk adjustment, it is true that the company will accept more projects in the computer
division, and fewer projects in the restaurant division. However, this will make the company riskier overall, raising its cost
of equity. Investors will discount their cash flows at a higher rate, and the company’s value will fall. In addition, some of
the computer projects might not exceed the appropriate risk-adjusted hurdle rate, and will actually be negative NPV
projects, further destroying value. Therefore, statement a is false. Because fewer of the restaurant projects will be
accepted, the restaurant division will become a smaller part of the overall company. Therefore, statement b is false. As
explained above, statement c is true.
129
. By not risk adjusting the cost of capital, the firm will tend to reject low-risk projects since their returns will be lower
than the average cost of capital, and it will take on high-risk projects since their returns will be higher than the average
cost of capital.
130
. Risk adjustment Answer: a Diff: M
kA = 13% - 3% = 10%.
If the cash flows are cost only outflows, and the analyst wants to correctly
reflect their risk, the discount rate should be adjusted downward (in this
case by subtracting 3 percentage points) to make the discounted flows
comparatively larger.
131
. Risk analysis Answer: e Diff: E
132
. Methods of analysis Answer: a Diff: M
133
. REQUIRED: he technique used to evaluate cash flows from the purchase of a machine.
DISCUSSION: (A) Simulation is a technique used to describe the behavior of a real-world system over time. This
technique usually employs a computer program to perform the simulation computations. Sensitivity analysis examines
how outcomes change as the model parameters change.
Answer (B) is incorrect because linear programming is a mathematical technique for optimizing a given objective
function subject to certain constraints. Answer (C) is incorrect because correlation analysis is a statistical procedure for
studying the relation between variables. Answer (D) is incorrect because differential analysis is used for decision
making that compares differences in costs (revenues) of two or more options.
134
. Answer (C) is correct. Capital budgeting is the process of planning expenditures for assets, the returns on which are
expected to continue beyond 1 year. Simulation (Monte Carlo simulation) as applied to capital budgeting is a technique
for experimenting with logical/mathematical models using a computer. The computer is used to generate many
examples of results based upon various assumptions.
Answer (A) is incorrect because this is a method used to rank projects for capital-budgeting decision purposes. Answer
(B) is incorrect because this is a method used to rank projects for capital-budgeting decision purposes. Answer (D) is
incorrect because this is a method used to rank projects for capital-budgeting decision purposes.
135
. Answer (B) is correct. The term "rates of return" suggests the net present value and internal rate of return methods
in this context, but simulation analysis can also be used. Simulation is a mathematical modeling technique for
performing a what-if analysis of estimated data. For instance, it may determine how profitable a company will be if
Machine A is purchased rather than Machine B.
Answer (A) is incorrect because regression analysis is based on past data and is often used to determine trends or
divide costs into their fixed and variable components. Answer (C) is incorrect because Markov chain analysis is used in
decision problems in which the probability of the occurrence of a future state depends only on the current state. A
characteristic of the Markov process is that the initial state matters less and less as times goes on, because the process
will eventually reach its steady state. Answer (D) is incorrect because Gantt charting involves drawing a bar chart
showing the progress of a project.
136
. Sensitivity, scenario, and simulation analyses Answer: c Diff: E N

Statement a is false; sensitivity analysis measures a project’s stand-alone risk. Statement b is false; sensitivity
analysis doesn’t take into account probabilities, while scenario analysis does. Statement c is correct.

137
. Answer (C) is correct. After a problem has been formulated into any mathematical model, it may be subjected to
sensitivity analysis, which is a trial-and-error method used to determine the sensitivity of the estimates used. For
example, forecasts of many calculated NPVs under various assumptions may be compared to determine how sensitive
the NPV is to changing conditions. Changing the assumptions about a certain variable or group of variables may
drastically alter the NPV, suggesting that the risk of the investment may be excessive.
Answer (A) is incorrect because sensitivity analysis is useful when cash flows, or other assumptions, are uncertain.
Answer (B) is incorrect because sensitivity analysis can be used with any of the capital budgeting methods. Answer (D)
is incorrect because sensitivity analysis is not a ranking technique; it calculates results under varying assumptions.
138
. Answer (A) is correct. Sensitivity analysis is a technique to evaluate a model in terms of the effect of changing the
values of the parameters. It answers "what if" questions. In capital budgeting models, sensitivity analysis is the
examination of alternative outcomes under different assumptions.
Answer (B) is incorrect because probability (risk) analysis is used to examine the array of possible outcomes given
alternative parameters. Answer (C) is incorrect because cost behavior (variance) analysis concerns historical costs, not
predictions of future cash inflows and outflows. Answer (D) is incorrect because ROI analysis is appropriate for
determining the profitability of a company, segment, etc.
139
. Answer (B) is correct. After a problem has been formulated into any mathematical model, it may be subjected to
sensitivity analysis, which is a trial-and-error method used to determine the sensitivity of the estimates used. For
example, forecasts of many calculated NPVs under various assumptions may be compared to determine how sensitive
the NPV is to changing conditions. Changing the assumptions about a certain variable or group of variables may
drastically alter the NPV, suggesting that the risk of the investment may be excessive.
Answer (A) is incorrect because sensitivity analysis is a means of making several estimates of inputs into a capital
budgeting decision to determine the effect of changes in assumptions. Answer (C) is incorrect because sensitivity
analysis is not a simulation technique; it is simply a process of making more than one estimate of unknown variables.
Answer (D) is incorrect because sensitivity analysis would not identify the required market share; instead, it would be
used to make several estimates of market share to determine how sensitive the decision is to changes in market share.
140
. Monte Carlo simulation Answer: e Diff: M
141
. Answer (D) is correct. An increase in the discount rate would lower the net present value, as would a decrease in
cash flows or an increase in the initial investment.
Answer (A) is incorrect because a decrease in the tax rate would decrease tax expense, thus increasing cash flows and
the NPV. Answer (B) is incorrect because a decrease in the initial investment amount would increase the NPV. Answer
(C) is incorrect because an extension of the project life and associated cash inflows would increase the NPV.
142
12A-. NPV and depreciation Answer: c Diff: E
143
.Statements a, b, c, and d are false. Statement e is correct because you can think of a firm as a big project. If the stock
is correctly priced, i.e., the stock market is efficient, the NPV of this project should be zero.
144
. REQUIRED: The true statement regarding the NPV profiles of two mutually exclusive capital projects.
DISCUSSION: (A) The NPV is the excess of the present value of the estimated cash inflows over the net cost of the
investment. Thus, Project 2 has a higher internal rate of return. The internal rate of return is the cost of capital at which
the NPV is zero, that is, the cost of capital at which the NPV profile crosses the horizontal axis. The NPV profile for
Project 2 intersects the horizontal axis at a higher cost of capital percentage than does that for Project 1.
Answer (B) is incorrect because Project 1 has the lower internal rate of return. Answers (C) and (D) are incorrect
because the profiles of Projects 1 and 2 intersect. Neither project will have a higher NPV at all cost of capital
percentages. To the left of the intersection point, Project 1 has a higher NPV. To the right of the intersection point,
Project 2 has a higher NPV.
145
.You can draw the NPV profiles to get an idea of what is happening. (See the diagram below.) Statement a is false;
Project B could have a higher NPV at some WACC if the NPV profiles cross. Statement b is false; Project B could have
a negative NPV when A’s NPV is positive. Statement c is false; the IRR is unaffected by the WACC. Statement d is the
correct choice.
NPV
$

B
A

k
0 10% IRRB IRRA

146
.
NPV
($)

Discount rate (%)


0 16% 17% 18% 30%

Draw the NPV profiles using the information given in the problem. It is clear that Project A will have a higher NPV when
the cost of capital is 12 percent. Therefore, statement a is false. At a 17 percent cost of capital, Project B will have a
higher NPV than Project A. Therefore, statement b is true. If the cost of capital were 0, then the NPV of the projects
would be the simple sum of all the cash flows. In order for statement c to be correct, B’s NPV at a 0 cost of capital would
have to be higher than A’s. From the diagram we see that this is clearly incorrect. So, statement c is false.
147
. NPV profiles

Answer: d Diff: M N

NPV
$

k
10% 15% 20% %

The diagram above can be drawn from the statements in this question. From the diagram drawn, statements a, b, and c
are correct; therefore, statement d is the correct choice.
148
.

NPV ($)

10% k

First, draw the NPV profiles as shown above. Make sure the profiles cross at 10 percent because the projects have the
same NPV at a cost of capital of 10 percent. When WACC is less than 10 percent, C has a higher NPV, so C’s NPV
profile is above D’s NPV profile to the left of the crossover point (10%).

Statement a is true. IRR is always independent of the cost of capital, and from the diagram above, we can see that
D’s IRR is to the right of C’s where the two lines cross the X-axis. Statement b is false. IRR is independent of the
cost of capital, and from the diagram C’s IRR is always lower than D’s. Statement c is true. D’s MIRR will be
somewhere between the cost of capital and the IRR. Therefore, the correct choice is statement d.
149
.
NPV

k
0 7% 12% 15%

Since both projects have an IRR greater than the cost of capital, both will have a positive NPV. Therefore, statement a is
true. At 6 percent, the cost of capital is less than the crossover rate and Project A has a higher NPV than B. Therefore,
statement b is false. If the cost of capital is 13 percent, then the cost of capital is greater than the crossover rate and B
would have a higher NPV than A. Therefore, statement c is true. Since statements a and c are both true, the correct
choice is statement e.
150
.
NPV
$

k
0 7% 12% 14%

Statement a is correct, because at any point to the right of the crossover point B will have a higher NPV than A.
Statement b is correct for the same reason that statement a is true; at any point to the right of the crossover point, B will
have a higher NPV than A. Statement c is correct. If B’s cost of capital is 9 percent, when MIRR is calculated the cash
flows are being reinvested at 9 percent. When IRR is used, the IRR calculation assumes that cash flows are being
reinvested at the IRR (which is higher than the cost of capital.) Statement e is the correct choice.
151
. REQUIRED: The true statement about the IRR.
DISCUSSION: (B) the IRR is the discount rate at which the net present value is zero. Because the present value of a
dollar is higher the sooner it is received, projects with later cash flows will have lower net present values for any given
discount rate than will projects with earlier cash flows, if other factors are constant. Hence, projects with later cash flows
will have a lower IRR.
Answer (A) is incorrect because the present value of the cash inflows is inversely related to the discount rate, that is, if
the discount rate is higher, the present value of the cash inflows is lower. If the investment cost is lower, a higher
discount rate (the IRR) will be required to set the net present value equal to zero. Answer (C) is incorrect because the
larger the cash inflows, the higher the IRR will be. Higher cash inflows have a higher present value at any given
discount rate. A higher discount rate will be required to set the net present value equal to zero. Answer (D) is incorrect
because projects with shorter payback periods have higher cash inflow early in the life of the project. Projects with
earlier cash inflows have the higher IRRs.
152
. Answer (A) is correct. Investments with present values in excess of their costs (positive NPVs) that can be identified
or created by the capital budgeting activities of the firm will have a positive impact on firm value and on the price of the
common shares of the firm. Accordingly, the more effective capital budgeting is, the higher the share price.
Answer (B) is incorrect because positive NPV investments will increase, not decrease, firm value and share price.
Answer (C) is incorrect because investments with present values equal to their costs have a zero NPV and neither
increase nor decrease firm value and share price. Answer (D) is incorrect because investments with present values
equal to their costs have a zero NPV and neither increase nor decrease firm value and share price.
153
. Answer (A) is correct. The value of the firm is the present value of the expected cash flows, which is given by the
following expression:
n
Sigma CFt
t 1

(1  K) t
If CF is net cash flow, K is the discount rate (cost of capital), and t is time, value will rise as CF increases.
Answer (B) is incorrect because an increase in systematic (market or undiversifiable) risk will increase the overall cost of
capital and thereby increase K, the discount rate. As a result, the value of the firm will fall. Answer (C) is incorrect
because an increase in unsystematic (or firm-specific) risk will have no effect on the value of the firm. The total risk is the
sum of systematic and unsystematic risk. By definition, the latter is the risk that can be eliminated by diversification.
Answer (D) is incorrect because an increase in the discount rate will reduce the value of the firm.
154
. Risk analysis Answer: c Diff: E N

Statement a is false. Stand-alone risk is measured by standard deviation. Therefore, since Y’s standard deviation is
higher than X’s, Y has higher stand-alone risk than X. Statement b is false. Corporate risk is measured by the
correlation of project cash flows with other company cash flows. Therefore, since Y’s cash flows are highly
correlated with the cash flows of existing projects, while X’s are not, Y has higher corporate risk than X. Market
risk is measured by beta. Therefore, since X’s beta is greater than Y’s, statement c is true.

155
. Statement a is correct. The IRR’s of both exceed the cost of capital. Statement b is incorrect. We
cannot determine this without knowing the NPV’s of the projects. Statement c is correct. To see why draw
the NPV profiles. Statement d is incorrect. Therefore, statement e is the correct answer.
156
. This is the only project with either a positive NPV or an IRR which exceeds the cost of capital.
157
. Draw out the NPV profiles of these two projects. As B’s NPV declines more rapidly with an increase in
discount rates, this implies that more of the cash flows are coming later on. Therefore, Project A has a
faster payback than Project B.
158
.
NPV
$

X
Crossover

k
10% IRRY 12% IRRX

If IRRX is greater than MIRRX, then its IRR must be higher than the cost of capital. (Remember that the MIRR will be
somewhere between the cost of capital and the IRR). Therefore, statement a must be true. Similarly, if IRR Y is less than
MIRRY, then its IRR must be lower than the cost of capital. Therefore, statement b must be true. At a cost of capital of 10
percent they have the same NPV, so this is the crossover rate. From statements a and b we know that IRR X must be
greater than IRRY, so to the right of the crossover rate NPV X will be larger than NPVY. Consequently, to the left of the
crossover rate NPVX must be smaller than NPVY. Therefore, statement c is also true. Since statements a, b, and c are all
true, the correct choice is statement d.
159
. This statement reflects exactly the difference between the NPV and IRR methods.
160
. Both statements a and c are correct; therefore, statement d is the correct choice. Due to reinvestment
rate assumptions, NPV and IRR can lead to conflicts; however, there will be no conflict between NPV and
MIRR if the projects are equal in size (which is one of the assumptions in this question).
161
. Statement a is true. To see this, sketch out a NPV profile for a normal, independent project, which
means that only one NPV profile will appear on the graph. If WACC < IRR, then IRR says accept. But in that
case, NPV > 0, so NPV will also say accept. Statement d is false. Here is the reasoning:
1. For the NPV profiles to cross, then one project must have a higher NPV at k = 0 than the other project, that is, their
vertical axis intercepts will be different.
2. A second condition for NPV profiles to cross is that one have a higher IRR than the other.
3. The third condition necessary for profiles to cross is that the project with the higher NPV at k = 0 will have the lower
IRR.
One can sketch out two NPV profiles on a graph to see that these three conditions are indeed required.
4. The project with the higher NPV at k = 0 must have more cash inflows, because it has the higher NPV when cash
flows are not discounted, which is the situation if k = 0.
5. If the project with more total cash inflows also had its cash flows come in earlier, it would dominate the other
project--its NPV would be higher at all discount rates, and its IRR would also be higher, so the profiles would not
cross. The only way the profiles can cross is for the project with more total cash inflows to get a relatively high
percentage of those inflows in distant years, so that their PVs are low when discounted at high rates. Most students
either grasp this intuitively or else just guess at the question!
162
. Answer (A) is correct. If the economic results of alternative capital investments were known with certainty or with
minimal risk, the quantitative analyses would reveal the absolute best investment options. However, if the economic
results are highly uncertain, using a decision-making process that combines rational analysis with intuition is
appropriate. Moreover, nonquantifiable variables may be involved.
Answer (B) is incorrect because the decision-making process described combines rational quantitative analysis with
intuition. In addition, research has shown that intuition can improve decision making. Answer (C) is incorrect because
knowing with certainty which investment is the most profitable is not possible. Answer (D) is incorrect because the term
bounded rationality refers to the inability to perceive all aspects of a situation and the tendency to simplify, not to intuitive
decision making.
163
. Statement e is correct; the other statements are false. IRR can lead to conflicting decisions with NPV
even with normal cash flows if the projects are mutually exclusive. Cash outflows are discounted at the cost
of capital with the MIRR method, while cash inflows are compounded at the cost of capital. Conflicts
between NPV and IRR arise when the cost of capital is below the crossover point. The discounted payback
method does correct the problem of ignoring the time value of money, but it still does not account for cash
flows beyond the payback period.
164
. Statements a and c are correct; therefore, statement d is the correct choice. The discounted payback
method still ignores cash flows after the payback period.
165
. Statement a is correct; the other statements are false. Multiple IRRs can occur only for projects with
nonnormal cash flows. Mutually exclusive projects imply that only one project should be chosen. The project
with the highest NPV should be chosen.
166
. Statement a is correct; the other statements are false. Sketch the profiles. From the information given,
D has the higher IRR. The project’s scale cannot be determined from the information given. As C’s NPV
declines more rapidly with an increase in rates, this implies that more of the cash flows are coming later on.
So C would have a slower payback than D.
167
. Answer (D) is correct. MACRS is an accelerated method of depreciation under which depreciation expense will be
greater during the early years of an asset's life. Thus, the outflows for income taxes will be less in the early years, but
greater in the later years, and the NPV (present value of net cash inflows - investment) will be increased. The profitability
index (present value of net cash inflows ÷ the investment) must increase if the NPV increases.
Answer (A) is incorrect because the NPV will increase. The present value of the net inflows will increase with no change
in the investment. Answer (B) is incorrect because the IRR will increase. Deferring expenses to later years increases the
discount rate needed to reduce the NPV to $0. Answer (C) is incorrect because the payback period will be shortened.
Switching to MACRS defers expenses and increases cash flows early in the project's life.
168
12A-. Depreciation cash flows Answer: c Diff: M
169
. Answer (D) is correct. MACRS for assets with lives of 10 years or less is based on the 200% declining-balance
method of depreciation. Thus, an asset with a 3-year life would have a straight-line rate of 33-1/3%, or a double-
declining-balance rate of 66-2/3%.
Answer (A) is incorrect because the straight-line method uses the same percentage each year during an asset's life, but
MACRS uses various percentages. Answer (B) is incorrect because MACRS is unrelated to the units-of-production
method. Answer (C) is incorrect because MACRS is unrelated to SYD depreciation.
170
. Answer (A) is correct. For tax purposes, straight-line depreciation is an alternative to the MACRS method. Both
methods will result in the same total depreciation over the life of the asset; however, MACRS will result in greater
depreciation in the early years of the asset's life because it is an accelerated method. Given that MACRS results in
larger depreciation deductions in the early years, taxes will be lower in the early years and higher in the later years.
Because the incremental benefits will be discounted over a shorter period than the incremental depreciation costs,
MACRS is preferable to the straight-line method.
Answer (B) is incorrect because both methods will produce the same total depreciation over the life of the asset. Answer
(C) is incorrect because both methods will produce the same total tax payments (assuming rates do not change).
However, given that the tax payments will be lower in the early years under MACRS, discounting for the time value of
money makes the straight-line alternative less advantageous. Answer (D) is incorrect because both methods will
produce the same total depreciation over the life of the asset.
171
. Taxes on gain on sale

Answer: b Diff: E N

When the machine is sold the total accumulated depreciation on it is: (0.2 + 0.32 + 0.19)  $1,000,000 = $710,000.
The book value of the equipment is: $1,000,000 - $710,000 = $290,000. The machine is sold for $400,000, so the
gain is $400,000 - $290,000 = $110,000. Taxes are calculated as $110,000  0.4 = $44,000.
172

. Risk-adjusted discount rate Answer: c Diff: E


ks = 10% + (16% - 10%)1.5 = 10% + 9% = 19%.
Original IRR = 21%. 21% - Risk adjustment 1% = 20%.
Risk adjusted IRR = 20% > ks = 19%.
173

. Risk-adjusted discount rate Answer: b Diff: M


Time lines:
Project A:
0 k = 12% 1 2 3 4 5 6

CFsA -15,000 4,000 4,000 4,000 4,000 4,000 4,000


NPVA = ? = 3,978.60 5,000 Salvage value
Terminal CF = 9,000
Project B:
0 k = ? 1 2 6 Years
  
CFsB -14,815 5,100 5,100 5,100
NPVB = NPVA = 3,978.60 0 Salvage value
Terminal CF = 5,100

Tabular solution:
Solve for the NPV of Project A, which is also the NPV of Project B at some k
= ?
NPVA = -$15,000 + $4,000(PVIFA12%,6) + $5,000(PVIF12%,6)
= -$15,000 + $4,000(4.1114) + $5,000(0.5066) = $3,978.60.
Solve for kB
NPVB = $3,978.60 = -$14,815 + $5,100(PVIFAk,6)
$18,793.60 = $5,100(PVIFAk,6)
PVIFAk,6 = 3.68502.
Look across the row for 6 years in the PVIFA table. The factor for 16 percent
is 3.6847; therefore, the risk-adjusted rate for Project B is approximately
16 percent.

Financial calculator solution:


A: Inputs: CF0 = -15,000; CF1 = 4,000; Nj = 5; CF2 = 9,000; I = 12.
Output: NPV = $3,978.78.
B: Inputs: CF0 = -18,793.78; CF1 = 5,100; Nj = 6.
Output: IRR = 15.997%  16%.
174
. Risk-adjusted discount rate Answer: e Diff: T
Time lines:
Project A:
0 1 2 3 4 Years
k = 12%

CFsA -25,000 13,000 13,000 13,000 13,000


NPVA = ? = 17,663 Terminal value = 5,000
CF4 = 18,000
Project B:
0 1 2 3 4 Years
k = ?

CFsB -25,000 15,247 15,247 15,247 15,247


NPVA = NPVB = 17,663 Terminal value = 0
CF4 = 15,247
Tabular solution:
NPVA = -$25,000 + $13,000(PVIFA12%,3) + $18,000(PVIF12%,4)
= -$25,000 + $13,000(2.4018) + $18,000(0.6355) = $17,662.40.
NPVB = $17,662.40 = -$25,000 + $15,247(PVIFAk,4)
$42,662.40 = $15,247(PVIFAk,4)
(PVIFAk,4) = 2.79808
k  16%.

Financial calculator solution:


A: Inputs: CF0 = -25,000; CF1 = 13,000; Nj = 3; CF2 = 18,000; I = 12.
Output: NPVA = 17,663.13.
B: Inputs: CF0 = -42,663.13; CF1 = 15,247; Nj = 4.
Output: IRR = 16.0% = k.
175

. New project NPVAnswer: e Diff: M


Time line:
0 k = 18% 1 2 3 Years

-3,000 1,728 1,920 1,152


NPV = ?

Project analysis worksheet:


I Initial outlay
1) Cost ($4,000)
2) Decrease in NOWC 1,000
3) Total net investment ($3,000)
II Operating flows: Year: 0 1 2 3
4) EBT and depreciation $2,000 $2,000 $2,000
5) Earnings after taxes
(line 4  0.6) 1,200 1,200 1,200
6) Depreciation (from table) 1,320 1,800 600
7) Tax savings from
depreciation (5  0.4) 528 720 240
8) Net operating flows
(line 5 + 7) $1,728 $1,920 $1,440
III Terminal year cash flows:
9) Estimated salvage value $1,000
10) Tax on salvage value
((1,000 - 280)  0.4) (288)
11) Return of NOWC (1,000)
12) Total termination CFs ($ 288)
IV Net cash flows:
13) Total net cash flows ($3,000) $1,728 $1,920 $1,152

Tabular solution:
NPV = -$3,000 + $1,728(PVIF18%,1) + $1,920(PVIF18%,2) + $1,152(PVIF18%,3)
= -$3,000 + $1,728(0.8475) + $1,920(0.7182) + $1,152(0.6086) = $544.53.

Financial calculator solution:


Inputs: CF0 = -3000; CF1 = 1728; CF2 = 1920; CF3 = 1152; I = 18.
Output: NPV = $544.46  $544.
176
. New project NPV Answer: d Diff: M
Time line:
0 1 2 3 4 5 Years
k = 12%

-45,000 7,800 10,680 7,560 5,880 -1,920


NPV = ?

Depreciation cash flows:


MACRS Depreciable Annual
Year Percent Basis Depreciation
1 0.20 $60,000 $12,000
2 0.32 60,000 19,200
3 0.19 60,000 11,400
4 0.12 60,000 7,200
5 0.11 60,000 6,600
6 0.06 60,000 3,600
$60,000

Project analysis worksheet:


I Initial outlay
1) Machine cost ($60,000)
2) Decrease in NOWC 15,000
3) Total net inv. ($45,000)
II Operating cash flows
Year: 0 1 2 3 4 5
4) Reduction in cost $ 5,000 $ 5,000 $ 5,000 $ 5,000 $ 5,000
5) After-tax dec. in cost 3,000 3,000 3,000 3,000 3,000
6) Deprec. (from table) 12,000 19,200 11,400 7,200 6,600
7) Tax savings deprec.
(line 6  0.4) 4,800 7,680 4,560 2,880 2,640
8) Net operating CFs
(line 5 + 7) $ 7,800 $10,680 $ 7,560 $ 5,880 $ 5,640
III Terminal year CFs
9) Estimated salvage value $10,000
10) Tax on salvage value
(10,000 - 3,600)(0.4) (2,560)
11) Return of NOWC (15,000)
12) Total termination CFs (7,560)
IV Net CFs
13) Total Net CFs ($45,000) $ 7,800 $10,680 $ 7,560 $ 5,880 ($ 1,920)

Tabular solution:
NPV = -$45,000 + $7,800(PVIF12%,1) + $10,680(PVIF12%,2) + $7,560(PVIF12%,3)
+ $5,880(PVIF12%,4) - $1,920(PVIF12%,5)
= -$45,000 + $7,800(0.8929) + $10,680(0.7972) + $7,560(0.7118)
+ $5,880(0.6355) - $1,920(0.5674) = -$21,492.74  -$21,493.
Financial calculator solution:
Inputs: CF0 = -45,000; CF1 = 7,800; CF2 = 10,680; CF3 = 7,560;
CF4 = 5,880; CF5 = -1,920; I = 12.
Output: NPV = -$21,493.24  $21,493.
177

. New project NPVAnswer: b Diff: M


Time line:
0 k = 9% 1 2 3 4 5 Years

-40,000 9,800 11,720 9,640 8,520 15,320 NPV


= ?

Depreciation cash flows:


MACRS Depreciable Annual
Year Percent Basis Depreciation
1 0.20 $40,000 $ 8,000
2 0.32 40,000 12,800
3 0.19 40,000 7,600
4 0.12 40,000 4,800
5 0.11 40,000 4,400
6 0.06 40,000 2,400
$40,000
Project analysis worksheet:
I Initial outlay
1) Machine cost ($40,000)
2) Decrease in NOWC --
3) Total net inv. ($40,000)
II Operating cash flows
Year: 0 1 2 3 4 5
4) Inc. in earnings
before deprec. & tax $ 6,000 $ 6,000 $ 6,000 $ 6,000 $ 6,000
5) After-tax increase in
earnings (line 4  0.6) 3,600 3,600 3,600 3,600 3,600
6) Before tax reduction
in cost 5,000 5,000 5,000 5,000 5,000
7) After tax reduction
in cost (line 6  0.4) 3,000 3,000 3,000 3,000 3,000
8) Deprec. (from table) 8,000 12,800 7,600 4,800 4,400
9) Deprec. tax savings
(line 8  0.4) 3,200 5,120 3,040 1,920 1,760
10) Net operating CFs _______ _______ _______ _______ _______
(line 5 + 7 + 9) $ 9,800 $11,720 $ 9,640 $ 8,520 $ 8,360
III Terminal year CFs
11) Estimated salvage value $10,000
12) Tax on salvage value
(10,000 - 2,400)(0.4) (3,040)
13) Return of NOWC --
14) Total termination CFs 6,960
IV Net CFs
15) Total Net CFs ($40,000) $ 9,800 $11,720 $ 9,640 $ 8,520 $15,320

Tabular solution:
NPV = -$40,000 + $9,800(PVIF9%,1) + $11,720(PVIF9%,2) + $9,640(PVIF9%,3)
+ $8,520(PVIF9%,4) + $15,320(PVIF9%,5)
= -$40,000 + $9,800(0.9174) + $11,720(0.8417) + $9,640(0.7722)
+ $8,520(0.7084) + $15,320(0.6499) = $2,291.29  $2,292.

Financial calculator solution:


Inputs: CF0 = -40,000; CF1 = 9,800; CF2 = 11,720; CF3 = 9,640;
CF4 = 8,520; CF5 = 15,320; I = 9.
Output: NPV = $2,291.90  $2,292.
178
. New project NPV Answer: a Diff: M N

0 1 2 3
Equipment purchase -$600,000
NOWC -50,000

Sales increase $2,000,000 $2,000,000 $2,000,000


Operating costs 1,400,000 1,400,000 1,400,000
Operating income $ 600,000 $ 600,000 $ 600,000
Depreciation 200,000 200,000 200,000
EBIT $ 400,000 $ 400,000 $ 400,000
Taxes (35%) 140,000 140,000 140,000
EBIT(1 - T) $ 260,000 $ 260,000 $ 260,000
+Depreciation 200,000 200,000 200,000
Operating cash flow $ 460,000 $ 460,000 $ 460,000
Recovery of NOWC 50,000
Equipment sale +100,000
Taxes on sale _________ __________ __________ -35,000
Net CF -$650,000 $ 460,000 $ 460,000 $ 575,000

NPV = -$650,000 + $460,000/1.12 + $460,000/(1.12)2 + $575,000/(1.12)3


= -$650,000 + $410,714.29 + $366,709.18 + $409,273.64
= $536,697.11  $536,697.
179

. New project NPV Answer: b Diff: M N

Year 0 1 2 3 4
Project cost -5,000,000
NOWC* -300,000

Sales $3,000,000 $4,000,000 $5,000,000 $2,000,000


Operating costs (75%) 2,250,000 3,000,000 3,750,000 1,500,000
Operating income before deprec. $ 750,000 $1,000,000 $1,250,000 $ 500,000
Depreciation 1,650,000 2,250,000 750,000 350,000
EBIT -$ 900,000 -$1,250,000 $ 500,000 $ 150,000
Taxes (40%) -360,000 -500,000 200,000 60,000
EBIT(1 - T) -$ 540,000 -$ 750,000 $ 300,000 $ 90,000
Plus: Depreciation 1,650,000 2,250,000 750,000 350,000
Operating CF $1,110,000 $1,500,000 $1,050,000 $ 440,000
Recovery of NOWC 300,000
Net CF -$5,300,000 $1,110,000 $1,500,000 $1,050,000 $ 740,000

*An increase in inventories is a use of funds for the company, and an increase in accounts payable is a source of
funds for the company. Thus, the change in net operating working capital will be $200,000 - $500,000 = -$300,000
at time 0.

NPV = -$5,300,000 + $1,110,000/1.10 + $1,500,000/(1.10)2 + $1,050,000/(1.10)3 + $740,000/(1.10)4


NPV = -$5,300,000 + $1,009,091 + $1,239,669 + $788,881 + $505,430
NPV = -$1,756,929.
180
. New project NPV

Answer: a Diff: M N

0 1 2 3 4
Project cost ($500,000)
NOWC (40,000)

Sales $800,000 $800,000 $500,000 $500,000


Operating costs 480,000 480,000 300,000 300,000
Depreciation ________ 165,000 225,000 75,000 35,000
EBIT $155,000 $ 95,000 $125,000 $165,000
Taxes (40%) 62,000 38,000 50,000 66,000
EBIT(1 - T) $ 93,000 $ 57,000 $ 75,000 $ 99,000
Plus: Depreciation 165,000 225,000 75,000 35,000
Recovery of NOWC ________ ________ ________ ________ 40,000
Net CF ($540,000) $258,000 $282,000 $150,000 $174,000

NPV = -$540,000 + $258,000/1.10 + $282,000/(1.10)2 + $150,000/(1.10)3 + $174,000/(1.10)4


= -$540,000 + $234,545.45 + $233,057.85 + $112,697.22 + $118,844.34
= $159,144.86  $159,145.
181
. New project NPVAnswer: d Diff: T
Step 1: Calculate depreciation:
Dep 1 = 100,000(0.33) = 33,000.
Dep 2 = 100,000(0.45) = 45,000.
Dep 3 = 100,000(0.15) = 15,000.
Dep 4 = 100,000(0.07) = 7,000.

Step 2: Calculate cash flows:


CF 0 = -100,000 - 5,000 = -105,000.
CF 1 = 50,000 + 33,000 = 83,000.
CF 2 = 60,000 + 45,000 = 105,000.
CF 3 = 70,000 + 15,000 = 85,000.
CF 4 = 60,000 + 7,000 + 5,000 + 15,000 = 87,000.

Step 3: Calculate NPV:


Use CF key on calculator. Enter cash flows shown above. Enter I/YR =
12%. Solve for NPV = $168,604.

182
. New project NPVAnswer: d Diff: T
First, find the after-tax CFs associated with the project. This is
accomplished by subtracting the depreciation expense from the raw CF,
reducing this net CF by taxes and then adding back the depreciation expense.

For t = 1: ($45,000 - $33,000)(1 - 0.4) + $33,000 = $40,200.


Similarly, the after-tax CFs for t = 2, t = 3, and t = 4 are $45,000,
$33,000, and $29,800, respectively.

Now, enter these CFs along with the cost of the equipment to find the pre-
salvage NPV (note that the salvage value is not yet accounted for in these
CFs). The appropriate discount rate for these CFs is 11 percent. This yields
a pre-salvage NPV of $16,498.72.

Finally, the salvage value must be discounted. The PV of the salvage value
is: N = 4; I = 12; PMT = 0; FV = -10,000; and PV = $6,355.18. Adding the PV
of the salvage amount to the pre-salvage NPV yields the project NPV of
$22,853.90.
183

. New project NPVAnswer: d Diff: T


The cash flows for each of the years are as follows:
0 -100,000
1 [90,000 - 50,000 - (100,000)(0.20)](1 - 0.4) + (100,000)(0.20) = 32,000
2 [90,000 - 50,000 - (100,000)(0.32)](1 - 0.4) + (100,000)(0.32) = 36,800
3 [90,000 - 50,000 - (100,000)(0.19)](1 - 0.4) + (100,000)(0.19) = 31,600
4 [90,000 - 50,000 - (100,000)(0.12)](1 - 0.4) + (100,000)(0.12) = 28,800
5 [90,000 - 50,000 - (100,000)(0.11)](1 - 0.4) + (100,000)(0.11) = 28,400
6 [90,000 - 50,000 - (100,000)(0.06)](1 - 0.4) + (100,000)(0.06) +
(10,000)(1 - 0.4) = 32,400

Enter the cash flows and solve for the NPV = $38,839.56.
184

. New project NPVAnswer: c Diff: T


Get the depreciation using the MACRS table provided in the question.
0 1 2 3 4
Cost (500,000)
Inventory ( 50,000)
Accounts Payable 10,000
Sales 600,000 600,000 600,000 600,000
Operating Cost 400,000 400,000 400,000 400,000
Depreciation 165,000 225,000 75,000 35,000
EBIT 35,000 ( 25,000) 125,000 165,000
Taxes 14,000 ( 10,000) 50,000 66,000
EBIT(1 - T) 21,000 ( 15,000) 75,000 99,000
After-tax salvage value 30,000
Return of NOWC 40,000
+ Depreciation ________ 165,000 225,000 75,000 35,000
Net CF ($540,000) $186,000 $210,000 $150,000 $204,000

Note in year 4 your $40,000 of net operating working capital is recovered


plus the after tax salvage value of $30,000.

Enter the cash flows into the cash flow register and solve for the NPV using
the WACC of 10%. NPV = $54,676.59.
185
. Risk-adjusted NPV Answer: a Diff: M
Time lines:
Project A:
0 k = 14% 1 2 3 4 Years

CFsA -200,000 71,104 71,104 71,104 71,104


NPVA = ?
Project B:
0 k = 10% 1 2 3 4 Years

CFsB -200,000 0 0 146,411 146,411


NPVB = ?

Calculate required returns on A and B:


Project A High risk kRisk adjusted = 12% + 2% = 14%.
Project B Low risk kRisk adjusted = 12% - 2% = 10%.

Tabular solution:
NPVA = $71,104(PVIFA14%,4) - $200,000
= $71,104(2.9137) - $200,000 = $7,175.72.
NPVB = $146,411(PVIF10%,3) + $146,411(PVIF10%,4) - $200,000
= $146,411(0.7513) + $146,411(0.6830) - $200,000 = $9,997.30.
Project B has the higher NPV. Since they are mutually exclusive, select
Project B.

Financial calculator solution:


A Inputs: CF0 = -200,000; CF1 = 71,104; Nj = 4; I = 14.
Output: NPVA = $7,176.60  $7,177.
B Inputs: CF0 = -200,000; CF1 = 0; Nj = 2; CF2 = 146,411; Nj = 2; I = 10.
Output: NPVB = $10,001.43  $10,001.
Note: The difference in the NPVB between the tabular solution and financial
calculator cash flow solution of $4.13 is due to rounding of PVIF factors to
four significant decimals. Greater precision in the PVIF factors produces
identical answers.
186
. Risk-adjusted NPV Answer: e Diff: M
Time line:
0 k = 12% 1 2 10 Years
  
-50,000 6,000 6,000 6,000
NPV = ? Salvage value 10,000
CF10 = 16,000
kProject = 6% + 4%(1.5) = 12%.

Tabular solution:
NPV = -$50,000 + $6,000(PVIFA12%,10) + $10,000(PVIF12%,10)
= -$50,000 + $6,000(5.6502) + $10,000(0.3220)
= -$12,878.80  -$12,879.

Financial calculator solution:


Inputs: CF0 = -50,000; CF1 = 6,000; Nj = 9; CF10 = 16,000; I = 12%.
Output: NPV = -$12,878.93  -$12,879.
187
. Risk-adjusted NPV Answer: c Diff: M
Time lines:
Project A:
0 k = 12% 1 2 3 Periods

CFsA -5,000 2,000 2,500 2,250 NPVA


= ?

Project B:
0 k = 14% 1 2 3 Periods

CFsB -5,000 3,000 2,600 2,900 NPVB


= ?
Project A: kAverage risk = 12%.
Project B: kHigh risk = 12% + 2% = 14%.

Tabular solution:
NPVA = $2,000(PVIF12%,1) + $2,500(PVIF12%,2) + $2,250(PVIF12%,3) - $5,000
= $2,000(0.8929) + $2,500(0.7972) + $2,250(0.7118) - $5,000
= $380.35  $380.
NPVB = $3,000(PVIF14%,1) + $2,600(PVIF14%,2) + $2,900(PVIF14%,3) - $5,000
= $3,000(0.8772) + $2,600(0.7695) + $2,900(0.6750) - $5,000
= $1,589.80  $1,590.

Financial calculator solution:


A: Inputs: CF0 = -5,000; CF1 = 2,000; CF2 = 2,500; CF3 = 2,250; I% = 12.
Output: NPV = $380.20  $380.
B: Inputs: CF0 = -5,000; CF1 = 3,000; CF2 = 2,600; CF3 = 2,900; I% = 14.
Output: NPV = $1,589.61  $1,590.
188

. NPV and risk-adjusted discount rate Answer: e Diff: T N

The following table shows the cash flows:

0 1 2 3 4 5
Initial invest. outlay -$30.0
Sales $20.0 $20.0 $20.0 $20.0 $20.0
Oper. cost 12.0 12.0 12.0 12.0 12.0
Deprec. 10.0 10.0 10.0 0.0 0.0
EBIT -$ 2.0 -$ 2.0 -$ 2.0 $ 8.0 $ 8.0
Less: Taxes -0.8 -0.8 -0.8 3.2 3.2
EBIT(1 - T) -$ 1.2 -$ 1.2 -$ 1.2 $ 4.8 $ 4.8
Add back: Deprec. 10.0 10.0 10.0 0.0 0.0
NCF -$30.0 $ 8.8 $ 8.8 $ 8.8 $ 4.8 $ 4.8

Step 1: Determine the NPV of net cash flows:


NPV = -$30 + $8.8/1.10 + $8.8/(1.10)2 + $8.8/(1.10)3 + $4.8/(1.10)4 + $4.8/(1.10)5
= -$30 + $8 + $7.2727 + $6.6116 + $3.2785 + $2.9804
= -$1.8568 million.

Step 2: Determine the NPV of the project’s AT salvage value:


$1.2/(1.12)5 = $0.6809 million.

Step 3: Determine the project’s NPV:


Add the PV of the salvage value to the NPV of the cash flows to get the project’s NPV.
NPV = -$1.8568 + $0.6809 = -$1.1759 million  -$1.18 million.

Financial calculator solution:


Step 1: Determine the NPV of net cash flows:
Enter the following inputs in the calculator:
CF0 = -30, CF1-3 = 8.8, CF4-5 = 4.8, I = 10, and then solve for NPV = -$1.8568 million.

Step 2: Determine the NPV of the project’s AT salvage value:


Enter the following inputs in the calculator:
CF0 = 0, CF1-4 = 0, CF5 = 1.2, I = 12, and then solve for NPV = $0.6809 million.

Step 3: Determine the project’s NPV:


Add the PV of the salvage value to the NPV of the cash flows to get the project’s NPV.
-$1.8568 + $0.6809 = -$1.1759 million  $-1.18 million.
189

. Discounting risky outflows Answer: e Diff: M


Time line:
0 1 2 3 4
| | | | |
CFsNew Tech -1,500 -315 -315 -315 -315
NPVNew Tech = ?
CFsOld Tech -600 -600 -600 -600 -600
NPVOld Tech = ?
Recognize that (1) risk outflows must be discounted at lower rates, and (2)
since Project New Tech is risky, it must be discounted at a rate of 12% - 3%
= 9%. Project Old Tech must be discounted at 12%.
Tabular solution:
PVNew Tech = -$1,500 - $315(PVIFA9%,4) = -$1,500 - $315(3.2397)
= -$2,520.51.
PVOld Tech = -$600 - $600(PVIFA12%,4) = -$600 - $600(3.0373) = -$2,422.38.
PVOld Tech is a smaller outflow than NPVNew Tech, thus, Project Old Tech is the
better project.
Financial calculator solution:
Project New Tech: Inputs: CF0 = -1,500; CF1 = -315; Nj = 4; I = 9.
Output: NPV = -$2,520.51.
Project Old Tech: Inputs: CF0 = -600; CF1 = -600; Nj = 4; I = 12.
Output: NPV = -$2,422.41.
190

. Risky Projects Answer: d Diff: M


Look at the NPV, IRR, and hurdle rate for each project:
Project A B C D E
Hurdle 9.00% 9.00% 11.00% 13.00% 13.00%
NPV $13,822 $11,998
IRR 12.11% 14.04% 10.85% 16.64% 11.63%

Projects A and B are mutually exclusive, so we pick project A because it has


the largest NPV. Projects C, D, and E are independent so we pick the ones
whose IRR exceeds the cost of capital, in this case, just D. Therefore, the
projects undertaken are A and D.
191

. Scenario analysis Answer: c Diff: M


Calculate expected value of NPV:
Probability of Unit Sales Sales NPV
Outcome, Pi Volume Price (In 1000s) Pi(x)
Worst case 0.30 6,000 $3,600 -$6,000 0.3(-6,000) = -1,800
Base case 0.50 10,000 4,200 13,000 0.5(13,000) = 6,500
Best case 0.20 13,000 4,400 28,000 0.2(28,000) = 5,600
Expected NPV = $10,300

Calculate standard deviation of NPV (in thousands):


Pi(x - x )2 (x - x )2 Pi(x - x )2
_____________________________________ ____________________ _________________________

Worst case 0.3(-6 - 10.3)2 265.69 79.707


Base case 0.5(13 - 10.3)2 7.29 3.645
Best case 0.2(28 - 10.3)2 313.29 62.658
Sum 146.01
(146.01)½ = 12,083 thousand.

Calculate coefficient of variation (CV) of NPV:


CVNPV = σNPV/E(NPV) = $12,083/$10,300 = 1.17.
192
. New project investment Answer: a Diff: E
Initial investment:
Cost ($40,000)
Change in NOWC (2,000)
($42,000)
193

. Operating cash flow Answer: e Diff: M


Depreciation schedule:
Depreciable basis = $40,000.
MACRS
Depreciation Depreciable Annual
Year Rate Basis Depreciation
1 0.33 $40,000 $13,200
2 0.45 40,000 18,000
3 0.15 40,000 6,000
4 0.07 40,000 2,800
$40,000

Operating cash flows:


Year 1 2 3
1) Increase in revenues $20,000 $20,000 $20,000
2) Increase in costs (5,000) (5,000) (5,000)
3) Before-tax change in
earnings 15,000 15,000 15,000
4) After-tax change in
earnings (line 3  0.60) 9,000 9,000 9,000
5) Depreciation 13,200 18,000 6,000
6) Tax savings deprec.
(line 6  0.40) 5,280 7,200 2,400
7) Net operating CFs
(line 4 + 6) $14,280 $16,200 $11,400
194
. Non-operating cash flows Answer: a Diff: M
Additional Year 3 cash flows:
3
Salvage value $25,000
Tax on Salvage value (8,880)*
Recovery of NOWC 2,000
$18,120

*(Market value - Book value)(Tax rate)


($25,000 - $2,800)(0.40) = $8,880.
195
. New project NPVAnswer: c Diff: M
Time line:
0 k = 14% 1 2 3 Years

-42,000 14,280 16,200 11,400


TV = 18,120
29,520
Tabular solution:
NPVk = 14% = -$42,000 + $14,280(PVIF14%,1)
+ $16,200(PVIF14%,2) + $29,520(PVIF14%,3)
= -$42,000 + $14,280(0.8772) + $16,200(0.7695)
+ $29,520(0.6750) = $2,918.32.

Financial calculator solution:


Inputs: CF0 = -42,000; CF1 = 14,280; CF2 = 16,200; CF3 = 29,520; I = 14.
Output: NPV = $2,916.85  $2,917.

Note: Tabular solution differs from calculator solution due to interest


factor rounding.
196

. New project investment Answer: d Diff: E


Initial investment:
Cost ($50,000)
Modification (10,000)
Change in NOWC (2,000)
Total net investment = ($62,000)
197

. Operating cash flow Answer: c Diff: M


Depreciation schedule:
Depreciable basis = $60,000.
MACRS
Depreciation Depreciable Annual
Year Rate Basis Depreciation
1 0.33 $60,000 $19,800
2 0.45 60,000 27,000
3 0.15 60,000 9,000
4 0.07 60,000 4,200
$60,000

Operating cash flows:


Year 1 2 3
1) Before-tax cost reduction $20,000 $20,000 $20,000
2) After-tax cost reduction
(line 1  0.6) 12,000 12,000 12,000
3) Depreciation 19,800 27,000 9,000
4) Tax savings from deprec.
(line 3  0.4) 7,920 10,800 3,600
5) Net operating CFs $19,920 $22,800 $15,600
198
. Non-operating cash flows Answer: c Diff: M
Additional Year 3 cash flows:
3
Salvage value $20,000
Tax on salvage value (6,320)*
Recovery of NOWC 2,000
Total terminal year CF $15,680

*(Market value - Book value)(Tax rate)


($20,000 - $4,200)(0.40) = $6,320.
199

. New project NPVAnswer: a Diff: M


Time line:
0 k = 10% 1 2 3 Years

-62,000 19,920 22,800 15,600


TV = 15,680
31,280
Tabular solution:
NPVk = 10% = -$62,000 + $19,920(PVIF10%,1) + $22,800(PVIF10%,2)
+ $31,280(PVIF10%,3)
= -$62,000 + $19,920(0.9091) + $22,800(0.8264)
+ $31,280(0.7513) = -$1,548.14.

Financial calculator solution:


Inputs: CF0 = -62,000; CF1 = 19,920; CF2 = 22,800; CF3 = 31,280; I = 10.
Output: NPV = -$1,546.81  -$1,547.

Note: Tabular solution differs from calculator solution due to interest


factor rounding.

You might also like